Day-536 | Daily MCQs | UPSC Prelims | ECONOMY

Day-536

Time limit: 0

Quiz-summary

0 of 5 questions completed

Questions:

  1. 1
  2. 2
  3. 3
  4. 4
  5. 5

Information

To attempt the Quiz, simply click on START Button.

You have already completed the quiz before. Hence you can not start it again.

Quiz is loading...

You must sign in or sign up to start the quiz.

You have to finish following quiz, to start this quiz:

Results

0 of 5 questions answered correctly

Your time:

Time has elapsed

You have reached 0 of 0 points, (0)

Categories

  1. Not categorized 0%
  1. 1
  2. 2
  3. 3
  4. 4
  5. 5
  1. Answered
  2. Review
  1. Question 1 of 5
    1. Question

    1. Consider the following statements regarding ‘Accommodating transactions’
    under Balance of Payments (BoP):

    1. It refers to those transactions which are made independently of the state of
    the BoP.
    2. The official reserve transactions are considered as the accommodating item in
    the BoP.
    3. It compensates for the surplus or deficit brought by autonomous transactions.

    How many of the above statements are correct ?

    Correct

    Answer: B
    Explanation:

    Accommodating transactions:
    ● Statement 1 is incorrect: Accommodating transactions are also termed as the ‘below the line’
    items. It implies those economic transactions carried out with the purpose of meeting the gap
    in autonomous transactions. They are determined by the net consequences of the
    autonomous items, that is, whether the BoP is in surplus or deficit.
    ● Statement 2 is correct: The official reserve transactions are seen as the accommodating item
    in the BoP .
    ● Statement 3 is correct: Accommodating transactions compensate for the surplus or deficit
    brought about by autonomous transactions. It seeks to bring equality between the payments
    and receipts of foreign exchange.
    ● These transactions are carried out for maintaining the balance in the Balance of Payment
    Account. These transactions fall in the capital account only.
    ● These transactions happen in the Government sector only.
    Autonomous transaction:
    ● International economic transactions are called autonomous when transactions are made
    independently of the state of the BoP (for instance due to profit motive).
    ● Autonomous transactions will include imports and exports (current account transactions) and
    also lending or borrowing of loans (cross border) or payment or receipt of interests thereon.

    Incorrect

    Answer: B
    Explanation:

    Accommodating transactions:
    ● Statement 1 is incorrect: Accommodating transactions are also termed as the ‘below the line’
    items. It implies those economic transactions carried out with the purpose of meeting the gap
    in autonomous transactions. They are determined by the net consequences of the
    autonomous items, that is, whether the BoP is in surplus or deficit.
    ● Statement 2 is correct: The official reserve transactions are seen as the accommodating item
    in the BoP .
    ● Statement 3 is correct: Accommodating transactions compensate for the surplus or deficit
    brought about by autonomous transactions. It seeks to bring equality between the payments
    and receipts of foreign exchange.
    ● These transactions are carried out for maintaining the balance in the Balance of Payment
    Account. These transactions fall in the capital account only.
    ● These transactions happen in the Government sector only.
    Autonomous transaction:
    ● International economic transactions are called autonomous when transactions are made
    independently of the state of the BoP (for instance due to profit motive).
    ● Autonomous transactions will include imports and exports (current account transactions) and
    also lending or borrowing of loans (cross border) or payment or receipt of interests thereon.

  2. Question 2 of 5
    2. Question

    Recently Gresham’s Law was in the news. Which of the following statements
    is correct about it?

    Correct

     Answer: C

    Explanation:
    ● Sir Thomas Gresham lived from 1519 to 1579 and wrote about the value and minting of coins
    while working as a financier.
    ● It states that, in a fixed exchange rate system, “bad money drives out good”.
    ● It comes into play when the exchange rate between two currencies is fixed by the government
    at a certain ratio that is different from market exchange rate.
    Or
    ● When the price of a currency in terms of another currency is arbitrarily fixed by lawmakers, it
    leads to a shortage in the supply of the currency that is undervalued, while there is at the
    same time an over-supply of the currency that is overvalued.
    ● The law observes that legally overvalued currency will drive legally undervalued currency out
    of circulation.

    Incorrect

     Answer: C

    Explanation:
    ● Sir Thomas Gresham lived from 1519 to 1579 and wrote about the value and minting of coins
    while working as a financier.
    ● It states that, in a fixed exchange rate system, “bad money drives out good”.
    ● It comes into play when the exchange rate between two currencies is fixed by the government
    at a certain ratio that is different from market exchange rate.
    Or
    ● When the price of a currency in terms of another currency is arbitrarily fixed by lawmakers, it
    leads to a shortage in the supply of the currency that is undervalued, while there is at the
    same time an over-supply of the currency that is overvalued.
    ● The law observes that legally overvalued currency will drive legally undervalued currency out
    of circulation.

  3. Question 3 of 5
    3. Question

    Consider the following sectors:
    1. Drones and Drone Components
    2. White Goods
    3. Toys
    4. Specialty Steel
    5. Cottage industry
    6. Automobiles and Auto Components
    7. Leather and footwear
    How many of the above sectors are covered under the Production Linked
    Incentive Schemes?

    Correct

    3. Answer: A
    Explanation:
    ● Production Linked Incentive Scheme (PLI) was launched in April , 2020. PLI offers a production
    linked incentive to boost domestic manufacturing and attract large investments in various
    sectors.
    ● The scheme shall extend an incentive of 4% to 6% on incremental sales (over base year) of
    goods manufactured in India and covered under target segments, to eligible companies, for a
    period of five (5) years subsequent to the base year as defined.
    ● The Scheme is open for applications for a period of 4 months initially which may be extended.
    Support under the Scheme shall be provided for a period of five (5) years subsequent to the
    base year

    Incorrect

  4. Question 4 of 5
    4. Question

    4. Consider the following pairs:
    Terms Descriptions
    1. Tax Base – It is the total number of people who pay tax in the

    economy.

    2. Tax Buoyancy – It measures the responsiveness of tax revenue to changes

    in tax rate.

    3. Tax Elasticity – It is an indicator to measure efficiency and responsiveness
    of revenue mobilization in response to growth in the GDP

    How many of the above pairs are correctly matched?

    Correct

    4. Answer: D
    Explanation:
    ● Tax Base: Tax base is defined as the total value of the financial streams or assets on which
    tax can be imposed by the government. For example, in case of income tax, the tax base is
    all the income that can be taxed by the government (taxable income). If the minimum
    amount/threshold of the income on which tax is imposed is lowered, this will automatically
    increase (widen) the tax base; if it is raised, the tax base will be narrowed. In case of service
    tax, tax base is the value of services on which service tax is imposed. For example, if CCD sells
    tea/coffee worth Rs. 10 crore and of course it collects service tax/GST on 10 crores, so tax
    base will be Rs. 10 crores. In case of property tax, tax base is the value of property on which
    property tax is imposed. Because the size of the tax base influences the taxable revenues that
    are available to a government, the size and growth of the tax base is crucial to the planning
    efforts of any government.
    ● Tax Buoyancy: Tax buoyancy is an indicator to measure efficiency and responsiveness of
    revenue mobilization in response to growth in the GDP or National Income. It is measured
    as a ratio of growth in Tax Revenue to the growth in nominal GDP (i.e., ratio of percentage
    change in tax revenue to percentage change in nominal GDP). Tax buoyancy greater than one
    is good for the economy.
    ● Tax Elasticity: The tax elasticity measures the responsiveness of tax revenue to changes in tax
    rate and is defined as the ratio of percentage change in tax revenue to percentage change in
    tax rate.

    Incorrect

  5. Question 5 of 5
    5. Question

    5. Consider the following:
    1. Capital gains tax
    2. Stamp duty
    3. Property tax
    4. Securities transaction tax
    5. Luxury tax
    6. Equalization levy
    How many of the above are direct tax?

    Correct

    5. Answer: B
    Explanation:
    ● A direct tax is a tax that a person or organization pays directly to the entity that imposed it.
    Examples include income tax, real property tax, personal property tax, and taxes on assets, all
    of which are paid by an individual taxpayer directly to the government.
    ● List of direct tax:

    THE PT QUEST 2024 LUKMAAN IAS

    o Income Tax
    o Corporate Tax
    o Securities Transaction Tax
    o Capital Gains Tax
    o Gift Tax
    o Wealth Tax
    o Land revenue
    o Property tax
    o Dividend distribution tax
    o Equalization levy
    ● Indirect tax is the tax levied on the consumption of goods and services. It is not directly levied
    on the income of a person. Instead, he/she has to pay the tax along with the price of goods or
    services bought by the seller.
    ● List of indirect tax:
    o Sales Tax
    o Value Added Tax (VAT)
    o Goods & Services Tax (GST)
    o Luxury tax
    o Stamp duty
    o Custom Duty

    Incorrect

window.wpAdvQuizInitList = window.wpAdvQuizInitList || []; window.wpAdvQuizInitList.push({ id: '#wpAdvQuiz_575', init: { quizId: 575, mode: 0, globalPoints: 10, timelimit: 0, resultsGrade: [0], bo: 0, qpp: 0, catPoints: [10], formPos: 0, lbn: "Finish quiz", json: {"2668":{"type":"single","id":2668,"catId":0,"points":2,"correct":[0,1,0,0]},"2669":{"type":"single","id":2669,"catId":0,"points":2,"correct":[0,0,1,0]},"2670":{"type":"single","id":2670,"catId":0,"points":2,"correct":[1,0,0,0]},"2671":{"type":"single","id":2671,"catId":0,"points":2,"correct":[0,0,0,1]},"2672":{"type":"single","id":2672,"catId":0,"points":2,"correct":[0,1,0,0]}} } });




Day-537 | Daily MCQs | UPSC Prelims | HISTORY

Day-537

Time limit: 0

Quiz-summary

0 of 5 questions completed

Questions:

  1. 1
  2. 2
  3. 3
  4. 4
  5. 5

Information

To attempt the Quiz, simply click on START Button.

You have already completed the quiz before. Hence you can not start it again.

Quiz is loading...

You must sign in or sign up to start the quiz.

You have to finish following quiz, to start this quiz:

Results

0 of 5 questions answered correctly

Your time:

Time has elapsed

You have reached 0 of 0 points, (0)

Categories

  1. Not categorized 0%
Your result has been entered into leaderboard
Loading
captcha
  1. 1
  2. 2
  3. 3
  4. 4
  5. 5
  1. Answered
  2. Review
  1. Question 1 of 5
    1. Question
    2 points

    1. With reference to the history of India, the term ‘suyurghals’ refers to:

    Correct

    Answer: B
    Explanation:

    ‘Suyurghals’ is the practice of granting revenue producing lands to scholars, theologians and
    others by the Mughal emperors which is tax free.
    It was an Mongolian tradition. The suyurghals holder was free from any administrative or
    judicial interference from the central government. There was no compulsory obligation to
    provide for military contingents.

    Incorrect

    Answer: B
    Explanation:

    ‘Suyurghals’ is the practice of granting revenue producing lands to scholars, theologians and
    others by the Mughal emperors which is tax free.
    It was an Mongolian tradition. The suyurghals holder was free from any administrative or
    judicial interference from the central government. There was no compulsory obligation to
    provide for military contingents.

  2. Question 2 of 5
    2. Question
    2 points

    2. Consider the following statements in the context of Mughals:
    1. They traced their paternal ancestry from Chingiz Khan and maternal ancestry
    from Timur.
    2. They followed the principle of primoginature in the inheritance of the throne.
    Which of the statements given above is/are correct?

    Correct

    Answer: D
    Explanation:
    Statement 1 is incorrect: The Mughals in India proudly traced their ancestry to both Chingiz
    and Timur. In Babur Nama, emperor Babur talked of Chingiz, an ancestor of his mother and
    Timur, his paternal ancestor.
    Statement 2 is incorrect: The Mughals adopted the Timurid ‘appanage system’ or the
    principle of heritable division of territory. It means the entire empire will be divided amongst
    all the sons. Primoginature is the system in which the eldest son acquires the throne.

    Incorrect

  3. Question 3 of 5
    3. Question
    2 points

    Consider the following pairs:

    Kingdoms                  Dynasties
    1. Bijapur                    Barid Shahi
    2. Berar                      Imad Shahi
    3. Golconda               Nizam Shahi
    4. Bidar                        Qutb Shahi

    How many pairs given above are correctly matched?

    Correct

     Answer: A
    Explanation:
    During the reign of Vijaynagara’s greatest ruler Krishnadeva Raya (1509-29), the power of the
    Bahmanis declined, leading to the emergence of five kingdoms:
    ● the Nizam Shahis of Ahmadnagar;
    ● the Adil Shahis of Bijapur;
    ● the Imad Shahis of Berar,
    ● the Qutb Shahis of Golconda and
    ● the Barid Shahis of Bidar

    Incorrect

  4. Question 4 of 5
    4. Question
    2 points

    4. Consider the following statements about the idea of Dvaitadvaita:
    1. It was given by Ramanuja against the philosophy of maya by Shankara.
    2. The idea emphasises the identicality and the distinctness of the Supreme soul,
    individual soul and the inanimate world.
    Which of the statements given above is/are correct?

    Correct

    Answer: B
    Explanation:
    Statement 1 is incorrect: Nimbarka (12th century), a Telegu Brahman, propounded the
    idea of Dvaitadvaita (bheda-bheda; dualistic-nondualism),against Shankara’s philosophy of
    maya (illusion).
    Statement 2 is correct: Nimbarka emphasizes that the Supreme soul (God) and individual
    soul and the inanimate world are both ‘identical’ (monism) and at the same time were
    ‘distinct’ (dualism). ‘They are identical in the sense that the individual soul and the
    inanimate world are entirely dependent on God and have no independent existence’. Thus
    Nimbarka’s philosophy is both ‘monistic and pluralistic’.

    Incorrect

  5. Question 5 of 5
    5. Question
    2 points

    5. Consider the following pairs:

    Regions                         Known in ancient times

    1. Godavari basin                    Asmaka
    2. Krishna river valley           Kuntala
    3. Konkan                                Aparanta
    How many pairs given above are correctly matched?

    Correct

    Answer: C
    Explanation:

    The land south of the river Narmada was called by the Aryan settlers as Dakshinapatha and
    included the Dandakaranya which was so named from the vast forest that stretched
    southwards from the Tapti to the Godavari. During the early centuries of the common era
    Mahārāshtra consisted of three distinct portions, viz. the first, Vidarbha or Berar; the
    second, Asmaka or the Godavari basin (later known as Seuna Desh); and the third Kuntala,
    that is the valley of the river Krishnā. It also embraced the western coastal region known as
    Aparänta or Konkan, stretching from Daman in the north to Goa or even Karwar in the
    south.

    Incorrect

window.wpAdvQuizInitList = window.wpAdvQuizInitList || []; window.wpAdvQuizInitList.push({ id: '#wpAdvQuiz_576', init: { quizId: 576, mode: 0, globalPoints: 10, timelimit: 0, resultsGrade: [0], bo: 0, qpp: 0, catPoints: [10], formPos: 0, lbn: "Finish quiz", json: {"2674":{"type":"single","id":2674,"catId":0,"points":2,"correct":[0,1,0,0]},"2675":{"type":"single","id":2675,"catId":0,"points":2,"correct":[0,0,0,1]},"2676":{"type":"single","id":2676,"catId":0,"points":2,"correct":[1,0,0,0]},"2677":{"type":"single","id":2677,"catId":0,"points":2,"correct":[0,1,0,0]},"2678":{"type":"single","id":2678,"catId":0,"points":2,"correct":[0,0,1,0]}} } });




Day-535 | Daily MCQs | UPSC Prelims | GEOGRAPHY

Day-535

Time limit: 0

Quiz-summary

0 of 5 questions completed

Questions:

  1. 1
  2. 2
  3. 3
  4. 4
  5. 5

Information

DAILY MCQ

You have already completed the quiz before. Hence you can not start it again.

Quiz is loading...

You must sign in or sign up to start the quiz.

You have to finish following quiz, to start this quiz:

Results

0 of 5 questions answered correctly

Your time:

Time has elapsed

You have reached 0 of 0 points, (0)

Categories

  1. Not categorized 0%
  1. 1
  2. 2
  3. 3
  4. 4
  5. 5
  1. Answered
  2. Review
  1. Question 1 of 5
    1. Question

    1. Which of the following institutions undertakes landslide susceptibility mapping in India?

    Correct

    Answer: D
    Explanation:
    GSI undertakes a national programme on landslide susceptibility mapping.
    About Geological Survey of India (GSI):
    ● GSI is a scientific agency of India.
    ● It was founded in 1851, as a Government of India organization under the Ministry of Mines, one of the oldest of such organisations in the world and the second oldest survey in India after the Survey of India (founded in 1767), for conducting geological surveys and studies of India, and also as the prime provider of basic earth science information to government, industry and general public, as well as the official participant in steel, coal, metals, cement, power industries and international geo-scientific forums.
    ● Headquartered in Kolkata, GSI has six regional offices at Lucknow, Jaipur, Nagpur, Hyderabad, Shillong and Kolkata and state unit offices in almost all the states of the country.

    Incorrect

    Answer: D
    Explanation:
    GSI undertakes a national programme on landslide susceptibility mapping.
    About Geological Survey of India (GSI):
    ● GSI is a scientific agency of India.
    ● It was founded in 1851, as a Government of India organization under the Ministry of Mines, one of the oldest of such organisations in the world and the second oldest survey in India after the Survey of India (founded in 1767), for conducting geological surveys and studies of India, and also as the prime provider of basic earth science information to government, industry and general public, as well as the official participant in steel, coal, metals, cement, power industries and international geo-scientific forums.
    ● Headquartered in Kolkata, GSI has six regional offices at Lucknow, Jaipur, Nagpur, Hyderabad, Shillong and Kolkata and state unit offices in almost all the states of the country.

  2. Question 2 of 5
    2. Question

    2. Consider the following pairs:
    Nuclear Plants – States
    1. Tarapur – Maharashtra
    2. Rawatbhata – Rajasthan
    3. Kalpakkam – Karnataka
    4. Kaiga – Tamil Nadu
    How many of the pairs given above are correctly matched?

    Correct

    Answer: B
    Explanation:
    Only pairs 1 and 2 are correctly matched:
    ● Atomic Energy Commission was established in 1948, progress could be made only after the establishment of the Atomic Energy Institute at Trombay in 1954 which was renamed as the Bhabha Atomic Research Centre in 1967.
    The important nuclear power projects are Tarapur (Maharashtra), Rawatbhata near Kota (Rajasthan), Kalpakkam (Tamil Nadu), Narora (Uttar Pradesh), Kaiga (Karnataka) and
    Kakarapara (Gujarat).

    Incorrect

    Answer: B
    Explanation:
    Only pairs 1 and 2 are correctly matched:
    ● Atomic Energy Commission was established in 1948, progress could be made only after the establishment of the Atomic Energy Institute at Trombay in 1954 which was renamed as the Bhabha Atomic Research Centre in 1967.
    The important nuclear power projects are Tarapur (Maharashtra), Rawatbhata near Kota (Rajasthan), Kalpakkam (Tamil Nadu), Narora (Uttar Pradesh), Kaiga (Karnataka) and
    Kakarapara (Gujarat).

  3. Question 3 of 5
    3. Question

    3. Consider the following statements about quinary activities:
    1. The highest level of decision-makers or policymakers performs quinary activities.
    2. Banking and Legal Advisory are the prime examples of quinary activities.
    Which of the statements given above is/are correct?

    Correct

    Answer: C
    Explanation:
    ● Quinary activities are services that focus on the creation, re-arrangement, and interpretation of new and existing ideas; data interpretation, and the use and evaluation of new technologies.
    ● Often referred to as ‘gold collar’ professions, they represent another subdivision of the tertiary sector representing special and highly paid skills of senior business executives, government officials, research scientists, financial and legal consultants, etc.
    ● Their importance in the structure of advanced economies far outweighs their numbers.
    Statement 1 is correct:
    ● The highest level of decision-makers or policymakers performs quinary activities.
    ● These are subtly different from the knowledge-based industries that the quinary sector in general deals with.
    Statement 2 is correct:
    ● New trends in quinary services include knowledge processing outsourcing (KPO) and ‘home shoring’, the latter as an alternative to outsourcing.
    ● The KPO industry is distinct from Business Process Outsourcing (BPO) as it involves highly skilled workers.
    ● It is information-driven knowledge outsourcing. KPO enables companies to create additional business opportunities.
    ● Examples of KPOs include research and development (R and D) activities, e-learning, business research, intellectual property (IP) research, the legal profession, and the banking sector.

    Incorrect

    Answer: C
    Explanation:
    ● Quinary activities are services that focus on the creation, re-arrangement, and interpretation of new and existing ideas; data interpretation, and the use and evaluation of new technologies.
    ● Often referred to as ‘gold collar’ professions, they represent another subdivision of the tertiary sector representing special and highly paid skills of senior business executives, government officials, research scientists, financial and legal consultants, etc.
    ● Their importance in the structure of advanced economies far outweighs their numbers.
    Statement 1 is correct:
    ● The highest level of decision-makers or policymakers performs quinary activities.
    ● These are subtly different from the knowledge-based industries that the quinary sector in general deals with.
    Statement 2 is correct:
    ● New trends in quinary services include knowledge processing outsourcing (KPO) and ‘home shoring’, the latter as an alternative to outsourcing.
    ● The KPO industry is distinct from Business Process Outsourcing (BPO) as it involves highly skilled workers.
    ● It is information-driven knowledge outsourcing. KPO enables companies to create additional business opportunities.
    ● Examples of KPOs include research and development (R and D) activities, e-learning, business research, intellectual property (IP) research, the legal profession, and the banking sector.

  4. Question 4 of 5
    4. Question

    4. Which of the following geographical factors have made the Steppe regions, ‘the granaries of the world’?
    1. Cool and moist climate during the spring season helps in the growth of wheat.
    2. Warm and sunny summer is advantageous during the harvesting period.
    Select the correct answer using the code given below:

    Correct

    Answer: C
    Explanation:
    Statement 1 is correct:
    ● The temperate grasslands are ideal for extensive wheat cultivation. Here winter wheat is grown and in the months of spring, the climate is cool and moist that helps in the early growth of wheat.
    Statement 2 is correct:
    ● The light showers in the ripening period i.e at the beginning of summer months help to swell the grains to ensure a good yield.
    ● The warm and sunny summer is advantageous for harvesting and also enables the straw to be dried for farm use.
    ● In addition to this, the levelness of the Steppes and other temperate grasslands all over the world along with the widespread black earth soil makes mechanized ploughing and harvesting a comparatively easy job.

    Incorrect

    Answer: C
    Explanation:
    Statement 1 is correct:
    ● The temperate grasslands are ideal for extensive wheat cultivation. Here winter wheat is grown and in the months of spring, the climate is cool and moist that helps in the early growth of wheat.
    Statement 2 is correct:
    ● The light showers in the ripening period i.e at the beginning of summer months help to swell the grains to ensure a good yield.
    ● The warm and sunny summer is advantageous for harvesting and also enables the straw to be dried for farm use.
    ● In addition to this, the levelness of the Steppes and other temperate grasslands all over the world along with the widespread black earth soil makes mechanized ploughing and harvesting a comparatively easy job.

  5. Question 5 of 5
    5. Question

    5. In the context of economic geography, Ports of Call are:

    Correct

    Answer: D
    Explanation:
    Ports are classified on the basis of various parameters like the types of traffic which they handle, the cargo handled, on the basis of location, and on the basis of specialized functions performed.
    The types of the port on the basis of specialized functions are:
    Option D is correct:
    ● Oil ports: Deal in processing and shipping of oil eg: Tripoli in Lebanon.
    ● Ports of Call: These are stock replenishment ports where ships anchor for refueling, watering and taking food items. eg. Aden, now developed as a commercial port.
    ● Packet Stations: These are concerned with the transportation of passengers and mail across water bodies covering short distances. E.g. Dover in England.
    ● Entrepot Ports: These are collection centers where the goods are brought from different countries for export. E.g. Singapore.
    ● Naval Ports: They serve warships and have repair workshops for them. eg: Kochi

    Incorrect

    Answer: D
    Explanation:
    Ports are classified on the basis of various parameters like the types of traffic which they handle, the cargo handled, on the basis of location, and on the basis of specialized functions performed.
    The types of the port on the basis of specialized functions are:
    Option D is correct:
    ● Oil ports: Deal in processing and shipping of oil eg: Tripoli in Lebanon.
    ● Ports of Call: These are stock replenishment ports where ships anchor for refueling, watering and taking food items. eg. Aden, now developed as a commercial port.
    ● Packet Stations: These are concerned with the transportation of passengers and mail across water bodies covering short distances. E.g. Dover in England.
    ● Entrepot Ports: These are collection centers where the goods are brought from different countries for export. E.g. Singapore.
    ● Naval Ports: They serve warships and have repair workshops for them. eg: Kochi

window.wpAdvQuizInitList = window.wpAdvQuizInitList || []; window.wpAdvQuizInitList.push({ id: '#wpAdvQuiz_574', init: { quizId: 574, mode: 0, globalPoints: 10, timelimit: 0, resultsGrade: [0], bo: 0, qpp: 0, catPoints: [10], formPos: 0, lbn: "Finish quiz", json: {"2663":{"type":"single","id":2663,"catId":0,"points":2,"correct":[0,0,0,1]},"2664":{"type":"single","id":2664,"catId":0,"points":2,"correct":[0,1,0,0]},"2665":{"type":"single","id":2665,"catId":0,"points":2,"correct":[0,0,1,0]},"2666":{"type":"single","id":2666,"catId":0,"points":2,"correct":[0,0,1,0]},"2667":{"type":"single","id":2667,"catId":0,"points":2,"correct":[0,0,0,1]}} } });




Day-534 | Daily MCQs | UPSC Prelims | POLITY

Day-534

Time limit: 0

Quiz-summary

0 of 5 questions completed

Questions:

  1. 1
  2. 2
  3. 3
  4. 4
  5. 5

Information

DAILY MCQ

You have already completed the quiz before. Hence you can not start it again.

Quiz is loading...

You must sign in or sign up to start the quiz.

You have to finish following quiz, to start this quiz:

Results

0 of 5 questions answered correctly

Your time:

Time has elapsed

You have reached 0 of 0 points, (0)

Categories

  1. Not categorized 0%
  1. 1
  2. 2
  3. 3
  4. 4
  5. 5
  1. Answered
  2. Review
  1. Question 1 of 5
    1. Question

    1. Consider the following statements about the Deputy Speaker of the Lok Sabha:
    1. He/she is nominated by the Speaker of the Lok Sabha in consultation with the President.
    2. He/she cannot perform the functions of the office of the Speaker when it is vacant.
    3. He/she cannot preside over the joint sitting of the Houses of the Parliament.
    How many of the above given statements are correct?

    Correct

    Answer: D
    Explanation:
    Statement 1 is incorrect: The Deputy Speaker of the Lok Sabha is elected by the members from amongst its members and is not nominated by the Speaker of the Lok Sabha. Though the date of election of the Deputy Speaker is fixed by the Speaker of the Lok Sabha.
    Statement 2 is incorrect: As per Article 95 of the Constitution of India, the Deputy Speaker performs the duties of the Speaker’s office when it is vacant and acts as the Speaker when the latter is absent from the sitting of the House. In both the cases, he assumes all the powers of the Speaker.
    Statement 3 is incorrect: In case the Speaker is absent the Deputy Speaker can preside over the joint sitting of the Houses of the Parliament.

    Incorrect

    Answer: D
    Explanation:
    Statement 1 is incorrect: The Deputy Speaker of the Lok Sabha is elected by the members from amongst its members and is not nominated by the Speaker of the Lok Sabha. Though the date of election of the Deputy Speaker is fixed by the Speaker of the Lok Sabha.
    Statement 2 is incorrect: As per Article 95 of the Constitution of India, the Deputy Speaker performs the duties of the Speaker’s office when it is vacant and acts as the Speaker when the latter is absent from the sitting of the House. In both the cases, he assumes all the powers of the Speaker.
    Statement 3 is incorrect: In case the Speaker is absent the Deputy Speaker can preside over the joint sitting of the Houses of the Parliament.

  2. Question 2 of 5
    2. Question

    2. Consider the following statements:
    1. Pre-legislative review is a process where a bill is scrutinized by the political branches of government prior to enactment to assess its constitutionality or rights-compliance.
    2. Pre-legislative review ensures better protection for constitutional rights than stand-alone judicial review.
    3. Examination by a parliamentary committee is an informal way of pre-legislative review.
    How many of the above given statements are correct?

    Correct

    Answer: B
    Explanation:
    Statements 1 and 2 are correct: Pre-legislative review is a process where a bill is scrutinized by the political branches of government prior to enactment to assess its constitutionality or rights-compliance, thereby increasing constitutional engagement by the political branches of government.
    This can enhance constitutional culture in three ways: first, pre-legislative review ensures better protection for constitutional rights than stand-alone judicial review because it encourages the political branches to legislate in a more rights-conscious manner; second, it democratizes access to constitutional rights and values by placing them beyond the gamut of an unelected judiciary, and into the political realm; third, the review provides broader avenues for constitutional scrutiny of legislation, since it is usually supplemented by judicial review.
    Pre-legislative review assists in overcoming the limitations of judicial review. The expansion of the scope of review by an unelected judiciary raises concerns regarding constitutional and democratic legitimacy. For instance, judicial review, unlike the legislative process, results in restricted public participation in the policy-formulation process since the diverse viewpoints of relevant stakeholders remain unaccounted for before the judicial stage.
    Statement 3 is incorrect: Parliamentary Committees draw their authority from Article 105 and 118 of the Constitution of India. They are the formal ways for the political branches to engage with the constitutionality of legislation at the pre-legislative stage.

    Incorrect

    Answer: B
    Explanation:
    Statements 1 and 2 are correct: Pre-legislative review is a process where a bill is scrutinized by the political branches of government prior to enactment to assess its constitutionality or rights-compliance, thereby increasing constitutional engagement by the political branches of government.
    This can enhance constitutional culture in three ways: first, pre-legislative review ensures better protection for constitutional rights than stand-alone judicial review because it encourages the political branches to legislate in a more rights-conscious manner; second, it democratizes access to constitutional rights and values by placing them beyond the gamut of an unelected judiciary, and into the political realm; third, the review provides broader avenues for constitutional scrutiny of legislation, since it is usually supplemented by judicial review.
    Pre-legislative review assists in overcoming the limitations of judicial review. The expansion of the scope of review by an unelected judiciary raises concerns regarding constitutional and democratic legitimacy. For instance, judicial review, unlike the legislative process, results in restricted public participation in the policy-formulation process since the diverse viewpoints of relevant stakeholders remain unaccounted for before the judicial stage.
    Statement 3 is incorrect: Parliamentary Committees draw their authority from Article 105 and 118 of the Constitution of India. They are the formal ways for the political branches to engage with the constitutionality of legislation at the pre-legislative stage.

  3. Question 3 of 5
    3. Question

    3. Consider the following statements:
    Statement-I: A petitioner is entitled to relief under curative petition even if he is not a party to the litigation, but the judgement adversely affected his interests.
    Statement-II: A Curative Petition is the final remedy available where the Supreme Court can reconsider a dismissed review petition.
    Which one of the following is correct in respect of the above statements?

    Correct

    Answer: B
    Explanation:
    Both Statement-I and Statement-II are correct, and Statement-II is not the correct explanation for Statement-I.
    Statement-I is correct: A petitioner is entitled to relief under curative petition, if he establishes:
    ● violation of principles of natural justice where he was not a party to the list but the judgement adversely affected his interests.
    ● He was a party to the list, but he was not served with notice of the proceedings and the matter proceeded as if he has the choice.
    ● the parties giving scope for an apprehension of bias and the judgement adversely affects the petitioner.
    The curative petition should also contain a certification by a Senior Advocate with regard to the fulfilment of the above requirements.
    Statement-II is correct: Curative petition is the last constitutional remedy available to a person whose review petition has been dismissed by the Supreme Court.
    Though the Constitution explicitly speaks about the review power of the Supreme Court under Article 137, it is silent about ‘curative power’.
    The curative petition was given shape and form in the case of Rupa Ashok Hurra v. Ashok Hurra (2002), where the apex court reconsidered its judgment in exercise of its inherent power to prevent abuse of its process and to cure a gross miscarriage of justice.
    It was explained in the said decision that the curative power of the Court flows from Article 142 of the Constitution, which gives the Court power to do complete justice.

    Incorrect

    Answer: B
    Explanation:
    Both Statement-I and Statement-II are correct, and Statement-II is not the correct explanation for Statement-I.
    Statement-I is correct: A petitioner is entitled to relief under curative petition, if he establishes:
    ● violation of principles of natural justice where he was not a party to the list but the judgement adversely affected his interests.
    ● He was a party to the list, but he was not served with notice of the proceedings and the matter proceeded as if he has the choice.
    ● the parties giving scope for an apprehension of bias and the judgement adversely affects the petitioner.
    The curative petition should also contain a certification by a Senior Advocate with regard to the fulfilment of the above requirements.
    Statement-II is correct: Curative petition is the last constitutional remedy available to a person whose review petition has been dismissed by the Supreme Court.
    Though the Constitution explicitly speaks about the review power of the Supreme Court under Article 137, it is silent about ‘curative power’.
    The curative petition was given shape and form in the case of Rupa Ashok Hurra v. Ashok Hurra (2002), where the apex court reconsidered its judgment in exercise of its inherent power to prevent abuse of its process and to cure a gross miscarriage of justice.
    It was explained in the said decision that the curative power of the Court flows from Article 142 of the Constitution, which gives the Court power to do complete justice.

  4. Question 4 of 5
    4. Question

    4. Consider the following:
    1. State Election Commission
    2. State Finance Commission
    3. District Planning Committee
    4. Reservation of seats for women in gram panchayat
    How many of the above given were provided by the Seventy-third Constitutional Amendment Act, 1992?

    Correct

    Answer: C
    Explanation:
    73rd Constitution Amendment Act gave the constitutional status to the panchayati raj systems in India.
    Few of the Compulsory Provisions provided by the amendment are:
    ● Organization of Gram Sabha in villages
    ● Establishment of Panchayats at three levels, i.e. Village, Block and District level.
    ● Direct election to all the seats of the Panchayats.
    ● Reservation of seats for SC-ST(according to population) and women(1/3rd)
    ● Establishment of State Finance Commission after every five years to review the financial position of Panchayats.
    ● The tenure of the Panchayats has been fixed for five years.
    ● Establishment of a State Election Commission for conducting elections to the panchayats.
    Point 3 is incorrect: District Planning Committee was provided by the Seventy-fourth Constitutional Amendment Act, 1992.

    Incorrect

    Answer: C
    Explanation:
    73rd Constitution Amendment Act gave the constitutional status to the panchayati raj systems in India.
    Few of the Compulsory Provisions provided by the amendment are:
    ● Organization of Gram Sabha in villages
    ● Establishment of Panchayats at three levels, i.e. Village, Block and District level.
    ● Direct election to all the seats of the Panchayats.
    ● Reservation of seats for SC-ST(according to population) and women(1/3rd)
    ● Establishment of State Finance Commission after every five years to review the financial position of Panchayats.
    ● The tenure of the Panchayats has been fixed for five years.
    ● Establishment of a State Election Commission for conducting elections to the panchayats.
    Point 3 is incorrect: District Planning Committee was provided by the Seventy-fourth Constitutional Amendment Act, 1992.

  5. Question 5 of 5
    5. Question

    5. Consider the following:
    Statement-I: The Constitution makes it mandatory for the states to reform their respective prison laws according to Model Prisons Act, 2023.
    Statement-II: According to Schedule VII of the Constitution, prison is a subject under the State List.
    Which one of the following is correct in respect of the above statements?

    Correct

    Answer: D
    Explanation:
    Statement-I is incorrect: With the objective of holistically providing guidance and addressing the gaps in the existing Prisons Act, including the use of technology in prison management the centre has come up with the Model prisons Act 2023.
    It contains provisions for grant of parole, furlough, remission to prisoners to encourage good conduct, special provision for women/ transgender inmates, physical and mental well-being of prisoners and focus on the reformation and rehabilitation of inmates, etc.
    The comprehensive ‘Model Prisons Act, 2023’, by the Ministry of Home Affairs may serve as a guiding document for the States, and for adoption in their jurisdiction. The Constitution does not bind the states to reform their respective prison laws according to the Model prison Act, 2023.
    Statement-II is correct: ‘Prisons’/’persons detained therein’ is a “State-List” subject under Entry 4 of List II of the Seventh Schedule to the Constitution of India.
    Administration and management of prisons and prisoners is the responsibility of respective State Governments who are competent to take appropriate action in this regard.
    However, given the significance of prisons in the Criminal Justice System, the Ministry of Home Affairs has been providing regular guidance and support to the States and UTs on diverse issues relating to prison administration.
    Additional Information:
    Along with ‘The Prisons Act, 1894’, ‘The Prisoners Act, 1900’ and ‘The Transfer of Prisoners Act, 1950’ have also been reviewed by the Ministry of Home Affairs and relevant provisions of these Acts have been assimilated in the ‘Model Prisons Act, 2023.’
    State Governments and Union Territory Administrations can benefit from the Model Prisons Act, 2023 by adopting it in their jurisdictions, with such modifications which they may consider necessary, and repeal the existing three Acts in their jurisdictions.
    Some salient features of the new Model Prisons Act are:
    ● Provision for security assessment and segregation of prisoners, individual sentence planning,
    ● Grievance redressal, prison development board, attitudinal change towards prisoners.
    ● Provision of separate accommodation for women prisoners, transgender, etc.
    ● Provision for use of technology in prison administration with a view to bring transparency in prison administration.
    ● Provision for video conferencing with courts, scientific and technological interventions in prisons, etc.
    ● Provision of punishment for prisoners and jail staff for use of prohibited items like mobile phones etc. in jails.
    ● Provision regarding establishment and management of high security jail, open jail (open and semi open), etc.
    ● Provision for protecting the society from the criminal activities of hardened criminals and habitual offenders, etc.
    ● Provision for legal aid to prisoners, provision of parole, furlough and premature release etc. to incentivise good conduct.
    ● Focus on vocational training and skill development of prisoners and their reintegration into the society.

    Incorrect

    Answer: D
    Explanation:
    Statement-I is incorrect: With the objective of holistically providing guidance and addressing the gaps in the existing Prisons Act, including the use of technology in prison management the centre has come up with the Model prisons Act 2023.
    It contains provisions for grant of parole, furlough, remission to prisoners to encourage good conduct, special provision for women/ transgender inmates, physical and mental well-being of prisoners and focus on the reformation and rehabilitation of inmates, etc.
    The comprehensive ‘Model Prisons Act, 2023’, by the Ministry of Home Affairs may serve as a guiding document for the States, and for adoption in their jurisdiction. The Constitution does not bind the states to reform their respective prison laws according to the Model prison Act, 2023.
    Statement-II is correct: ‘Prisons’/’persons detained therein’ is a “State-List” subject under Entry 4 of List II of the Seventh Schedule to the Constitution of India.
    Administration and management of prisons and prisoners is the responsibility of respective State Governments who are competent to take appropriate action in this regard.
    However, given the significance of prisons in the Criminal Justice System, the Ministry of Home Affairs has been providing regular guidance and support to the States and UTs on diverse issues relating to prison administration.
    Additional Information:
    Along with ‘The Prisons Act, 1894’, ‘The Prisoners Act, 1900’ and ‘The Transfer of Prisoners Act, 1950’ have also been reviewed by the Ministry of Home Affairs and relevant provisions of these Acts have been assimilated in the ‘Model Prisons Act, 2023.’
    State Governments and Union Territory Administrations can benefit from the Model Prisons Act, 2023 by adopting it in their jurisdictions, with such modifications which they may consider necessary, and repeal the existing three Acts in their jurisdictions.
    Some salient features of the new Model Prisons Act are:
    ● Provision for security assessment and segregation of prisoners, individual sentence planning,
    ● Grievance redressal, prison development board, attitudinal change towards prisoners.
    ● Provision of separate accommodation for women prisoners, transgender, etc.
    ● Provision for use of technology in prison administration with a view to bring transparency in prison administration.
    ● Provision for video conferencing with courts, scientific and technological interventions in prisons, etc.
    ● Provision of punishment for prisoners and jail staff for use of prohibited items like mobile phones etc. in jails.
    ● Provision regarding establishment and management of high security jail, open jail (open and semi open), etc.
    ● Provision for protecting the society from the criminal activities of hardened criminals and habitual offenders, etc.
    ● Provision for legal aid to prisoners, provision of parole, furlough and premature release etc. to incentivise good conduct.
    ● Focus on vocational training and skill development of prisoners and their reintegration into the society.

window.wpAdvQuizInitList = window.wpAdvQuizInitList || []; window.wpAdvQuizInitList.push({ id: '#wpAdvQuiz_573', init: { quizId: 573, mode: 0, globalPoints: 10, timelimit: 0, resultsGrade: [0], bo: 0, qpp: 0, catPoints: [10], formPos: 0, lbn: "Finish quiz", json: {"2658":{"type":"single","id":2658,"catId":0,"points":2,"correct":[0,0,0,1]},"2659":{"type":"single","id":2659,"catId":0,"points":2,"correct":[0,1,0,0]},"2660":{"type":"single","id":2660,"catId":0,"points":2,"correct":[0,1,0,0]},"2661":{"type":"single","id":2661,"catId":0,"points":2,"correct":[0,0,1,0]},"2662":{"type":"single","id":2662,"catId":0,"points":2,"correct":[0,0,0,1]}} } });




Day-533 | Daily MCQs | UPSC Prelims | HISTORY OF MODERN INDIA

Day-533

Time limit: 0

Quiz-summary

0 of 5 questions completed

Questions:

  1. 1
  2. 2
  3. 3
  4. 4
  5. 5

Information

DAILY MCQ

You have already completed the quiz before. Hence you can not start it again.

Quiz is loading...

You must sign in or sign up to start the quiz.

You have to finish following quiz, to start this quiz:

Results

0 of 5 questions answered correctly

Your time:

Time has elapsed

You have reached 0 of 0 points, (0)

Categories

  1. Not categorized 0%
  1. 1
  2. 2
  3. 3
  4. 4
  5. 5
  1. Answered
  2. Review
  1. Question 1 of 5
    1. Question

    1. With reference to the history of India, the earliest resolution for the development of local self-government was given by-

    Correct

    Answer: D
    Explanation:
    ● Decentralisation and local governance have been a major concern in India for long. It goes back to Lord Mayo’s Resolution of 1870, which advocated decentralisation and made arrangements for strengthening the municipal institutions and involving more and more Indians in these bodies. Hence, option D is correct.
    ● Lord Ripon’s Resolution of 1882 too strongly advocated the cause of decentralisation of administration through the establishment of local self-governing institutions. The Resolution of 1882 was hailed as the Magna Carta of local government.
    ● The Decentralisation Commission Report, 1909 and the Government of India Acts 1919 and 1935 also favoured local governance. The Government of India Act 1919 introduced ‘Dyarchy’ in the provinces and local self-government became a transferred subject under the charge of a popular minister of the provincial legislature. The Act increased the taxation powers of the local bodies, lowered the franchise, reduced the nominated element and extended the communal electorate to a large number of municipalities.
    ● The Act of 1935 for the first time introduced a federal form of government and conferred ‘Provincial Autonomy’ on the provinces and envisaged self-government for the country as a whole.

    Incorrect

    Answer: D
    Explanation:
    ● Decentralisation and local governance have been a major concern in India for long. It goes back to Lord Mayo’s Resolution of 1870, which advocated decentralisation and made arrangements for strengthening the municipal institutions and involving more and more Indians in these bodies. Hence, option D is correct.
    ● Lord Ripon’s Resolution of 1882 too strongly advocated the cause of decentralisation of administration through the establishment of local self-governing institutions. The Resolution of 1882 was hailed as the Magna Carta of local government.
    ● The Decentralisation Commission Report, 1909 and the Government of India Acts 1919 and 1935 also favoured local governance. The Government of India Act 1919 introduced ‘Dyarchy’ in the provinces and local self-government became a transferred subject under the charge of a popular minister of the provincial legislature. The Act increased the taxation powers of the local bodies, lowered the franchise, reduced the nominated element and extended the communal electorate to a large number of municipalities.
    ● The Act of 1935 for the first time introduced a federal form of government and conferred ‘Provincial Autonomy’ on the provinces and envisaged self-government for the country as a whole.

  2. Question 2 of 5
    2. Question

    2. Consider the following statements:
    Statement I: The nineteenth century reform movements failed to improve remarkably the social conditions of women.
    Statement II: The reform movements failed to initiate any movement for female education.
    Which one of the following is correct in respect of the above statements?

    Correct

    Answer: C
    Explanation:
    The answer to the puzzle, that why reform movements of the nineteenth century failed to bring remarkable changes in the condition of the women lies in the motives behind these reforms.
    ● It was not that reformers have failed to take initiative for women education. The initiative was taken in Calcutta by men like Radhakanta Deb and the School Book Society and later by Keshub Chandra Sen and the Brahmo Samaj, in western India by Mahadev Govind Ranade and Prarthana Samaj, in north India by Swami Dayanand and his Arya Samaj and in Madras by Annie Besant and the Theosophical Society.
    ● The colonial government from the administration of Lord Dalhousie (1848–56) also took particular interest in female education. J.E. Drinkwater Bethune, the law member in the governor general’s council opened in 1849 what eventually became the most well-known girls’ school in Calcutta. The Hunter Commission recommended liberal grants-in-aid and special scholarships for women’s education. During the next two decades significant improvements were seen in women’s enrolment in both universities and secondary schools.
    ● The aim of the reform movement was never the emancipation of women. The colonial government wanted female education as it wanted the Indian civil servants to be married to educated wives, so that they did not have to face the psychological trauma of a split household. Also, English educated mothers were expected to breed loyal subjects.
    ● The educated Indian middle-class males, on the other hand, dreamt of the Victorian ideal of companionate marriage. What they wanted was an educated bhadramahila (gentlewoman) who would be an ideal companion to the enlightened Hindu bhadralok.
    Education thus far from being emancipatory, further confined women to idealised domestic roles as good wives and better mothers.

    Incorrect

    Answer: C
    Explanation:
    The answer to the puzzle, that why reform movements of the nineteenth century failed to bring remarkable changes in the condition of the women lies in the motives behind these reforms.
    ● It was not that reformers have failed to take initiative for women education. The initiative was taken in Calcutta by men like Radhakanta Deb and the School Book Society and later by Keshub Chandra Sen and the Brahmo Samaj, in western India by Mahadev Govind Ranade and Prarthana Samaj, in north India by Swami Dayanand and his Arya Samaj and in Madras by Annie Besant and the Theosophical Society.
    ● The colonial government from the administration of Lord Dalhousie (1848–56) also took particular interest in female education. J.E. Drinkwater Bethune, the law member in the governor general’s council opened in 1849 what eventually became the most well-known girls’ school in Calcutta. The Hunter Commission recommended liberal grants-in-aid and special scholarships for women’s education. During the next two decades significant improvements were seen in women’s enrolment in both universities and secondary schools.
    ● The aim of the reform movement was never the emancipation of women. The colonial government wanted female education as it wanted the Indian civil servants to be married to educated wives, so that they did not have to face the psychological trauma of a split household. Also, English educated mothers were expected to breed loyal subjects.
    ● The educated Indian middle-class males, on the other hand, dreamt of the Victorian ideal of companionate marriage. What they wanted was an educated bhadramahila (gentlewoman) who would be an ideal companion to the enlightened Hindu bhadralok.
    Education thus far from being emancipatory, further confined women to idealised domestic roles as good wives and better mothers.

  3. Question 3 of 5
    3. Question

    3. The Justice Party formed in 1916, which demanded separate communal representation for the non-Brahmans was related to-

    Correct

    Answer: B
    Explanation:
    The non-Brahman movement in the Madras Presidency was associated with the Vellalas and a Dravidian identity. On the political front, the movement witnessed the formation of the Justice Party in 1916.
    It opposed the Congress as a Brahman-dominated organisation, and claimed separate communal representation for the non-Brahmans as had been granted to the Muslims in the Morley–Minto reform. This demand, supported by the colonial bureaucracy, was granted in the Montagu- Chelmsford reform of 1919, as it allocated twenty-eight reserved seats to the non-Brahmans in the Madras Legislative Council.

    Incorrect

    Answer: B
    Explanation:
    The non-Brahman movement in the Madras Presidency was associated with the Vellalas and a Dravidian identity. On the political front, the movement witnessed the formation of the Justice Party in 1916.
    It opposed the Congress as a Brahman-dominated organisation, and claimed separate communal representation for the non-Brahmans as had been granted to the Muslims in the Morley–Minto reform. This demand, supported by the colonial bureaucracy, was granted in the Montagu- Chelmsford reform of 1919, as it allocated twenty-eight reserved seats to the non-Brahmans in the Madras Legislative Council.

  4. Question 4 of 5
    4. Question

    4. Consider the following pairs:
    Parties/Organisation – Leaders
    1. Unionist Party – B.S. Moonje
    2. Krishak Praja Party – A.K. Fazlul Huq
    3. Hindu Mahasabha – Chhotu Ram
    How many pairs given above are correctly matched?

    Correct

    Answer: A
    Explanation:
    The correct matches are given below:
    ● Hindu Mahasabha- B.S. Moonje
    ● Krishak Praja Party- A.K. Fazlul Huq
    ● Unionist Party- Chhotu Ram
    Additional Information
    ● Hindu Sabha’s sprang up in Lahore in 1882 and by 1906 a Hindu Sabha was established in almost in each district of the Punjab. In 1915, the all India Hindu Mahasabha held its first session in (Haridwar) UP. Some of its important leaders included- Lala Lajpat Rai, Madan Mohan Malviya, S.P. Mukherjee, V.D. Savarakr, etc.
    ● In 1923 the Unionist Party was formed in British Punjab by Chhotu Ram of Rohtak district with the help of Sir Fazl-i-Husain. The party’s main agenda was to protect the agrarian society’s interests and curtail the growing communal sentiments in the region.
    ● The Krishak Praja Party was formed in July 1936 at Dacca. Sir Abdur Rahim became the President of the KPP; Fazlul Huq, Mujibur Rahman, Abdul Karim, Abdullah Suhrawardy and Abdul Momen were its Vice-Presidents; Mohammad Akram Khan became its Secretary and the Party’s Joint Secretaries were Shamsuddin Ahmed and Tamizuddin Khan. The KPP was almost entirely an East Bengal Party and its leadership was mainly in the hands of the educated Muslim middle class who came from different walks of life.

    Incorrect

    Answer: A
    Explanation:
    The correct matches are given below:
    ● Hindu Mahasabha- B.S. Moonje
    ● Krishak Praja Party- A.K. Fazlul Huq
    ● Unionist Party- Chhotu Ram
    Additional Information
    ● Hindu Sabha’s sprang up in Lahore in 1882 and by 1906 a Hindu Sabha was established in almost in each district of the Punjab. In 1915, the all India Hindu Mahasabha held its first session in (Haridwar) UP. Some of its important leaders included- Lala Lajpat Rai, Madan Mohan Malviya, S.P. Mukherjee, V.D. Savarakr, etc.
    ● In 1923 the Unionist Party was formed in British Punjab by Chhotu Ram of Rohtak district with the help of Sir Fazl-i-Husain. The party’s main agenda was to protect the agrarian society’s interests and curtail the growing communal sentiments in the region.
    ● The Krishak Praja Party was formed in July 1936 at Dacca. Sir Abdur Rahim became the President of the KPP; Fazlul Huq, Mujibur Rahman, Abdul Karim, Abdullah Suhrawardy and Abdul Momen were its Vice-Presidents; Mohammad Akram Khan became its Secretary and the Party’s Joint Secretaries were Shamsuddin Ahmed and Tamizuddin Khan. The KPP was almost entirely an East Bengal Party and its leadership was mainly in the hands of the educated Muslim middle class who came from different walks of life.

  5. Question 5 of 5
    5. Question

    5. Consider the following:
    1. Export promotion in consumer goods
    2. Growth in internal trade
    3. Inflow of foreign capital
    How many of the above mentioned factors were responsible for the modest development of industries in India during the World War I period?

    Correct

    Answer: A
    Explanation:
    Till the end of World War One for various reasons the number of registered industrial enterprises had been steadily rising, while developments in the interwar period further strengthened their position.
    The factors which facilitated a modest Indian industrial development, despite an obstructing colonial presence, were many, such as a
    ● growing tendency towards import substitution in consumer goods,
    ● shifting of attention towards the domestic markets,
    ● growth in internal trade,
    ● shifting of traditionally accumulated capital through trade, moneylending and landowning to industrial investments,
    ● the outflow of foreign capital creating a space for indigenous entrepreneurs.
    Candidates should also keep in mind the difference between ‘import substitution’ and ‘export promotion’.
    Import substitution generally means the satisfaction of a greater proportion of a country’s total demand for goods (production plus imports) through its own domestic production, while export promotion is a strategy to increase the export trade in a country.

    Incorrect

    Answer: A
    Explanation:
    Till the end of World War One for various reasons the number of registered industrial enterprises had been steadily rising, while developments in the interwar period further strengthened their position.
    The factors which facilitated a modest Indian industrial development, despite an obstructing colonial presence, were many, such as a
    ● growing tendency towards import substitution in consumer goods,
    ● shifting of attention towards the domestic markets,
    ● growth in internal trade,
    ● shifting of traditionally accumulated capital through trade, moneylending and landowning to industrial investments,
    ● the outflow of foreign capital creating a space for indigenous entrepreneurs.
    Candidates should also keep in mind the difference between ‘import substitution’ and ‘export promotion’.
    Import substitution generally means the satisfaction of a greater proportion of a country’s total demand for goods (production plus imports) through its own domestic production, while export promotion is a strategy to increase the export trade in a country.

window.wpAdvQuizInitList = window.wpAdvQuizInitList || []; window.wpAdvQuizInitList.push({ id: '#wpAdvQuiz_572', init: { quizId: 572, mode: 0, globalPoints: 10, timelimit: 0, resultsGrade: [0], bo: 0, qpp: 0, catPoints: [10], formPos: 0, lbn: "Finish quiz", json: {"2653":{"type":"single","id":2653,"catId":0,"points":2,"correct":[0,0,0,1]},"2654":{"type":"single","id":2654,"catId":0,"points":2,"correct":[0,0,1,0]},"2655":{"type":"single","id":2655,"catId":0,"points":2,"correct":[0,1,0,0]},"2656":{"type":"single","id":2656,"catId":0,"points":2,"correct":[1,0,0,0]},"2657":{"type":"single","id":2657,"catId":0,"points":2,"correct":[1,0,0,0]}} } });




Day-532 | Daily MCQs | UPSC Prelims | CURRENT DEVELOPMENTS

Day-532

Time limit: 0

Quiz-summary

0 of 5 questions completed

Questions:

  1. 1
  2. 2
  3. 3
  4. 4
  5. 5

Information

DAILY MCQ

You have already completed the quiz before. Hence you can not start it again.

Quiz is loading...

You must sign in or sign up to start the quiz.

You have to finish following quiz, to start this quiz:

Results

0 of 5 questions answered correctly

Your time:

Time has elapsed

You have reached 0 of 0 points, (0)

Categories

  1. Not categorized 0%
  1. 1
  2. 2
  3. 3
  4. 4
  5. 5
  1. Answered
  2. Review
  1. Question 1 of 5
    1. Question

    1. Consider the following pairs:
    Geographical Indication Tag – States
    1. Yak Churpi – Assam
    2. Khaw Tai – Arunachal Pradesh
    3. Tangsa Textile – Tripura
    How many of the pairs given above are correctly matched?

    Correct

    Answer: A
    Explanation:
    Context: Arunachal Pradesh has received the Geographical Indication (GI) tag for Arunachal Yak Churpi, Khaw Tai (Khamti rice), and Tangsa textile.
    Given below is the correctly matched pairs:
    Geographical Indication Tag – States
    1. Yak Churpi – Arunachal Pradesh
    2. Khaw Tai – Arunachal Pradesh
    3. Tangsa Textile – Arunachal Pradesh
    Additional information:
    • Arunachal Yak Churpi: Arunachal Yak Churpi is derived from the milk of the Arunachali yak, a rare breed found primarily in the West Kameng and Tawang districts of Arunachal Pradesh. This milk is obtained from yaks reared by Brokpas, a tribal community known for their expertise in yak husbandry. These pastoralists practice seasonal migration, taking their yaks to higher altitudes during summers and descending to mid-altitude mountainous regions in winters, as yaks cannot survive at lower altitudes during summer.
    • Khaw Tai or Khamti Rice: It is a chewy sticky rice variety, hails from the Namsai region in Arunachal Pradesh and is cultivated by traditional Khampti tribal farmers.
    • Tangsa Textile: The Tangsa Textile products crafted by the Tangsa tribe of Changlang district are renowned for their exotic designs and vibrant colors. This traditional craftsmanship reflects the cultural richness of the region.

    Incorrect

    Answer: A
    Explanation:
    Context: Arunachal Pradesh has received the Geographical Indication (GI) tag for Arunachal Yak Churpi, Khaw Tai (Khamti rice), and Tangsa textile.
    Given below is the correctly matched pairs:
    Geographical Indication Tag – States
    1. Yak Churpi – Arunachal Pradesh
    2. Khaw Tai – Arunachal Pradesh
    3. Tangsa Textile – Arunachal Pradesh
    Additional information:
    • Arunachal Yak Churpi: Arunachal Yak Churpi is derived from the milk of the Arunachali yak, a rare breed found primarily in the West Kameng and Tawang districts of Arunachal Pradesh. This milk is obtained from yaks reared by Brokpas, a tribal community known for their expertise in yak husbandry. These pastoralists practice seasonal migration, taking their yaks to higher altitudes during summers and descending to mid-altitude mountainous regions in winters, as yaks cannot survive at lower altitudes during summer.
    • Khaw Tai or Khamti Rice: It is a chewy sticky rice variety, hails from the Namsai region in Arunachal Pradesh and is cultivated by traditional Khampti tribal farmers.
    • Tangsa Textile: The Tangsa Textile products crafted by the Tangsa tribe of Changlang district are renowned for their exotic designs and vibrant colors. This traditional craftsmanship reflects the cultural richness of the region.

  2. Question 2 of 5
    2. Question

    2. Consider the following statements:
    1. Dhole is the only endangered wild pack-living animal of dog family found in the tropical Indian forests.
    2. Dhole shares an antagonistic interaction with the tiger.
    Which of the statements given above is/are correct?

    Correct

    Answer: A
    Explanation:
    Context: Recently, a study conducted in Assam’s Manas National Park revealed a surprising positive relationship between dholes (Asiatic wild dogs) and tigers.
    • Statement 1 is correct: Dhole or Asiatic wild dog is the only endangered wild pack-living canid or animal of dog family in the tropical Indian forests.
    • Statement 2 is incorrect: Dhole shares a coexistence or even cooperative behaviour with tiger. The positive association between dholes and tigers may be attributed to overlapping prey availability or habitat suitability, suggesting a more complex ecological dynamic at play and prompting the need for further research.
    Additional information:
    • Dhole is a wild carnivorous animal and is a member of the family Canidae and the class Mammalia.
    • Dholes, historically widespread across southern Russia to southeast Asia, are now mainly found in south and southeast Asia, with northern populations in China.
    • In India, they are clustered in the Western and Eastern Ghats, central India, and northeast India, with Karnataka, Maharashtra, and Madhya Pradesh playing a crucial role in their conservation, according to a 2020 study.
    • In 2014, the Indian government sanctioned its first dhole conservation breeding center at the Indira Gandhi Zoological Park (IGZP) in Visakhapatnam.
    Protection status:
    • Wildlife Protection Act, 1972: Schedule 2.
    • International Union for Conservation of Nature’s Red List: Endangered.
    • The Convention on International Trade in Endangered Species of Wild Fauna and Flora (CITES): Appendix II.

    Incorrect

    Answer: A
    Explanation:
    Context: Recently, a study conducted in Assam’s Manas National Park revealed a surprising positive relationship between dholes (Asiatic wild dogs) and tigers.
    • Statement 1 is correct: Dhole or Asiatic wild dog is the only endangered wild pack-living canid or animal of dog family in the tropical Indian forests.
    • Statement 2 is incorrect: Dhole shares a coexistence or even cooperative behaviour with tiger. The positive association between dholes and tigers may be attributed to overlapping prey availability or habitat suitability, suggesting a more complex ecological dynamic at play and prompting the need for further research.
    Additional information:
    • Dhole is a wild carnivorous animal and is a member of the family Canidae and the class Mammalia.
    • Dholes, historically widespread across southern Russia to southeast Asia, are now mainly found in south and southeast Asia, with northern populations in China.
    • In India, they are clustered in the Western and Eastern Ghats, central India, and northeast India, with Karnataka, Maharashtra, and Madhya Pradesh playing a crucial role in their conservation, according to a 2020 study.
    • In 2014, the Indian government sanctioned its first dhole conservation breeding center at the Indira Gandhi Zoological Park (IGZP) in Visakhapatnam.
    Protection status:
    • Wildlife Protection Act, 1972: Schedule 2.
    • International Union for Conservation of Nature’s Red List: Endangered.
    • The Convention on International Trade in Endangered Species of Wild Fauna and Flora (CITES): Appendix II.

  3. Question 3 of 5
    3. Question

    3. Consider the following statements:
    1. The Representation of People Act, 1950 provides for the linking of Aadhaar number of a person with the electoral roll.
    2. Linking of Aadhaar number with the electoral roll is mandatory.
    Which of the statements given above is/are correct?

    Correct

    Answer: A
    Explanation:
    Context: Petitioners expressed concerns about voter privacy and alleged that the Centre and Election Commission were compelling voters to submit their Aadhaar numbers without providing an alternative option.
    • Statement 1 is correct: Linking of Aadhar number with the electoral roll comes under the provisions of Section 23 of the Representation of People Act, 1950.
    • Statement 2 is incorrect: Linking of Aadhaar numbers with the Electoral roll is not mandatory according to Rule 26-B of the Registration of Electors (Amendment) Rules 2022.
    Additional information:
    • Rule 26B, of RPA, 1950 deals with “special provision for providing Aadhaar number by existing electors”.
    • It states that “every person whose name is listed in the roll may intimate his Aadhaar number to the registration officer in Form 6B in accordance with sub-section (5) of Section 23 of the Representation of People Act, 1950.
    • Form 6B is a letter of information that contains a person’s Aadhaar number for the purpose of electoral roll authentication.

    Incorrect

    Answer: A
    Explanation:
    Context: Petitioners expressed concerns about voter privacy and alleged that the Centre and Election Commission were compelling voters to submit their Aadhaar numbers without providing an alternative option.
    • Statement 1 is correct: Linking of Aadhar number with the electoral roll comes under the provisions of Section 23 of the Representation of People Act, 1950.
    • Statement 2 is incorrect: Linking of Aadhaar numbers with the Electoral roll is not mandatory according to Rule 26-B of the Registration of Electors (Amendment) Rules 2022.
    Additional information:
    • Rule 26B, of RPA, 1950 deals with “special provision for providing Aadhaar number by existing electors”.
    • It states that “every person whose name is listed in the roll may intimate his Aadhaar number to the registration officer in Form 6B in accordance with sub-section (5) of Section 23 of the Representation of People Act, 1950.
    • Form 6B is a letter of information that contains a person’s Aadhaar number for the purpose of electoral roll authentication.

  4. Question 4 of 5
    4. Question

    4. Which of the following is correct regarding the objectives of MAHIR initiative?

    Correct

    Answer: C
    Explanation:
    Context: The Ministry of Power and the Ministry of New and Renewable Energy have jointly launched a National Mission named “Mission on Advanced and High-Impact Research (MAHIR)”.
    The MAHIR initiative aims to identify emerging technologies and areas of future relevance for the global power sector and develop them indigenously.
    The Mission is planned for an initial period of five years from 2023-24 to 2027-28 and will follow the technology life cycle approach of Idea to Product.
    Objectives of the Mission:
    • To identify emerging technologies and areas of future relevance for the global power sector and develop them indigenously.
    • To provide a platform for collective brainstorming and synergetic technology development among power sector stakeholders.
    • To support pilot projects of indigenous technologies developed by Indian start-ups and facilitate their commercialization.
    • To leverage foreign alliances and partnerships for research and development of advanced technologies and technology transfer.
    • To promote scientific and industrial R&D in the power sector and create an innovative ecosystem.
    • To position India among the leading countries in power system-related technologies and applications development.
    Funding:
    • It will be funded by pooling resources from the Ministry of Power, Ministry of New & Renewable Energy, and Central Public Sector Enterprises under these ministries.
    • Additional funding, if required, will be mobilized from the Government of India’s budgetary resources.
    Areas Identified for Research under MAHIR:
    • Alternatives to Lithium-Ion storage batteries
    • Modifying electric cookers/pans to suit Indian cooking methods
    • Green hydrogen for mobility (High Efficiency Fuel Cell)
    • Carbon capture
    • Geo-thermal energy
    • Solid state refrigeration
    • Nano technology for EV battery
    • Indigenous CRGO technology

    Incorrect

    Answer: C
    Explanation:
    Context: The Ministry of Power and the Ministry of New and Renewable Energy have jointly launched a National Mission named “Mission on Advanced and High-Impact Research (MAHIR)”.
    The MAHIR initiative aims to identify emerging technologies and areas of future relevance for the global power sector and develop them indigenously.
    The Mission is planned for an initial period of five years from 2023-24 to 2027-28 and will follow the technology life cycle approach of Idea to Product.
    Objectives of the Mission:
    • To identify emerging technologies and areas of future relevance for the global power sector and develop them indigenously.
    • To provide a platform for collective brainstorming and synergetic technology development among power sector stakeholders.
    • To support pilot projects of indigenous technologies developed by Indian start-ups and facilitate their commercialization.
    • To leverage foreign alliances and partnerships for research and development of advanced technologies and technology transfer.
    • To promote scientific and industrial R&D in the power sector and create an innovative ecosystem.
    • To position India among the leading countries in power system-related technologies and applications development.
    Funding:
    • It will be funded by pooling resources from the Ministry of Power, Ministry of New & Renewable Energy, and Central Public Sector Enterprises under these ministries.
    • Additional funding, if required, will be mobilized from the Government of India’s budgetary resources.
    Areas Identified for Research under MAHIR:
    • Alternatives to Lithium-Ion storage batteries
    • Modifying electric cookers/pans to suit Indian cooking methods
    • Green hydrogen for mobility (High Efficiency Fuel Cell)
    • Carbon capture
    • Geo-thermal energy
    • Solid state refrigeration
    • Nano technology for EV battery
    • Indigenous CRGO technology

  5. Question 5 of 5
    5. Question

    5. In the context of Fermi energy, consider the following statements:
    1. Fermi energy is the energy of the highest occupied state of electrons in a material at absolute zero temperature.
    2. Fermi energy determines electrical and thermal conductivity of materials.
    3. Metals such as copper, aluminum, and silver exhibit high Fermi energies.
    How many of the statements given above are correct?

    Correct

    Answer: C
    Explanation:
    Context: Recently, Fermi energy has gained significant attention due to its wide range of daily practical applications in various fields, driven by the principles of quantum physics.
    ▪ Statement 1 is correct: Fermi energy is the energy of the highest occupied state of electrons in a material at absolute zero temperature (-273º C or 0K).
    ▪ Statement 2 is correct: Fermi energy determines various physical properties of materials, including electrical and thermal conductivity, heat capacity, magnetism, and superconductivity.
    ▪ Statement 3 is correct: Metals, such as copper, aluminum, and silver, exhibit high Fermi energies, even at extremely low temperatures.
    • Fermi energy determines electron velocity in conduction, as only electrons with energies close to the Fermi energy can participate in the conduction process.
    • The Fermi energy and fermionic behavior of electrons, governed by quantum mechanics, are responsible for various properties of metals, including their reflectivity, electrical conductivity, and thermal conductivity.
    • Understanding Fermi energy is essential for comprehending the fundamental behaviors and applications of metals in our daily lives.
    Additional information:
    • Quantum physics is the branch of physics that deals with the behavior and properties of the smallest particles of matter and energy, such as atoms, electrons, photons, and quarks.
    • Quantum physics reveals that these particles can behave in strange and surprising ways, such as being in two places at once (superimposition), tunneling through barriers, or entanglement with each other over long distances.
    • Quantum physics also explains how atoms and molecules form the basis of all matter and how light and other electromagnetic waves are produced and interact with matter.
    • Quantum physics impacts daily life through electronics, computing, lasers, and optics. It enables technologies like MRI for medical imaging and improves renewable energy systems.
    • Quantum cryptography ensures secure communication, while materials science and nanotechnology benefit from quantum principles.

    Incorrect

    Answer: C
    Explanation:
    Context: Recently, Fermi energy has gained significant attention due to its wide range of daily practical applications in various fields, driven by the principles of quantum physics.
    ▪ Statement 1 is correct: Fermi energy is the energy of the highest occupied state of electrons in a material at absolute zero temperature (-273º C or 0K).
    ▪ Statement 2 is correct: Fermi energy determines various physical properties of materials, including electrical and thermal conductivity, heat capacity, magnetism, and superconductivity.
    ▪ Statement 3 is correct: Metals, such as copper, aluminum, and silver, exhibit high Fermi energies, even at extremely low temperatures.
    • Fermi energy determines electron velocity in conduction, as only electrons with energies close to the Fermi energy can participate in the conduction process.
    • The Fermi energy and fermionic behavior of electrons, governed by quantum mechanics, are responsible for various properties of metals, including their reflectivity, electrical conductivity, and thermal conductivity.
    • Understanding Fermi energy is essential for comprehending the fundamental behaviors and applications of metals in our daily lives.
    Additional information:
    • Quantum physics is the branch of physics that deals with the behavior and properties of the smallest particles of matter and energy, such as atoms, electrons, photons, and quarks.
    • Quantum physics reveals that these particles can behave in strange and surprising ways, such as being in two places at once (superimposition), tunneling through barriers, or entanglement with each other over long distances.
    • Quantum physics also explains how atoms and molecules form the basis of all matter and how light and other electromagnetic waves are produced and interact with matter.
    • Quantum physics impacts daily life through electronics, computing, lasers, and optics. It enables technologies like MRI for medical imaging and improves renewable energy systems.
    • Quantum cryptography ensures secure communication, while materials science and nanotechnology benefit from quantum principles.

window.wpAdvQuizInitList = window.wpAdvQuizInitList || []; window.wpAdvQuizInitList.push({ id: '#wpAdvQuiz_571', init: { quizId: 571, mode: 0, globalPoints: 10, timelimit: 0, resultsGrade: [0], bo: 0, qpp: 0, catPoints: [10], formPos: 0, lbn: "Finish quiz", json: {"2648":{"type":"single","id":2648,"catId":0,"points":2,"correct":[1,0,0,0]},"2649":{"type":"single","id":2649,"catId":0,"points":2,"correct":[1,0,0,0]},"2650":{"type":"single","id":2650,"catId":0,"points":2,"correct":[1,0,0,0]},"2651":{"type":"single","id":2651,"catId":0,"points":2,"correct":[0,0,1,0]},"2652":{"type":"single","id":2652,"catId":0,"points":2,"correct":[0,0,1,0]}} } });




Day-531 | Daily MCQs | UPSC Prelims | ENVIRONMENT AND ECOLOGY

Day-531

Time limit: 0

Quiz-summary

0 of 5 questions completed

Questions:

  1. 1
  2. 2
  3. 3
  4. 4
  5. 5

Information

DAILY MCQ

You have already completed the quiz before. Hence you can not start it again.

Quiz is loading...

You must sign in or sign up to start the quiz.

You have to finish following quiz, to start this quiz:

Results

0 of 5 questions answered correctly

Your time:

Time has elapsed

You have reached 0 of 0 points, (0)

Categories

  1. Not categorized 0%
  1. 1
  2. 2
  3. 3
  4. 4
  5. 5
  1. Answered
  2. Review
  1. Question 1 of 5
    1. Question

    1. Consider the following statements:
    1. Flex-fuel vehicles can operate on maximum 85% ethanol.
    2. Bioethanol is mainly derived from used cooking oil and animal fats.
    3. Flex fuels have higher fuel efficiency than petrol.
    How many of the above statements are correct?

    Correct

    Answer: D
    Explanation:
    Statement 1 is incorrect: Flex fuel vehicles (FFV) are capable of running on 100 per cent petrol or 100 per cent bio-ethanol or a combination of both. A flex-fuel or flexible fuel vehicle typically has an internal combustion engine (ICE), but unlike a regular petrol vehicle, it can run on more than one type of fuel, or a mixture of these fuels. The most common versions use a blend of petrol and ethanol or methanol.
    Flex-fuel vehicles such as the prototype Hycross (by Toyota) can run on blends of ethanol that are far higher than the current standard 20% mix (E20). This is made possible by equipping the engine with a fuel mix sensor and an engine control module (ECM) programming those senses and automatically adjusts for any ratio of designated fuels.

    Statement 2 is incorrect: While Biodiesel is produced from used cooking oils or animal fats, the feedstock for first-generation bioethanol is mainly edible food crops such as rice, wheat, barley, potato, corn, sugarcane, and vegetable oil like soybean oil, sunflower oil, olive oil, canola oil, mustard oil, etc.
    However, second-generation bioethanol production process exploits non-food crops, food crops residues, wastes of wood-based or food-based industries such as wood chips, skins or pulps from fruit pressing, etc.
    Typically, bioethanol is produced from the fermentation of various feedstocks that contain mainly fermentable sugars or carbohydrates. Fermentation is the process of decomposing an organic substrate into products (e.g. bioethanol) by bacteria, yeast, fungi and other microorganisms usually present in gut.
    According to a NITI Aayog report, in 2019-20, more than 90% of the ethanol produced in India came from sugarcane.
    Statement 3 is incorrect: The fuel efficiency of flex fuels is lower than that of petrol. Flex-fuel cars typically take a 4-8% hit on fuel efficiency when using ethanol for motive power. Bio-ethanol contains less energy per litre than petrol. However, the calorific value (energy contained in the fuel) of bio-ethanol will become on par with petrol with use of advanced technology.

    Incorrect

    Answer: D
    Explanation:
    Statement 1 is incorrect: Flex fuel vehicles (FFV) are capable of running on 100 per cent petrol or 100 per cent bio-ethanol or a combination of both. A flex-fuel or flexible fuel vehicle typically has an internal combustion engine (ICE), but unlike a regular petrol vehicle, it can run on more than one type of fuel, or a mixture of these fuels. The most common versions use a blend of petrol and ethanol or methanol.
    Flex-fuel vehicles such as the prototype Hycross (by Toyota) can run on blends of ethanol that are far higher than the current standard 20% mix (E20). This is made possible by equipping the engine with a fuel mix sensor and an engine control module (ECM) programming those senses and automatically adjusts for any ratio of designated fuels.

    Statement 2 is incorrect: While Biodiesel is produced from used cooking oils or animal fats, the feedstock for first-generation bioethanol is mainly edible food crops such as rice, wheat, barley, potato, corn, sugarcane, and vegetable oil like soybean oil, sunflower oil, olive oil, canola oil, mustard oil, etc.
    However, second-generation bioethanol production process exploits non-food crops, food crops residues, wastes of wood-based or food-based industries such as wood chips, skins or pulps from fruit pressing, etc.
    Typically, bioethanol is produced from the fermentation of various feedstocks that contain mainly fermentable sugars or carbohydrates. Fermentation is the process of decomposing an organic substrate into products (e.g. bioethanol) by bacteria, yeast, fungi and other microorganisms usually present in gut.
    According to a NITI Aayog report, in 2019-20, more than 90% of the ethanol produced in India came from sugarcane.
    Statement 3 is incorrect: The fuel efficiency of flex fuels is lower than that of petrol. Flex-fuel cars typically take a 4-8% hit on fuel efficiency when using ethanol for motive power. Bio-ethanol contains less energy per litre than petrol. However, the calorific value (energy contained in the fuel) of bio-ethanol will become on par with petrol with use of advanced technology.

  2. Question 2 of 5
    2. Question

    2. Consider the following statements:
    Statement I: Belem declaration has been concluded successfully by the International Tropical Timber Organisation and Food and Agriculture Organisation (FAO).
    Statement II: The Belem declaration has special emphasis on curbing deforestation in Amazon rainforest and recognises indigenous knowledge as a condition for biodiversity conservation.
    Which one of the following is correct in respect of the above statements?

    Correct

    Answer: D
    Explanation:
    Statement 1 is incorrect: Leaders from the eight countries across the Amazon, including Bolivia, Brazil, Colombia, Ecuador, Guyana, Peru, Suriname and Venezuela, failed to agree on the goal to protect the rainforest at the recently concluded Amazon Summit organised by the Amazon Cooperation Treaty Organization (ACTO) on 8-9 August,2023. This agreement is referred to as Belem declaration.
    The declaration could not arrive at full consensus of all Amazon basin countries. Colombia had proposed that 80 per cent of the Amazon should be protected from deforestation and degradation by 2025 but did not find support from all the members.
    The failure of consensus on protected areas could have implications on the overall goals and targets set under the Convention on Biological Diversity’s Global Biodiversity Framework set in December 2022. Under this, member countries had agreed to protect at least 30 per cent of land and sea by 2030.
    Statement 2 is correct: The Belem Declaration released during the Amazon Summit recognises Indigenous knowledge as a condition for biodiversity conservation and calls for ensuring full and effective participation of Indigenous Peoples in decision-making and public policy formulation processes. The declaration contends that at least 80 per cent of forest needs to be protected to avoid an irreversible tipping point; thus, the need for cumulative action by all Amazon basin countries is required to curb the deforestation in the Amazon rainforests.
    The declaration promotes sustainable use of biodiversity resources in the Amazon. At the moment, there is no indication of how much money members will invest to fund the declaration’s proposed objectives or support minimum conservation standards.
    Additional information:
    ● The Amazon Cooperation Treaty Organization (ACTO) is an intergovernmental organization formed by the eight Amazonian countries: Bolivia, Brazil, Colombia, Ecuador, Guyana, Peru, Suriname, and Venezuela, which signed the Amazon Cooperation Treaty (ACT), becoming the only socio-environmental block in Latin America.
    ● The Amazon Cooperation Treaty (ACT) signed on July 3, 1978 gave rise to the ACTO, that is oriented to promote the harmonious development of the Amazonian territories in such a way that the joint actions of the Amazonian countries produce equitable and mutually beneficial results in achieving the sustainable development of the Amazon Region.
    ● In 1995, the eight nations decided to create the Amazon Cooperation Treaty Organization (ACTO), to strengthen and implement the objectives of the Amazon Cooperation Treaty.
    ● Permanent Secretariat of ACTO was established in Brasilia (Brazil) on December 13, 2002.

    Incorrect

    Answer: D
    Explanation:
    Statement 1 is incorrect: Leaders from the eight countries across the Amazon, including Bolivia, Brazil, Colombia, Ecuador, Guyana, Peru, Suriname and Venezuela, failed to agree on the goal to protect the rainforest at the recently concluded Amazon Summit organised by the Amazon Cooperation Treaty Organization (ACTO) on 8-9 August,2023. This agreement is referred to as Belem declaration.
    The declaration could not arrive at full consensus of all Amazon basin countries. Colombia had proposed that 80 per cent of the Amazon should be protected from deforestation and degradation by 2025 but did not find support from all the members.
    The failure of consensus on protected areas could have implications on the overall goals and targets set under the Convention on Biological Diversity’s Global Biodiversity Framework set in December 2022. Under this, member countries had agreed to protect at least 30 per cent of land and sea by 2030.
    Statement 2 is correct: The Belem Declaration released during the Amazon Summit recognises Indigenous knowledge as a condition for biodiversity conservation and calls for ensuring full and effective participation of Indigenous Peoples in decision-making and public policy formulation processes. The declaration contends that at least 80 per cent of forest needs to be protected to avoid an irreversible tipping point; thus, the need for cumulative action by all Amazon basin countries is required to curb the deforestation in the Amazon rainforests.
    The declaration promotes sustainable use of biodiversity resources in the Amazon. At the moment, there is no indication of how much money members will invest to fund the declaration’s proposed objectives or support minimum conservation standards.
    Additional information:
    ● The Amazon Cooperation Treaty Organization (ACTO) is an intergovernmental organization formed by the eight Amazonian countries: Bolivia, Brazil, Colombia, Ecuador, Guyana, Peru, Suriname, and Venezuela, which signed the Amazon Cooperation Treaty (ACT), becoming the only socio-environmental block in Latin America.
    ● The Amazon Cooperation Treaty (ACT) signed on July 3, 1978 gave rise to the ACTO, that is oriented to promote the harmonious development of the Amazonian territories in such a way that the joint actions of the Amazonian countries produce equitable and mutually beneficial results in achieving the sustainable development of the Amazon Region.
    ● In 1995, the eight nations decided to create the Amazon Cooperation Treaty Organization (ACTO), to strengthen and implement the objectives of the Amazon Cooperation Treaty.
    ● Permanent Secretariat of ACTO was established in Brasilia (Brazil) on December 13, 2002.

  3. Question 3 of 5
    3. Question

    3. In the context of the International Blue Carbon Partnership (IBCP), consider the following statements:
    1. It was launched at the UNFCCC COP27 in Sharm-el-Sheikh.
    2. IUCN and WWF collaborated to launch the IBCP.
    Which of the statements given above is/are correct?

    Correct

    Answer: D
    Explanation:
    Statement 1 is incorrect: The International Partnership for Blue Carbon (IPBC) was launched at the United Nations Framework Convention on Climate Change (UNFCCC) Conference of the Parties (COP21) in Paris in 2015.
    It connects government agencies with non-governmental organisations, intergovernmental organisations and research institutions from around the world, with a joint vision to protect, sustainably manage and restore global coastal blue carbon ecosystems contributing to climate change mitigation, adaptation, biodiversity, ocean economies and livelihoods of coastal communities.
    Statement 2 is incorrect: The International Partnership for Blue Carbon is the collaborative effort of:
    ● Australia, Indonesia, Costa Rica,
    ● the Blue Carbon Initiative (Intergovernmental Oceanographic Commission – UNESCO, Conservation International, IUCN),
    ● GRID-Arendal,
    ● the Secretariat of the Pacific Regional Environment Program (SPREP),
    ● the Pacific Islands Forum Secretariat and Office of the Pacific Oceanscape Commissioner,
    ● the Centre for International Forestry Research; and the Global Change Institute.
    Additional Information:
    ● The term ‘coastal blue carbon ecosystems’ refers to three main types of vegetated coastal habitats – mangroves, tidal marshes and seagrasses. When degraded or lost, coastal blue carbon ecosystems can become significant emission sources.
    ● Mangrove deforestation is estimated to be responsible for as much as 10 per cent of emissions from deforestation globally.

    Incorrect

    Answer: D
    Explanation:
    Statement 1 is incorrect: The International Partnership for Blue Carbon (IPBC) was launched at the United Nations Framework Convention on Climate Change (UNFCCC) Conference of the Parties (COP21) in Paris in 2015.
    It connects government agencies with non-governmental organisations, intergovernmental organisations and research institutions from around the world, with a joint vision to protect, sustainably manage and restore global coastal blue carbon ecosystems contributing to climate change mitigation, adaptation, biodiversity, ocean economies and livelihoods of coastal communities.
    Statement 2 is incorrect: The International Partnership for Blue Carbon is the collaborative effort of:
    ● Australia, Indonesia, Costa Rica,
    ● the Blue Carbon Initiative (Intergovernmental Oceanographic Commission – UNESCO, Conservation International, IUCN),
    ● GRID-Arendal,
    ● the Secretariat of the Pacific Regional Environment Program (SPREP),
    ● the Pacific Islands Forum Secretariat and Office of the Pacific Oceanscape Commissioner,
    ● the Centre for International Forestry Research; and the Global Change Institute.
    Additional Information:
    ● The term ‘coastal blue carbon ecosystems’ refers to three main types of vegetated coastal habitats – mangroves, tidal marshes and seagrasses. When degraded or lost, coastal blue carbon ecosystems can become significant emission sources.
    ● Mangrove deforestation is estimated to be responsible for as much as 10 per cent of emissions from deforestation globally.

  4. Question 4 of 5
    4. Question

    4. In the context of Plastic Overshoot Day 2023, consider the following statements:
    1. Plastic Overshoot Day Report is published by the Global Footprint Network.
    2. Plastic Overshoot Day is determined based on a country’s Mismanaged Waste Index (MWI).
    3. India ranks first in the Mismanaged Waste Index (MWI).
    How many of the above statements are correct?

    Correct

    Answer: A
    Explanation:
    Statement 1 is incorrect: On July 28, 2023, the Earth saw its first Plastic Overshoot Day: The point at which the amount of plastics exceeds the global waste management capacity. The Report has been released by Swiss-based research consultancy Earth Action (EA). According to the Report, under current scenarios, despite pledges and increased waste management capacity, increased production of plastics will lead to global plastics pollution tripling by 2040.
    Global Footprint Network brings out the Earth Overshoot Day report. Earth Overshoot Day marks the date when humanity has exhausted nature’s budget for the year. In 2023, Earth Overshoot Day fell on August 2, 2023.
    Earth Overshoot Day is computed by dividing the planet’s biocapacity (the amount of ecological resources Earth is able to generate that year), by humanity’s Ecological Footprint (humanity’s demand for that year), and multiplying by 365, the number of days in a year.
    Statement 2 is correct: Plastic Overshoot Day is determined based on a country’s Mismanaged Waste Index (MWI). The gap in waste management capacity and plastic consumption is called MWI.
    Plastic Overshoot Day for India, or the date when the amount of plastic waste outweighed the country’s ability to manage it, was January 6 2023.
    Statement 3 is incorrect: India ranks fourth in the MWI, with 98.55 per cent of generated waste being mismanaged and fares poorly in the management of plastics waste, according to the EA report.
    Additional Information:
    ● India is among the 12 countries, along with China, Brazil, Indonesia, Thailand, Russia, Mexico, the United States, Saudi Arabia, the Democratic Republic of Congo, Iran and Kazakhstan, which are responsible for 52 per cent of the world’s mismanaged plastic waste.
    ● Iceland fares the worst when it comes to plastic consumption on a per-person basis, with annual consumption of 128.9 kg per person. This is 24.3 times higher than the yearly consumption per person of 5.3 kg in India.
    ● The global average consumption of plastic per person per year is 20.9 kg.

    Incorrect

    Answer: A
    Explanation:
    Statement 1 is incorrect: On July 28, 2023, the Earth saw its first Plastic Overshoot Day: The point at which the amount of plastics exceeds the global waste management capacity. The Report has been released by Swiss-based research consultancy Earth Action (EA). According to the Report, under current scenarios, despite pledges and increased waste management capacity, increased production of plastics will lead to global plastics pollution tripling by 2040.
    Global Footprint Network brings out the Earth Overshoot Day report. Earth Overshoot Day marks the date when humanity has exhausted nature’s budget for the year. In 2023, Earth Overshoot Day fell on August 2, 2023.
    Earth Overshoot Day is computed by dividing the planet’s biocapacity (the amount of ecological resources Earth is able to generate that year), by humanity’s Ecological Footprint (humanity’s demand for that year), and multiplying by 365, the number of days in a year.
    Statement 2 is correct: Plastic Overshoot Day is determined based on a country’s Mismanaged Waste Index (MWI). The gap in waste management capacity and plastic consumption is called MWI.
    Plastic Overshoot Day for India, or the date when the amount of plastic waste outweighed the country’s ability to manage it, was January 6 2023.
    Statement 3 is incorrect: India ranks fourth in the MWI, with 98.55 per cent of generated waste being mismanaged and fares poorly in the management of plastics waste, according to the EA report.
    Additional Information:
    ● India is among the 12 countries, along with China, Brazil, Indonesia, Thailand, Russia, Mexico, the United States, Saudi Arabia, the Democratic Republic of Congo, Iran and Kazakhstan, which are responsible for 52 per cent of the world’s mismanaged plastic waste.
    ● Iceland fares the worst when it comes to plastic consumption on a per-person basis, with annual consumption of 128.9 kg per person. This is 24.3 times higher than the yearly consumption per person of 5.3 kg in India.
    ● The global average consumption of plastic per person per year is 20.9 kg.

  5. Question 5 of 5
    5. Question

    5. Consider the following statements about African Lions and Asiatic Lions:
    1. African lions are well-adapted to survive in open scrublands and deserts unlike Asiatic lions which are only found in the deciduous forests of India.
    2. The longitudinal fold of skin under its belly distinguishes African lion from Asiatic lion which lacks such skin folds.
    3. Both African lions and Asiatic lions are listed as ‘Endangered’ as per the IUCN Red List.
    How many of the above statements are correct?

    Correct

    Answer: D
    Explanation:
    Statement 1 is incorrect: African lions are highly adaptable big cats and can be found in a wide variety of habitats, including semi-arid desert areas and even very dry habitats such as the Kalahari desert. They prefer open woodland and thick bush, as well as scrub and grasslands, to help them stalk their prey and also find shade to rest during the hottest part of the day.
    African lions used to be spread across most of the continent, but now are only found in sub-Saharan Africa, with 80% in eastern or southern Africa. Three of the five largest populations are in Tanzania.
    On the other hand, Asiatic lions have an extremely restricted habitat. These lions can only be found in the Gir Forest National Park in the Indian state of Gujarat. This park is a protected area and is considered one of the last strongholds of the Asiatic lion. The park’s terrain is a mixture of dry deciduous forest and savanna grasslands providing an ideal habitat for these big cats.
    Statement 2 is incorrect: The most striking morphological character, which is always seen in Asiatic lions, and rarely in African lions, is a longitudinal fold of skin running along its belly. African lions are larger in size than the Asiatic lions.
    Statement 3 is incorrect:
    IUCN status of Asiatic Lion: Endangered
    IUCN status of African lion: Vulnerable. Three-quarters of their populations are in decline. The main threats to African lions are human-wildlife conflict and natural prey decline, as well as habitat loss, climate change and wildlife trade.
    Differences between African lion and Asiatic lion in a nutshell-
    PARAMETER – AFRICAN LION – ASIATIC LION
    Size – Bigger in size than Asiatic counterparts.A male lion weighs about 500 pounds with females typically weighing about 345 pounds. – Relatively smaller in size. The males weigh about 35-450 pounds.
    Mane (Only male lions have mane.) – Have a fuller and more prominent mane covering entire head and cascading down their shoulders. – Have much sparser, darker and less developed mane compared to that of African lions.It is also shorter due to which ears of Asiatic lion are more visible.
    Skin folds – Do not have skin folds. Instead, they have a smooth belly, contributing to their regal and powerful look. – Impressive skin folds are another feature that sets Asiatic lions apart from their cousins.
    Tail and elbow tufts – They have minimal tufts of hair on their elbows and tails, giving them a sleeker appearance. – Have a more pronounced and flamboyant style of tail hair and elbow hair.
    Skull – They have only a single infraorbital foramen. – They have bifurcated infraorbital foramina. These tiny openings in the skull ensure the smooth flow of blood and protect the nerves of the eyes.

    Additional Information-
    A recent study ‘Socio-political and ecological fragility of threatened, free-ranging African lion populations’, published in journal Nature Communications, observed that socio-political factors were threatening already fragile lion populations in Africa. Somalian and Malawian big cat populations were found to be the most threatened and Ethiopia’s Maze National Park had the most ecologically fragile geographic population.
    ● Threats to African lions: poaching for prey, indiscriminate killing due to human-lion conflict, bushmeat and others, prey depletion, livestock encroachment and small population size.
    ● The researchers found that bushmeat poaching with snares led to local extinction of lions in Nsumbu National Park in Zambia and Limpopo National Park in Mozambique, also driving lion populations to near extinction owing to poaching.

    Incorrect

    Answer: D
    Explanation:
    Statement 1 is incorrect: African lions are highly adaptable big cats and can be found in a wide variety of habitats, including semi-arid desert areas and even very dry habitats such as the Kalahari desert. They prefer open woodland and thick bush, as well as scrub and grasslands, to help them stalk their prey and also find shade to rest during the hottest part of the day.
    African lions used to be spread across most of the continent, but now are only found in sub-Saharan Africa, with 80% in eastern or southern Africa. Three of the five largest populations are in Tanzania.
    On the other hand, Asiatic lions have an extremely restricted habitat. These lions can only be found in the Gir Forest National Park in the Indian state of Gujarat. This park is a protected area and is considered one of the last strongholds of the Asiatic lion. The park’s terrain is a mixture of dry deciduous forest and savanna grasslands providing an ideal habitat for these big cats.
    Statement 2 is incorrect: The most striking morphological character, which is always seen in Asiatic lions, and rarely in African lions, is a longitudinal fold of skin running along its belly. African lions are larger in size than the Asiatic lions.
    Statement 3 is incorrect:
    IUCN status of Asiatic Lion: Endangered
    IUCN status of African lion: Vulnerable. Three-quarters of their populations are in decline. The main threats to African lions are human-wildlife conflict and natural prey decline, as well as habitat loss, climate change and wildlife trade.
    Differences between African lion and Asiatic lion in a nutshell-
    PARAMETER – AFRICAN LION – ASIATIC LION
    Size – Bigger in size than Asiatic counterparts.A male lion weighs about 500 pounds with females typically weighing about 345 pounds. – Relatively smaller in size. The males weigh about 35-450 pounds.
    Mane (Only male lions have mane.) – Have a fuller and more prominent mane covering entire head and cascading down their shoulders. – Have much sparser, darker and less developed mane compared to that of African lions.It is also shorter due to which ears of Asiatic lion are more visible.
    Skin folds – Do not have skin folds. Instead, they have a smooth belly, contributing to their regal and powerful look. – Impressive skin folds are another feature that sets Asiatic lions apart from their cousins.
    Tail and elbow tufts – They have minimal tufts of hair on their elbows and tails, giving them a sleeker appearance. – Have a more pronounced and flamboyant style of tail hair and elbow hair.
    Skull – They have only a single infraorbital foramen. – They have bifurcated infraorbital foramina. These tiny openings in the skull ensure the smooth flow of blood and protect the nerves of the eyes.

    Additional Information-
    A recent study ‘Socio-political and ecological fragility of threatened, free-ranging African lion populations’, published in journal Nature Communications, observed that socio-political factors were threatening already fragile lion populations in Africa. Somalian and Malawian big cat populations were found to be the most threatened and Ethiopia’s Maze National Park had the most ecologically fragile geographic population.
    ● Threats to African lions: poaching for prey, indiscriminate killing due to human-lion conflict, bushmeat and others, prey depletion, livestock encroachment and small population size.
    ● The researchers found that bushmeat poaching with snares led to local extinction of lions in Nsumbu National Park in Zambia and Limpopo National Park in Mozambique, also driving lion populations to near extinction owing to poaching.

window.wpAdvQuizInitList = window.wpAdvQuizInitList || []; window.wpAdvQuizInitList.push({ id: '#wpAdvQuiz_570', init: { quizId: 570, mode: 0, globalPoints: 10, timelimit: 0, resultsGrade: [0], bo: 0, qpp: 0, catPoints: [10], formPos: 0, lbn: "Finish quiz", json: {"2643":{"type":"single","id":2643,"catId":0,"points":2,"correct":[0,0,0,1]},"2644":{"type":"single","id":2644,"catId":0,"points":2,"correct":[0,0,0,1]},"2645":{"type":"single","id":2645,"catId":0,"points":2,"correct":[0,0,0,1]},"2646":{"type":"single","id":2646,"catId":0,"points":2,"correct":[1,0,0,0]},"2647":{"type":"single","id":2647,"catId":0,"points":2,"correct":[0,0,0,1]}} } });




Day-530 | Daily MCQs | UPSC Prelims | SCIENCE AND TECHNOLOGY

Day-530

Time limit: 0

Quiz-summary

0 of 5 questions completed

Questions:

  1. 1
  2. 2
  3. 3
  4. 4
  5. 5

Information

DAILY MCQ

You have already completed the quiz before. Hence you can not start it again.

Quiz is loading...

You must sign in or sign up to start the quiz.

You have to finish following quiz, to start this quiz:

Results

0 of 5 questions answered correctly

Your time:

Time has elapsed

You have reached 0 of 0 points, (0)

Categories

  1. Not categorized 0%
  1. 1
  2. 2
  3. 3
  4. 4
  5. 5
  1. Answered
  2. Review
  1. Question 1 of 5
    1. Question

    1. Consider the following statements:
    Statement I: Kulasekharapatnam Spaceport is being developed as the second spaceport of the Indian Space Research Organisation.
    Statement II: It will help in cost effective launch of small satellites in geostationary orbit.
    Which one of the following is correct in respect of the above statements?

    Correct

    Answer: C
    Explanation:
    Context:
    The ISRO has planned to set up a second launch site and had identified the coastal town in Tamil Nadu for taking up missions that require small satellite launch vehicles as compared to the one in Sriharikota which is currently in use for missions undertaken by the space agency. The geographic location of the country’s second spaceport to be set up at Kulasekarapattinam in Tamil Nadu has a strategic advantage over launch pads in Sriharikota — small satellite launch vehicles (SSLV) can fly straight to the south pole without burning fuel to swerve around Sri Lanka on the way.

    ● Statement 1 is correct: Kulasekharapatnam Spaceport is being developed as the second spaceport of the Indian Space Research Organisation near Kulasekharapatnam town of southern India. It is located in Thoothukudi district, Tamil Nadu.
    ● Statement 2 is incorrect: It is inefficient and costly to launch small satellite launch vehicle (SSLV) which carries small payloads into polar orbit from Satish Dhawan Space Centre, Sriharikota. This is because if a satellite is directly launched in south direction for polar orbit insertion, there is a risk of direct crossing of launch vehicle over Sri Lanka which may pose a risk in case of an unfortunate mission failure. To avoid this risk, the rocket has to follow curved path. This increases fuel cost and reduces payload capacity. The new space port will ensure straight line trajectory which will be fuel efficient.

    Incorrect

    Answer: C
    Explanation:
    Context:
    The ISRO has planned to set up a second launch site and had identified the coastal town in Tamil Nadu for taking up missions that require small satellite launch vehicles as compared to the one in Sriharikota which is currently in use for missions undertaken by the space agency. The geographic location of the country’s second spaceport to be set up at Kulasekarapattinam in Tamil Nadu has a strategic advantage over launch pads in Sriharikota — small satellite launch vehicles (SSLV) can fly straight to the south pole without burning fuel to swerve around Sri Lanka on the way.

    ● Statement 1 is correct: Kulasekharapatnam Spaceport is being developed as the second spaceport of the Indian Space Research Organisation near Kulasekharapatnam town of southern India. It is located in Thoothukudi district, Tamil Nadu.
    ● Statement 2 is incorrect: It is inefficient and costly to launch small satellite launch vehicle (SSLV) which carries small payloads into polar orbit from Satish Dhawan Space Centre, Sriharikota. This is because if a satellite is directly launched in south direction for polar orbit insertion, there is a risk of direct crossing of launch vehicle over Sri Lanka which may pose a risk in case of an unfortunate mission failure. To avoid this risk, the rocket has to follow curved path. This increases fuel cost and reduces payload capacity. The new space port will ensure straight line trajectory which will be fuel efficient.

  2. Question 2 of 5
    2. Question

    2. The ‘Artemis Mission’, often seen in the news is-

    Correct

    Answer: B
    Explanation
    Context: India has recently landed Chandrayaan 3 on Moon’s South Pole. This has increased interest among the global fraternity in NASA’s Artemis mission.
    With Artemis mission, NASA will land the first woman and first person of color on the Moon, using innovative technologies to explore more of the lunar surface than ever before. NASA will collaborate with commercial and international partners and establish the first long-term presence on the Moon. Then, the NASA will use what we learn on and around the Moon to take the next giant leap.

    Incorrect

    Answer: B
    Explanation
    Context: India has recently landed Chandrayaan 3 on Moon’s South Pole. This has increased interest among the global fraternity in NASA’s Artemis mission.
    With Artemis mission, NASA will land the first woman and first person of color on the Moon, using innovative technologies to explore more of the lunar surface than ever before. NASA will collaborate with commercial and international partners and establish the first long-term presence on the Moon. Then, the NASA will use what we learn on and around the Moon to take the next giant leap.

  3. Question 3 of 5
    3. Question

    3. Consider the following statements:
    1. The nuclear submarines can operate underwater for a longer period as compared to diesel submarines.
    2. The nuclear submarines are bulkier than diesel submarines.
    Which of the above given statements is/are NOT correct?

    Correct

    Answer: D
    Explanation:
    Context: India is looking to fast track talks with France for three additional submarines as Pakistan is set to get four Chinese Yuan attack subs this year, marking a paradigm shift in Islamabad’s naval fleet. Mazagon Dock Shipbuilders, the state-run company which makes ships and submarines, is in talks with France’s Naval Group for building three advanced submarines at its Mumbai dockyard.

    ● Statement 1 is correct: The nuclear submarines use air independent propulsion system since they are powered by nuclear energy to generate steam to run the submarine. Air is not used for nuclear fission energy. On the other hand, diesel submarines require air for combustion to generate power which is used to recharge batteries. When diesel submarines go underwater, it is the battery which powers the submarines. So, a diesel submarine has to come to the surface frequently which reduces its stealth capability.
    ● Statement 2 is correct: The nuclear submarines are bulkier which reduces its swift movement. However, diesel submarine offers a wider range of customisation in terms of size. Moreover, they operate swiftly in shallow water.

    Incorrect

    Answer: D
    Explanation:
    Context: India is looking to fast track talks with France for three additional submarines as Pakistan is set to get four Chinese Yuan attack subs this year, marking a paradigm shift in Islamabad’s naval fleet. Mazagon Dock Shipbuilders, the state-run company which makes ships and submarines, is in talks with France’s Naval Group for building three advanced submarines at its Mumbai dockyard.

    ● Statement 1 is correct: The nuclear submarines use air independent propulsion system since they are powered by nuclear energy to generate steam to run the submarine. Air is not used for nuclear fission energy. On the other hand, diesel submarines require air for combustion to generate power which is used to recharge batteries. When diesel submarines go underwater, it is the battery which powers the submarines. So, a diesel submarine has to come to the surface frequently which reduces its stealth capability.
    ● Statement 2 is correct: The nuclear submarines are bulkier which reduces its swift movement. However, diesel submarine offers a wider range of customisation in terms of size. Moreover, they operate swiftly in shallow water.

  4. Question 4 of 5
    4. Question

    4. ‘Vulcan 20-20 Project’, which was recently in the news, is related to which of the following?

    Correct

    Answer: A
    Explanation:
    Context: The UK is undertaking a groundbreaking project to create the world’s most powerful laser, called “Vulcan 20-20,”. This laser, anticipated to be a million billion times brighter than the sun, is expected to be helpful in various scientific fields, including clean energy and cancer treatment.
    Vulcan 20-20 will be able to study a range of subjects in brand new detail, for example:
    ● This laser will generate a primary beam with a staggering energy output of 20 Petawatts (PW), accompanied by eight high-energy beams with outputs of up to 20 Kilojoules (KJ). That’s why the name Vulcan 20 20 Project.
    ● Learning about astrophysical phenomena by using lasers as tools to replicate interstellar conditions.
    ● Aiding in research towards a potential new frontier for clean energy, laser fusion.
    ● Expand the possibilities of plasma physics and its technological applications.
    ● The ground-breaking power of Vulcan 20-20 will be able to conduct physics experiments that step into brand new unknowns and explore beyond the present Standard Model.
    ● It is expected to create numerous skilled jobs in science and engineering, contributing to the UK’s science sector and overall economic growth.

    Incorrect

    Answer: A
    Explanation:
    Context: The UK is undertaking a groundbreaking project to create the world’s most powerful laser, called “Vulcan 20-20,”. This laser, anticipated to be a million billion times brighter than the sun, is expected to be helpful in various scientific fields, including clean energy and cancer treatment.
    Vulcan 20-20 will be able to study a range of subjects in brand new detail, for example:
    ● This laser will generate a primary beam with a staggering energy output of 20 Petawatts (PW), accompanied by eight high-energy beams with outputs of up to 20 Kilojoules (KJ). That’s why the name Vulcan 20 20 Project.
    ● Learning about astrophysical phenomena by using lasers as tools to replicate interstellar conditions.
    ● Aiding in research towards a potential new frontier for clean energy, laser fusion.
    ● Expand the possibilities of plasma physics and its technological applications.
    ● The ground-breaking power of Vulcan 20-20 will be able to conduct physics experiments that step into brand new unknowns and explore beyond the present Standard Model.
    ● It is expected to create numerous skilled jobs in science and engineering, contributing to the UK’s science sector and overall economic growth.

  5. Question 5 of 5
    5. Question

    5. With reference to Ribonucleic Acids (RNAs), consider the following statements:
    1. Circular RNAs are less stable than linear RNAs.
    2. Circular RNAs are only found in bacteria.
    3. Circular RNAs have been linked to life threatening diseases such as cancer and AIDS.
    How many of the above statements are correct?

    Correct

    Answer: A
    Explanation:
    Context: Researchers from the Indian Institute of Science Education and Research Bhopal (IISER Bhopal) identified a specific circular RNA (circRNA) called ‘ciTRAN’, which plays a crucial role in the multiplication of the AIDS-causing HIV-1 virus within the human body.

    ● Statement 1 is incorrect: Circular RNAs are more stable than linear RNAs because circular RNAs form extra covalent bond creating a continuous closed loop. Because circular RNA does not have ends, they are resistant to exonuclease-mediated degradation and are presumably more stable than most linear RNA in cells.
    ● Statement 2 is incorrect: Circular RNAs are found in all eukaryotes. Recent studies have confirmed their presence even in viruses which remain inactive outside host cells.
    ● Statement 3 is correct: Circular RNAs have been linked to diseases like AIDS and cancer. The RNA or Ribonucleic acid is a molecule in living cells that carries genetic information and helps in the production of proteins. RNAs are in general straight-chain, free-end structures but one form of RNA called ‘circRNA’ forms a closed-loop. The circRNA plays a pivotal role in regulating gene expression and is essential for various biological processes.

    Incorrect

    Answer: A
    Explanation:
    Context: Researchers from the Indian Institute of Science Education and Research Bhopal (IISER Bhopal) identified a specific circular RNA (circRNA) called ‘ciTRAN’, which plays a crucial role in the multiplication of the AIDS-causing HIV-1 virus within the human body.

    ● Statement 1 is incorrect: Circular RNAs are more stable than linear RNAs because circular RNAs form extra covalent bond creating a continuous closed loop. Because circular RNA does not have ends, they are resistant to exonuclease-mediated degradation and are presumably more stable than most linear RNA in cells.
    ● Statement 2 is incorrect: Circular RNAs are found in all eukaryotes. Recent studies have confirmed their presence even in viruses which remain inactive outside host cells.
    ● Statement 3 is correct: Circular RNAs have been linked to diseases like AIDS and cancer. The RNA or Ribonucleic acid is a molecule in living cells that carries genetic information and helps in the production of proteins. RNAs are in general straight-chain, free-end structures but one form of RNA called ‘circRNA’ forms a closed-loop. The circRNA plays a pivotal role in regulating gene expression and is essential for various biological processes.

window.wpAdvQuizInitList = window.wpAdvQuizInitList || []; window.wpAdvQuizInitList.push({ id: '#wpAdvQuiz_569', init: { quizId: 569, mode: 0, globalPoints: 10, timelimit: 0, resultsGrade: [0], bo: 0, qpp: 0, catPoints: [10], formPos: 0, lbn: "Finish quiz", json: {"2638":{"type":"single","id":2638,"catId":0,"points":2,"correct":[0,0,1,0]},"2639":{"type":"single","id":2639,"catId":0,"points":2,"correct":[0,1,0,0]},"2640":{"type":"single","id":2640,"catId":0,"points":2,"correct":[0,0,0,1]},"2641":{"type":"single","id":2641,"catId":0,"points":2,"correct":[1,0,0,0]},"2642":{"type":"single","id":2642,"catId":0,"points":2,"correct":[1,0,0,0]}} } });




Day-529 | Daily MCQs | UPSC Prelims | POLITY

Day-529

Time limit: 0

Quiz-summary

0 of 5 questions completed

Questions:

  1. 1
  2. 2
  3. 3
  4. 4
  5. 5

Information

To attempt the Quiz, simply click on START Button.

You have already completed the quiz before. Hence you can not start it again.

Quiz is loading...

You must sign in or sign up to start the quiz.

You have to finish following quiz, to start this quiz:

Results

0 of 5 questions answered correctly

Your time:

Time has elapsed

You have reached 0 of 0 points, (0)

Average score
 
 
Your score
 
 

Categories

  1. Not categorized 0%
Your result has been entered into leaderboard
Loading
captcha
  1. 1
  2. 2
  3. 3
  4. 4
  5. 5
  1. Answered
  2. Review
  1. Question 1 of 5
    1. Question
    2 points

    1. Consider the following statements in the context of Narcotic Drugs and Psychotropic Substances (NDPS) Act, 1985:
    1. The Act completely prohibits the cultivation of opium poppy or any cannabis plant in India.
    2. Both the central and state governments have the authority to add or omit the name of a substance from the list of psychotropic substances.
    3. As per the Act, the seized sample of the contraband should be duly certified by the Gazetted Officer to qualify as evidence in the court of law.
    How many of the above given statements are correct?

    Correct

    Answer: D
    Explanation:
    Statement 1 is incorrect: Section 8 of the NDPS Act, prohibits the cultivation of opium poppy or any cannabis plant except for medical or scientific purposes. Government may impose any restriction by way of licence, permit or authorisation.
    Statement 2 is incorrect: Power to add to or omit from the list of psychotropic substances lies with the Central Government. This can be done for two reasons as specified in the Act:
    ● If the new information of the effects of any substance (natural or synthetic) has become available
    ● To give effect to the modifications or provisions (if any) which have been made to any International Convention with respect to such substance.
    Statement 3 is incorrect: Section 52A of the NDPS Act mandates that for a seized contraband to be the primary evidence in the trial, has to be duly certified by the Magistrate.
    In a recent judgment given by the Supreme Court in Yusuf @ Asif V. State (2023) the apex court held that the procedure and manner of seizing, preparing the inventory of the seized material, forwarding the seized material and getting the inventory certified by the Magistrate concerned, are all laid out under Section 52A of the NDPS Act. The said provisions also provide that the inventory or the photographs of the seized substance and any list of the samples seized, on being certified by the Magistrate can be recognized as the primary evidence in the trial.

    Incorrect

    Answer: D
    Explanation:
    Statement 1 is incorrect: Section 8 of the NDPS Act, prohibits the cultivation of opium poppy or any cannabis plant except for medical or scientific purposes. Government may impose any restriction by way of licence, permit or authorisation.
    Statement 2 is incorrect: Power to add to or omit from the list of psychotropic substances lies with the Central Government. This can be done for two reasons as specified in the Act:
    ● If the new information of the effects of any substance (natural or synthetic) has become available
    ● To give effect to the modifications or provisions (if any) which have been made to any International Convention with respect to such substance.
    Statement 3 is incorrect: Section 52A of the NDPS Act mandates that for a seized contraband to be the primary evidence in the trial, has to be duly certified by the Magistrate.
    In a recent judgment given by the Supreme Court in Yusuf @ Asif V. State (2023) the apex court held that the procedure and manner of seizing, preparing the inventory of the seized material, forwarding the seized material and getting the inventory certified by the Magistrate concerned, are all laid out under Section 52A of the NDPS Act. The said provisions also provide that the inventory or the photographs of the seized substance and any list of the samples seized, on being certified by the Magistrate can be recognized as the primary evidence in the trial.

  2. Question 2 of 5
    2. Question
    2 points

    2. Consider the following statements in the context of remote voting:
    1. It will address the issue of migration induced ‘disenfranchisement of voters’.
    2. It will help registered voters to exercise their constitutional right.
    3. Remote voting solutions may enhance people’s participation in the elections.
    How many of the above given statements are correct?

    Correct

    Answer: B
    Explanation:
    Statement 1 is correct: Migration based disenfranchisement is not an option in the age of technological advancement. The voter turnout in General Elections 2019 was 67.4 % and the Election Commission of India is concerned about the issue of over 30 crore electors not exercising their franchise and also differential voter turnout in various states/UTs. Underlining the problems faced by domestic migrants travelling back home to vote, the Election Commission (EC) has developed remote polling stations.
    Statement 2 is incorrect: Right to vote is a statutory right. Though at various instances the judges of the Supreme Court have opined that it shall be the constitutional right.
    Statement 3 is correct: Since many of the migrant workers are employed at far off places from their home constituencies. Factors such as cost incurred in travelling, health and old age restricts them to cast their vote in home constituencies. Such inabilities to Vote are also cited by various governmental and non-governmental organisations as a factor for keeping the voter turnout low. The facility of remote voting will surely help in improving the voter turnouts.

    Incorrect

    Answer: B
    Explanation:
    Statement 1 is correct: Migration based disenfranchisement is not an option in the age of technological advancement. The voter turnout in General Elections 2019 was 67.4 % and the Election Commission of India is concerned about the issue of over 30 crore electors not exercising their franchise and also differential voter turnout in various states/UTs. Underlining the problems faced by domestic migrants travelling back home to vote, the Election Commission (EC) has developed remote polling stations.
    Statement 2 is incorrect: Right to vote is a statutory right. Though at various instances the judges of the Supreme Court have opined that it shall be the constitutional right.
    Statement 3 is correct: Since many of the migrant workers are employed at far off places from their home constituencies. Factors such as cost incurred in travelling, health and old age restricts them to cast their vote in home constituencies. Such inabilities to Vote are also cited by various governmental and non-governmental organisations as a factor for keeping the voter turnout low. The facility of remote voting will surely help in improving the voter turnouts.

  3. Question 3 of 5
    3. Question
    2 points

    3. Consider the following statements:
    Statement-I: NALSA was established to promote an inclusive legal system in order to ensure fair and meaningful justice to the marginalized and disadvantaged sections of the society.
    Statement-II: Article 39A of the Constitution of India provides that State shall provide free legal aid.
    Which one of the following is correct in respect of the above statements?

    Correct

    Answer: A
    Explanation:
    Both Statement-I and Statement-II are correct and Statement-II is the correct explanation for Statement-I.
    Statement-I is correct: In 1987, the Legal Services Authorities (LSA) Act was enacted to give free and competent legal services to the poor and paved the way for the constitution of the National Legal Service Authority (NALSA) and other legal service institutions at the State, district and taluka levels.
    Free legal services under LSA Act are available to a person belonging to Schedule Tribe and Schedule Caste, a woman, child, victim of human trafficking, a differently abled person, an industrial workman, and a person in custody in a protective home and the poor.
    Statement-II is correct: Article 39A of the Constitution of India provides that State shall secure that the operation of the legal system promotes justice on a basis of equal opportunity, and shall in particular, provide free legal aid, by suitable legislation or schemes or in any other way, to ensure that opportunities for securing justice are not denied to any citizen by reason of economic or other disability.
    Articles 14 and 22(1) also make it obligatory for the State to ensure equality before law and a legal system which promotes justice on a basis of equal opportunity to all. Legal aid strives to ensure that constitutional pledge is fulfilled in its letter and spirit and equal justice is made available to the poor, downtrodden and weaker sections of the society.
    In two important judgments A.M.Hoskot v. State of Maharashtra (1978) and Hussainara Khatoon v. Home Secretary, State of Bihar (1979) the Supreme Court held that legal aid and speedy trial are to treated as a part of the fundamental rights under Article 21 of the Constitution. It is enforceable by the Courts and the State is under a duty to provide legal aid and legal assistance to a poor and needy person at its own expense.

    Incorrect

    Answer: A
    Explanation:
    Both Statement-I and Statement-II are correct and Statement-II is the correct explanation for Statement-I.
    Statement-I is correct: In 1987, the Legal Services Authorities (LSA) Act was enacted to give free and competent legal services to the poor and paved the way for the constitution of the National Legal Service Authority (NALSA) and other legal service institutions at the State, district and taluka levels.
    Free legal services under LSA Act are available to a person belonging to Schedule Tribe and Schedule Caste, a woman, child, victim of human trafficking, a differently abled person, an industrial workman, and a person in custody in a protective home and the poor.
    Statement-II is correct: Article 39A of the Constitution of India provides that State shall secure that the operation of the legal system promotes justice on a basis of equal opportunity, and shall in particular, provide free legal aid, by suitable legislation or schemes or in any other way, to ensure that opportunities for securing justice are not denied to any citizen by reason of economic or other disability.
    Articles 14 and 22(1) also make it obligatory for the State to ensure equality before law and a legal system which promotes justice on a basis of equal opportunity to all. Legal aid strives to ensure that constitutional pledge is fulfilled in its letter and spirit and equal justice is made available to the poor, downtrodden and weaker sections of the society.
    In two important judgments A.M.Hoskot v. State of Maharashtra (1978) and Hussainara Khatoon v. Home Secretary, State of Bihar (1979) the Supreme Court held that legal aid and speedy trial are to treated as a part of the fundamental rights under Article 21 of the Constitution. It is enforceable by the Courts and the State is under a duty to provide legal aid and legal assistance to a poor and needy person at its own expense.

  4. Question 4 of 5
    4. Question
    2 points

    4. Which of the following is the most appropriate meaning of ‘doctrine of parens patriae’?

    Correct

    Answer: A
    Explanation:
    Statement 1 is correct: Parens patriae is a Latin term meaning “parent of the fatherland.” It is a legal term that refers to the government’s power to act as the legal guardian for people who are unable to care for themselves. Parens patriae is most commonly applied to cases regarding the custody and care of minor children and disabled adults. However, parens patriae is also applied in lawsuits between the states and in suits dealing with the wellbeing of a state’s entire population, e.g. environmental concerns or natural disasters.

    Incorrect

    Answer: A
    Explanation:
    Statement 1 is correct: Parens patriae is a Latin term meaning “parent of the fatherland.” It is a legal term that refers to the government’s power to act as the legal guardian for people who are unable to care for themselves. Parens patriae is most commonly applied to cases regarding the custody and care of minor children and disabled adults. However, parens patriae is also applied in lawsuits between the states and in suits dealing with the wellbeing of a state’s entire population, e.g. environmental concerns or natural disasters.

  5. Question 5 of 5
    5. Question
    2 points

    5. In the context of the President of India, which of the following statements is incorrect?

    Correct

    Answer: C
    Explanation:
    Option A is correct: The Indian Parliament has a number of quasi-judicial functions, which include the impeachment process for the removal of the President, Vice-president, judges of the Supreme Court and High Courts.
    Option B is correct: Under Article 61 of the Constitution of India, the President of India can be impeached only for the violation of the Constitution and that the charges against the President can be initiated by either of the House of the Parliament. Hence, Option D is also correct.
    Option C is incorrect: The Constitution of India does not explicitly define the ‘violation of the Constitution’ for the purpose of impeachment of the President.

    Additional Information:
    ● The notice for the charges against the President must be signed by at least a quarter of the members of the House.
    ● The resolution to impeach the President must be passed by a special majority (two-thirds) in the originating House.
    ● The other House acts as the investigating House. A select committee investigates the charges labelled against the President.
    ● The President of India has the right to defend himself against the charges.

    Incorrect

    Answer: C
    Explanation:
    Option A is correct: The Indian Parliament has a number of quasi-judicial functions, which include the impeachment process for the removal of the President, Vice-president, judges of the Supreme Court and High Courts.
    Option B is correct: Under Article 61 of the Constitution of India, the President of India can be impeached only for the violation of the Constitution and that the charges against the President can be initiated by either of the House of the Parliament. Hence, Option D is also correct.
    Option C is incorrect: The Constitution of India does not explicitly define the ‘violation of the Constitution’ for the purpose of impeachment of the President.

    Additional Information:
    ● The notice for the charges against the President must be signed by at least a quarter of the members of the House.
    ● The resolution to impeach the President must be passed by a special majority (two-thirds) in the originating House.
    ● The other House acts as the investigating House. A select committee investigates the charges labelled against the President.
    ● The President of India has the right to defend himself against the charges.

window.wpAdvQuizInitList = window.wpAdvQuizInitList || []; window.wpAdvQuizInitList.push({ id: '#wpAdvQuiz_1', init: { quizId: 1, mode: 1, globalPoints: 10, timelimit: 0, resultsGrade: [0], bo: 0, qpp: 0, catPoints: [10], formPos: 0, lbn: "Finish quiz", json: {"1":{"type":"single","id":1,"catId":0,"points":2,"correct":[0,0,0,1]},"2":{"type":"single","id":2,"catId":0,"points":2,"correct":[0,1,0,0]},"3":{"type":"single","id":3,"catId":0,"points":2,"correct":[1,0,0,0]},"4":{"type":"single","id":4,"catId":0,"points":2,"correct":[1,0,0,0]},"5":{"type":"single","id":5,"catId":0,"points":2,"correct":[0,0,1,0]}} } });




Day-528 | Daily MCQs | UPSC Prelims | ECONOMY

Day-528

Time limit: 0

Quiz-summary

0 of 5 questions completed

Questions:

  1. 1
  2. 2
  3. 3
  4. 4
  5. 5

Information

To attempt the Quiz, simply click on START Button.

You have already completed the quiz before. Hence you can not start it again.

Quiz is loading...

You must sign in or sign up to start the quiz.

You have to finish following quiz, to start this quiz:

Results

0 of 5 questions answered correctly

Your time:

Time has elapsed

You have reached 0 of 0 points, (0)

Average score
 
 
Your score
 
 

Categories

  1. Not categorized 0%
Your result has been entered into leaderboard
Loading
captcha
  1. 1
  2. 2
  3. 3
  4. 4
  5. 5
  1. Answered
  2. Review
  1. Question 1 of 5
    1. Question
    2 points

    1. Consider the following statements regarding Off-budget borrowings:
    1. These borrowings are not counted while calculating the fiscal deficit.
    2. In these borrowings, the government takes loans directly.
    3. The budgetary resources cannot be used for repayment of these borrowings.
    How many of the above statements are correct?

    Correct

    Answer: A
    Explanation:
    ● Statement 1 is correct: Off-budget borrowings refer to the borrowings undertaken by the public sector for which the principal and interest are serviced out of government budgets. These borrowings are kept out of the government books and are not counted while calculating the fiscal deficit.
    Or
    ● Off-budget borrowing or extra-budget borrowing is a way for the government to finance its expenditures while keeping the debt off the books. It means that the government asks another public institution to borrow on its behalf so that the collected loan is not counted in the fiscal deficit. This intends to help the government keep its fiscal deficit within acceptable limits, but on the other hand, it also reduces the transparency and accountability of its finances.
    ● Statement 2 is incorrect: Off-budget borrowings (OBBs) are taken not by the government directly but by another public institution in its direction. For example, loans by FCI for paying food subsidy bills (this practice is discontinued from FY 2020-21).
    ● Statement 3 is incorrect: Government uses the budgetary resources for the repayment of Off-budget borrowings.
    ● Reasons for resorting to OBB:
    o Bypass Fiscal Deficit targets under the FRBM Act, 2003,
    o Avoid borrowing limits under Article 293 (3) of the Constitution.
    o Article mandates States to take consent of the Centre before raising any loan if they have any outstanding loan to the Centre or loans where the Centre is the guarantor.

    Incorrect

    Answer: A
    Explanation:
    ● Statement 1 is correct: Off-budget borrowings refer to the borrowings undertaken by the public sector for which the principal and interest are serviced out of government budgets. These borrowings are kept out of the government books and are not counted while calculating the fiscal deficit.
    Or
    ● Off-budget borrowing or extra-budget borrowing is a way for the government to finance its expenditures while keeping the debt off the books. It means that the government asks another public institution to borrow on its behalf so that the collected loan is not counted in the fiscal deficit. This intends to help the government keep its fiscal deficit within acceptable limits, but on the other hand, it also reduces the transparency and accountability of its finances.
    ● Statement 2 is incorrect: Off-budget borrowings (OBBs) are taken not by the government directly but by another public institution in its direction. For example, loans by FCI for paying food subsidy bills (this practice is discontinued from FY 2020-21).
    ● Statement 3 is incorrect: Government uses the budgetary resources for the repayment of Off-budget borrowings.
    ● Reasons for resorting to OBB:
    o Bypass Fiscal Deficit targets under the FRBM Act, 2003,
    o Avoid borrowing limits under Article 293 (3) of the Constitution.
    o Article mandates States to take consent of the Centre before raising any loan if they have any outstanding loan to the Centre or loans where the Centre is the guarantor.

  2. Question 2 of 5
    2. Question
    2 points

    2. Consider the following statements regarding India Post Payments Bank (IPPB):
    1. It is set up under the Ministry of Finance.
    2. It is fully owned by the Government of India.
    3. It is a scheduled payment bank.
    4. It does not offer credit to private entities.
    How many of the above statements are correct?

    Correct

    Answer: C
    Explanation:
    ● Statement 1 is incorrect: In 2018, IPPB was established under the Department of Posts, Ministry of Communication.
    ● Statement 2 is correct: Its 100% equity is owned by the Government of India.
    o Vision: Build the most accessible, affordable and trusted bank for the common man in India.
    o Mandate: Spearheading financial inclusion by removing barriers and reducing costs for accessing banking services.
    ● IPPB is registered under the Banking Regulation Act, 1949 duly licensed by RBI to carry on the business of payments banks in India.
    o Statement 3 is correct: It is a scheduled payment bank offering a range of products like savings and current accounts, remittances and money transfer, Aadhar Enabled Payment System etc.
    o It offers three accounts: Safal (regular account); Sugam, (Basic Savings Bank Deposit Account (BSBDA)); and Saral (BSBDA-Small).
    o Statement 4 is correct: It can not offer loans or issue credit cards.
    ● Achievements of IPPB: More than 6 crore accounts have been opened across the country including 96 lakhs in aspirational districts.
    Functions of India Post Payments Bank (IPPB):
    ● DEPOSITS
    o Savings Account
    o Current Account
    ● MONEY TRANSFER
    o Simple & Secure
    o Instant
    o 24×7
    ● THIRD PARTY PRODUCTS
    o Loans referral services
    o Insurance
    o Investments
    o Post Office Savings schemes
    ● ENTERPRISE AND MERCHANT PAYMENTS
    o Postal products
    o Digital Payment of e-commerce delivery (CoD)
    o Small merchants/Kirana stores/unorganized retail
    o Offline payments
    o Cash Management Services
    Payment Banks:
    ● In August 2015, RBI granted licenses to 11 applicants for Payment Banks.
    ● RBI has put a cap of Rs. 2 lakhs on deposits that payment banks can receive from individual customers. This restriction will allow only those companies to seek payment bank licenses who are really interested in targeting the poor. Hence, the main target for payment banks will be migrant labourers, self-employed, low-income households etc. as they will offer low-cost savings accounts and remittance services so that those who now transact only in cash can take their first step into the formal banking system (payment banks will not be allowed to lend and issue credit cards. Payment banks will accept only demand deposits i.e., only savings account and current account facility will be available).
    ● The payment banks will be cashing in on mobile technology and applications to cater to the various services they will be offering and with the use of technology they can be cost efficient.
    ● The Payment banks will be acting as add-on to the already established banks, rather than their competitors.

    Incorrect

    Answer: C
    Explanation:
    ● Statement 1 is incorrect: In 2018, IPPB was established under the Department of Posts, Ministry of Communication.
    ● Statement 2 is correct: Its 100% equity is owned by the Government of India.
    o Vision: Build the most accessible, affordable and trusted bank for the common man in India.
    o Mandate: Spearheading financial inclusion by removing barriers and reducing costs for accessing banking services.
    ● IPPB is registered under the Banking Regulation Act, 1949 duly licensed by RBI to carry on the business of payments banks in India.
    o Statement 3 is correct: It is a scheduled payment bank offering a range of products like savings and current accounts, remittances and money transfer, Aadhar Enabled Payment System etc.
    o It offers three accounts: Safal (regular account); Sugam, (Basic Savings Bank Deposit Account (BSBDA)); and Saral (BSBDA-Small).
    o Statement 4 is correct: It can not offer loans or issue credit cards.
    ● Achievements of IPPB: More than 6 crore accounts have been opened across the country including 96 lakhs in aspirational districts.
    Functions of India Post Payments Bank (IPPB):
    ● DEPOSITS
    o Savings Account
    o Current Account
    ● MONEY TRANSFER
    o Simple & Secure
    o Instant
    o 24×7
    ● THIRD PARTY PRODUCTS
    o Loans referral services
    o Insurance
    o Investments
    o Post Office Savings schemes
    ● ENTERPRISE AND MERCHANT PAYMENTS
    o Postal products
    o Digital Payment of e-commerce delivery (CoD)
    o Small merchants/Kirana stores/unorganized retail
    o Offline payments
    o Cash Management Services
    Payment Banks:
    ● In August 2015, RBI granted licenses to 11 applicants for Payment Banks.
    ● RBI has put a cap of Rs. 2 lakhs on deposits that payment banks can receive from individual customers. This restriction will allow only those companies to seek payment bank licenses who are really interested in targeting the poor. Hence, the main target for payment banks will be migrant labourers, self-employed, low-income households etc. as they will offer low-cost savings accounts and remittance services so that those who now transact only in cash can take their first step into the formal banking system (payment banks will not be allowed to lend and issue credit cards. Payment banks will accept only demand deposits i.e., only savings account and current account facility will be available).
    ● The payment banks will be cashing in on mobile technology and applications to cater to the various services they will be offering and with the use of technology they can be cost efficient.
    ● The Payment banks will be acting as add-on to the already established banks, rather than their competitors.

  3. Question 3 of 5
    3. Question
    2 points

    3. Which of the following statements correctly describes the term “Inflation Tax”?

    Correct

    Answer: D
    Explanation:
    ● As a result of inflation (the opposite of deflation), the cost of goods and services rises, but the value of our money decreases. And that inflation is accompanied by inflation tax.
    ● Inflation tax is not a tax paid to the government. Instead “inflation tax” refers to the penalty for holding cash at a time of high inflation. When the government prints more money or reduces interest rates, it floods the market with cash, which raises inflation in the long run. If an investor is holding securities, real estate or other assets, the effect of inflation may be negligible. But, if a person is holding cash, the purchasing power of this cash reduces during a higher rate of inflation.

    Incorrect

    Answer: D
    Explanation:
    ● As a result of inflation (the opposite of deflation), the cost of goods and services rises, but the value of our money decreases. And that inflation is accompanied by inflation tax.
    ● Inflation tax is not a tax paid to the government. Instead “inflation tax” refers to the penalty for holding cash at a time of high inflation. When the government prints more money or reduces interest rates, it floods the market with cash, which raises inflation in the long run. If an investor is holding securities, real estate or other assets, the effect of inflation may be negligible. But, if a person is holding cash, the purchasing power of this cash reduces during a higher rate of inflation.

  4. Question 4 of 5
    4. Question
    2 points

    4. Consider the following statements related to the Standing Deposit Facility (SDF):
    1. This tool has been introduced through an amendment to the Banking Regulation Act, 1949.
    2. It enables the RBI to absorb liquidity from the economy.
    3. Banks can deposit any amount for the long term under SDF.
    How many of the above statements are correct?

    Correct

    Answer: A
    Explanation:
    Standing Deposit Facility (SDF):
    ● Statement 1 is incorrect: Introduced in April 2022 by an amendment in RBI Act 1934.
    ● Statement 2 is correct: A Standing Deposit Facility is an overnight deposit facility that allows banks to park excess liquidity (money) and earn interest. In a liquidity-surplus banking system, the RBI is focused on absorbing excess liquidity from banks without collateral.
    ● The following are its important features:
    o Statement 3 is incorrect: Banks can deposit any amount overnight with RBI at repo rate – 0.25% (it may change).
    o As there is no binding collateral constraint on RBI under SDF, it will strengthen the operating framework of monetary policy.
    o The SDF is also a financial stability tool in addition to its role in liquidity management.
    o SDF will be at the discretion of banks and will be available on all days of the week, throughout the year unlike repo and reverse repo which are available at the discretion of RBI.

    Incorrect

    Answer: A
    Explanation:
    Standing Deposit Facility (SDF):
    ● Statement 1 is incorrect: Introduced in April 2022 by an amendment in RBI Act 1934.
    ● Statement 2 is correct: A Standing Deposit Facility is an overnight deposit facility that allows banks to park excess liquidity (money) and earn interest. In a liquidity-surplus banking system, the RBI is focused on absorbing excess liquidity from banks without collateral.
    ● The following are its important features:
    o Statement 3 is incorrect: Banks can deposit any amount overnight with RBI at repo rate – 0.25% (it may change).
    o As there is no binding collateral constraint on RBI under SDF, it will strengthen the operating framework of monetary policy.
    o The SDF is also a financial stability tool in addition to its role in liquidity management.
    o SDF will be at the discretion of banks and will be available on all days of the week, throughout the year unlike repo and reverse repo which are available at the discretion of RBI.

  5. Question 5 of 5
    5. Question
    2 points

    5. Consider the following statements:
    Statement I: Inflation Indexed Bonds are tradable in the secondary market.
    Statement II: Inflation Indexed Bonds are the Government securities.
    Which one of the following is correct in respect of the above statements?

    Correct

    Answer: B
    Explanation:
    ● Inflation Indexed Bonds (IIBs) were issued in the name of Capital Indexed Bonds (CIBs) during 1997. The CIBs issued in 1997 provide inflation protection only to principal and not to interest payment. IIBs will provide inflation protection to both principal and interest payments.
    ● IIBs would be Government securities (G-Sec) and the different classes of investors eligible to invest in G-Secs would also be eligible to invest in IIBs.
    ● FIIs would be eligible to invest in the forthcoming IIBs but subject to the overall cap for their investment in G-Secs (currently USD 25 billion).
    ● Statement I and II are correct but its explanation is incorrect: G-Sec can be tradable in the secondary market like other G-Secs. Because IIBs are Government securities.
    ● Investors will be able to trade them in NDS-OM, NDS-OM (web-based), OTC market, and stock exchanges.
    ● As of now investors will not be able to participate in the primary auction of IIBs through a web-based platform.

    Incorrect

    Answer: B
    Explanation:
    ● Inflation Indexed Bonds (IIBs) were issued in the name of Capital Indexed Bonds (CIBs) during 1997. The CIBs issued in 1997 provide inflation protection only to principal and not to interest payment. IIBs will provide inflation protection to both principal and interest payments.
    ● IIBs would be Government securities (G-Sec) and the different classes of investors eligible to invest in G-Secs would also be eligible to invest in IIBs.
    ● FIIs would be eligible to invest in the forthcoming IIBs but subject to the overall cap for their investment in G-Secs (currently USD 25 billion).
    ● Statement I and II are correct but its explanation is incorrect: G-Sec can be tradable in the secondary market like other G-Secs. Because IIBs are Government securities.
    ● Investors will be able to trade them in NDS-OM, NDS-OM (web-based), OTC market, and stock exchanges.
    ● As of now investors will not be able to participate in the primary auction of IIBs through a web-based platform.

window.wpAdvQuizInitList = window.wpAdvQuizInitList || []; window.wpAdvQuizInitList.push({ id: '#wpAdvQuiz_2', init: { quizId: 2, mode: 1, globalPoints: 10, timelimit: 0, resultsGrade: [0], bo: 0, qpp: 0, catPoints: [10], formPos: 0, lbn: "Finish quiz", json: {"6":{"type":"single","id":6,"catId":0,"points":2,"correct":[1,0,0,0]},"7":{"type":"single","id":7,"catId":0,"points":2,"correct":[0,0,1,0]},"8":{"type":"single","id":8,"catId":0,"points":2,"correct":[0,0,0,1]},"9":{"type":"single","id":9,"catId":0,"points":2,"correct":[1,0,0,0]},"10":{"type":"single","id":10,"catId":0,"points":2,"correct":[0,1,0,0]}} } });




Day-527 | Daily MCQs | UPSC Prelims | GEOGRAPHY

Day-527

Time limit: 0

Quiz-summary

0 of 5 questions completed

Questions:

  1. 1
  2. 2
  3. 3
  4. 4
  5. 5

Information

To attempt the Quiz, simply click on START Button.

You have already completed the quiz before. Hence you can not start it again.

Quiz is loading...

You must sign in or sign up to start the quiz.

You have to finish following quiz, to start this quiz:

Results

0 of 5 questions answered correctly

Your time:

Time has elapsed

You have reached 0 of 0 points, (0)

Average score
 
 
Your score
 
 

Categories

  1. Not categorized 0%
Your result has been entered into leaderboard
Loading
captcha
  1. 1
  2. 2
  3. 3
  4. 4
  5. 5
  1. Answered
  2. Review
  1. Question 1 of 5
    1. Question
    2 points

    1. Consider the following pairs:
    Rivers – Sea/Ocean they drain into
    1. Amazon – Atlantic Ocean
    2. Nile – Red Sea
    3. Danube – Black Sea
    4. Orange – Indian Ocean
    How many of the above pairs are correctly matched?

    Correct

    Answer: B
    Explanation:
    Pair 1 is matched correctly: Amazon River falls into the Atlantic Ocean.
    Amazon River: It is the biggest river of South America and the largest drainage system in the world in terms of the volume of its flow and the area of its basin.
    The total length of the river is at least 4,000 miles (6,400 km), which makes it slightly shorter than the Nile River.

    Pair 2 is matched incorrectly: Nile River falls into Mediterranean Sea.
    Nile River
    ● Nile River is the longest river in the world, called the father of African rivers.
    ● It rises south of the Equator and flows northward through northeastern Africa to drain into the Mediterranean Sea.
    ● It has a length of about 4,132 miles (6,650 kilometres) and drains an area estimated at 1,293,000 square miles (3,349,000 square kilometres).
    ● Its basin includes parts of Tanzania, Burundi, Rwanda, the Democratic Republic of the Congo, Kenya, Uganda, South Sudan, Ethiopia, Sudan, and the cultivated part of Egypt.
    ● Its most distant source is the Kagera River in Burundi.

    Pair 3 is matched correctly: Danube River falls into Black Sea.
    Danube River:
    ● It is the second longest in Europe after the Volga.
    ● It rises in the Black Forest mountains of western Germany and flows for some 1,770 miles (2,850 km) to its mouth on the Black Sea.
    ● Along its course it passes through 10 countries:
    ● Germany, Austria, Slovakia, Hungary, Croatia, Serbia, Bulgaria, Romania, Moldova, and Ukraine.

    Pair 4 is matched incorrectly: Orange River falls into the Atlantic Ocean.
    Orange River:
    ● Orange River, river in southern Africa, one of the longest rivers on the continent and one of the longest south of the Tropic of Capricorn.
    ● After rising in the Lesotho Highlands, less than 125 miles (200 kilometres) from the Indian Ocean, the river flows to the Atlantic Ocean in a generally westerly direction for some 1,300 miles.
    ● The Orange traverses the veld region of South Africa, after which it defines the southern limit of the Kalahari and bisects the southern Namib before draining into the Atlantic at Alexander Bay.
    ● Along its course the river forms the eastern boundary of the South African province of Free State, as well as the boundary between Namibia and South Africa.

    Incorrect

    Answer: B
    Explanation:
    Pair 1 is matched correctly: Amazon River falls into the Atlantic Ocean.
    Amazon River: It is the biggest river of South America and the largest drainage system in the world in terms of the volume of its flow and the area of its basin.
    The total length of the river is at least 4,000 miles (6,400 km), which makes it slightly shorter than the Nile River.

    Pair 2 is matched incorrectly: Nile River falls into Mediterranean Sea.
    Nile River
    ● Nile River is the longest river in the world, called the father of African rivers.
    ● It rises south of the Equator and flows northward through northeastern Africa to drain into the Mediterranean Sea.
    ● It has a length of about 4,132 miles (6,650 kilometres) and drains an area estimated at 1,293,000 square miles (3,349,000 square kilometres).
    ● Its basin includes parts of Tanzania, Burundi, Rwanda, the Democratic Republic of the Congo, Kenya, Uganda, South Sudan, Ethiopia, Sudan, and the cultivated part of Egypt.
    ● Its most distant source is the Kagera River in Burundi.

    Pair 3 is matched correctly: Danube River falls into Black Sea.
    Danube River:
    ● It is the second longest in Europe after the Volga.
    ● It rises in the Black Forest mountains of western Germany and flows for some 1,770 miles (2,850 km) to its mouth on the Black Sea.
    ● Along its course it passes through 10 countries:
    ● Germany, Austria, Slovakia, Hungary, Croatia, Serbia, Bulgaria, Romania, Moldova, and Ukraine.

    Pair 4 is matched incorrectly: Orange River falls into the Atlantic Ocean.
    Orange River:
    ● Orange River, river in southern Africa, one of the longest rivers on the continent and one of the longest south of the Tropic of Capricorn.
    ● After rising in the Lesotho Highlands, less than 125 miles (200 kilometres) from the Indian Ocean, the river flows to the Atlantic Ocean in a generally westerly direction for some 1,300 miles.
    ● The Orange traverses the veld region of South Africa, after which it defines the southern limit of the Kalahari and bisects the southern Namib before draining into the Atlantic at Alexander Bay.
    ● Along its course the river forms the eastern boundary of the South African province of Free State, as well as the boundary between Namibia and South Africa.

  2. Question 2 of 5
    2. Question
    2 points

    2. Consider the following statements:
    Statement-I: Generally, tropical cyclones are not observed in the South Atlantic Ocean.
    Statement-II: Sea surface water temperatures in the tropical southern Atlantic Ocean are cooler than those in the tropical north Atlantic Ocean.
    Which one of the following is correct in respect of the above statements?

    Correct

    Answer: A
    Explanation:
    Tropical cyclones are violent storms that originate over oceans in tropical areas and move over to the coastal areas bringing about large scale destruction caused by violent winds, very heavy rainfall and storm surges.
    The conditions favourable for the formation and intensification of tropical storms are: (i) Large sea surface with temperature higher than 27° C; (ii) Presence of the Coriolis force; (iii) Small variations in the vertical wind speed; (iv) A pre-existing weak low-pressure area or low-level-cyclonic circulation; (v) Upper divergence above the sea level system.
    Tropical cyclones are unusual in South Atlantic Ocean because:
    ● Strong wind shear in the Troposphere, which disrupts the formation of these cyclones, as well as a lack of weather disturbances favourable for development in the South Atlantic Ocean (such as the tropical waves common in the southern North Atlantic) make any strong tropical system extremely rare.
    ● The Inter Tropical Convergence Zone (ITCZ), another potential breeding ground, drops one to two degrees south of the equator, not far enough from the equator for the spin of the Coriolis force to aid development.
    ● Water temperatures in the tropics of the southern Atlantic are cooler than those in the tropical north Atlantic.

    Incorrect

    Answer: A
    Explanation:
    Tropical cyclones are violent storms that originate over oceans in tropical areas and move over to the coastal areas bringing about large scale destruction caused by violent winds, very heavy rainfall and storm surges.
    The conditions favourable for the formation and intensification of tropical storms are: (i) Large sea surface with temperature higher than 27° C; (ii) Presence of the Coriolis force; (iii) Small variations in the vertical wind speed; (iv) A pre-existing weak low-pressure area or low-level-cyclonic circulation; (v) Upper divergence above the sea level system.
    Tropical cyclones are unusual in South Atlantic Ocean because:
    ● Strong wind shear in the Troposphere, which disrupts the formation of these cyclones, as well as a lack of weather disturbances favourable for development in the South Atlantic Ocean (such as the tropical waves common in the southern North Atlantic) make any strong tropical system extremely rare.
    ● The Inter Tropical Convergence Zone (ITCZ), another potential breeding ground, drops one to two degrees south of the equator, not far enough from the equator for the spin of the Coriolis force to aid development.
    ● Water temperatures in the tropics of the southern Atlantic are cooler than those in the tropical north Atlantic.

  3. Question 3 of 5
    3. Question
    2 points

    3. With respect to the “Chenab” river, consider the following statements:
    1. It originates near the Bara Lacha Pass in the Zaskar Range.
    2. It is the second largest tributary of the Indus river.
    3. The river is formed by the confluence of two rivers, the Chandra and the Bhaga.
    How many of the above statements are incorrect?

    Correct

    Answer: A
    Explanation:
    Statement 1 is correct: Chenab is a major river that flows in India and Pakistan, and is one of the 5 major rivers of the Punjab region.
    It originates from near the Bara Lacha Pass in the Lahul-Spiti part of the Zaskar Range.
    Statement 2 is incorrect: It is the largest tributary of river Indus.
    Statement 3 is correct:
    The river is formed by the confluence of two rivers, the Chandra and the Bhaga, at Tandi, 8 km southwest of Keylong, in the Lahaul and Spiti district in the Indian state of Himachal Pradesh.

    Incorrect

    Answer: A
    Explanation:
    Statement 1 is correct: Chenab is a major river that flows in India and Pakistan, and is one of the 5 major rivers of the Punjab region.
    It originates from near the Bara Lacha Pass in the Lahul-Spiti part of the Zaskar Range.
    Statement 2 is incorrect: It is the largest tributary of river Indus.
    Statement 3 is correct:
    The river is formed by the confluence of two rivers, the Chandra and the Bhaga, at Tandi, 8 km southwest of Keylong, in the Lahaul and Spiti district in the Indian state of Himachal Pradesh.

  4. Question 4 of 5
    4. Question
    2 points

    4. Consider the following statements:
    Statement-I: Shallow focus earthquakes are observed along the Circum-Pacific zone.
    Statement-II: Pacific plate being the heavier plate subducts under surrounding continental plates.
    Which one of the following is correct in respect of the above statements?

    Correct

    Answer: D
    Explanation:
    ● Statement I is incorrect: Deep focus earthquakes are observed along the Circum-Pacific zone as there are converging plate movements/boundaries.
    ● Statement II is correct: Pacific plate being the heavier plate (as it is oceanic plate) subducts under surrounding continental plates such as North American plate, South American plate and Eurasian plate.
    Note: Shallow focus earthquakes are observed along divergent plate boundaries. E.g. Mid-oceanic regions.

    Incorrect

    Answer: D
    Explanation:
    ● Statement I is incorrect: Deep focus earthquakes are observed along the Circum-Pacific zone as there are converging plate movements/boundaries.
    ● Statement II is correct: Pacific plate being the heavier plate (as it is oceanic plate) subducts under surrounding continental plates such as North American plate, South American plate and Eurasian plate.
    Note: Shallow focus earthquakes are observed along divergent plate boundaries. E.g. Mid-oceanic regions.

  5. Question 5 of 5
    5. Question
    2 points

    5. Consider the following statements:
    1. The Bass Strait connects the Caribbean Sea with the Gulf of Mexico.
    2. The Yucatan Channel separates the island state of Tasmania from the Australian mainland.
    3. The Bab al-Mandab Strait connects the Red Sea with the Gulf of Aden.
    How many of the above statements are correct?

    Correct

    Answer: A
    Explanation:
    Statement 1 is incorrect:
    The Bass Strait is a strait separating the island state of Tasmania from the Australian mainland. The strait provides the most direct waterway between the Great Australian Bight and the Tasman Sea and is also the only maritime route into the economically prominent Port Phillip Bay.

    Statement 2 is incorrect:
    The Yucatán Channel separates Cuba from the Yucatan Peninsula of Mexico and links the Caribbean Sea with the Gulf of Mexico.

    Statement 3 is correct:
    ● The Bab-el-Mandeb also known as the Gate of Grief is a strait between Yemen on the Arabian Peninsula, Djibouti, and Eritrea in the Horn of Africa.
    ● It connects the Red Sea to the Gulf of Aden.

    Incorrect

    Answer: A
    Explanation:
    Statement 1 is incorrect:
    The Bass Strait is a strait separating the island state of Tasmania from the Australian mainland. The strait provides the most direct waterway between the Great Australian Bight and the Tasman Sea and is also the only maritime route into the economically prominent Port Phillip Bay.

    Statement 2 is incorrect:
    The Yucatán Channel separates Cuba from the Yucatan Peninsula of Mexico and links the Caribbean Sea with the Gulf of Mexico.

    Statement 3 is correct:
    ● The Bab-el-Mandeb also known as the Gate of Grief is a strait between Yemen on the Arabian Peninsula, Djibouti, and Eritrea in the Horn of Africa.
    ● It connects the Red Sea to the Gulf of Aden.

window.wpAdvQuizInitList = window.wpAdvQuizInitList || []; window.wpAdvQuizInitList.push({ id: '#wpAdvQuiz_3', init: { quizId: 3, mode: 1, globalPoints: 10, timelimit: 0, resultsGrade: [0], bo: 0, qpp: 0, catPoints: [10], formPos: 0, lbn: "Finish quiz", json: {"11":{"type":"single","id":11,"catId":0,"points":2,"correct":[0,1,0,0]},"12":{"type":"single","id":12,"catId":0,"points":2,"correct":[1,0,0,0]},"13":{"type":"single","id":13,"catId":0,"points":2,"correct":[1,0,0,0]},"14":{"type":"single","id":14,"catId":0,"points":2,"correct":[0,0,0,1]},"15":{"type":"single","id":15,"catId":0,"points":2,"correct":[1,0,0,0]}} } });




Day-526 | Daily MCQs | UPSC Prelims | HISTORY OF ANCIENT INDIA

Day-526

Time limit: 0

Quiz-summary

0 of 5 questions completed

Questions:

  1. 1
  2. 2
  3. 3
  4. 4
  5. 5

Information

To attempt the Quiz, simply click on START Button.

You have already completed the quiz before. Hence you can not start it again.

Quiz is loading...

You must sign in or sign up to start the quiz.

You have to finish following quiz, to start this quiz:

Results

0 of 5 questions answered correctly

Your time:

Time has elapsed

You have reached 0 of 0 points, (0)

Average score
 
 
Your score
 
 

Categories

  1. Not categorized 0%
Your result has been entered into leaderboard
Loading
captcha
  1. 1
  2. 2
  3. 3
  4. 4
  5. 5
  1. Answered
  2. Review
  1. Question 1 of 5
    1. Question
    2 points

    1. With reference to the history of early medieval India, the term tirumandira-olai refers to:

    Correct

    Answer: C
    Explanation:
    The above given term is related to the administrative history of the Chola empire of Early Medieval India. It refers to high ranking functionaries associated with the court of the Chola kings. Other important functionaries included- adhikari, vayil ketpar, etc.
    Additional Information:
    ● The Chola kings followed a highly efficient system of administration. The entire Tanjore district, parts of Trichy, Pudukottai and South Arcot districts formed the part of the Chola Mandalam. The Cholas had three major administrative divisions called the Central Government, Provincial Government and Local Government. Tanjore was the capital of the Cholas.
    ● The Chola Empire was divided into nine provinces. They were also called mandalams. Each mandalam was divided into a number of Kottams or Valanadus. Each kottam was sub divided into nadu. Each nadu was further divided into (Urs) villages which form part of the last unit of the administration. Uttaramerur inscriptions speak about the administration of the Cholas.

    Incorrect

    Answer: C
    Explanation:
    The above given term is related to the administrative history of the Chola empire of Early Medieval India. It refers to high ranking functionaries associated with the court of the Chola kings. Other important functionaries included- adhikari, vayil ketpar, etc.
    Additional Information:
    ● The Chola kings followed a highly efficient system of administration. The entire Tanjore district, parts of Trichy, Pudukottai and South Arcot districts formed the part of the Chola Mandalam. The Cholas had three major administrative divisions called the Central Government, Provincial Government and Local Government. Tanjore was the capital of the Cholas.
    ● The Chola Empire was divided into nine provinces. They were also called mandalams. Each mandalam was divided into a number of Kottams or Valanadus. Each kottam was sub divided into nadu. Each nadu was further divided into (Urs) villages which form part of the last unit of the administration. Uttaramerur inscriptions speak about the administration of the Cholas.

  2. Question 2 of 5
    2. Question
    2 points

    2. Consider the following statements about the brahmadeyas villages:
    1. It introduced the brahmanas as the intermediary for the purpose of revenue collection.
    2. All brahmadeyas villages were exempted from the payment of taxes to the states.
    3. It also meant that the peasants were freed from the obligation of paying the taxes.
    How many of the statements given above are correct?

    Correct

    Answer: A
    Explanation:
    Brahmadeyas were a category of villages which were given for brahmana settlement, which were tax- free i.e., it was not that revenue was not collected from the peasants, but the State has given away its right to revenue collection to the brahmanas.
    ● Statement 1 is incorrect: Earlier the officials were the intermediaries who were responsible for the collection of revenue, but now brahmadeyas villages had brahmanas as revenue collectors. Also, as the state has given away its right to collect taxes, now there is only the brahmanas and peasants. The brahmanas are not intermediaries, but the sole beneficiaries of taxes.
    ● Statement 2 is correct: Brahmadeyas or agrahara villages were granted to brahmanas which were tax-free villages.
    ● Statement 3 is incorrect: Under the brahmadeyas villages peasants were still obliged to pay taxes, not to the state, but to the donees.

    Incorrect

    Answer: A
    Explanation:
    Brahmadeyas were a category of villages which were given for brahmana settlement, which were tax- free i.e., it was not that revenue was not collected from the peasants, but the State has given away its right to revenue collection to the brahmanas.
    ● Statement 1 is incorrect: Earlier the officials were the intermediaries who were responsible for the collection of revenue, but now brahmadeyas villages had brahmanas as revenue collectors. Also, as the state has given away its right to collect taxes, now there is only the brahmanas and peasants. The brahmanas are not intermediaries, but the sole beneficiaries of taxes.
    ● Statement 2 is correct: Brahmadeyas or agrahara villages were granted to brahmanas which were tax-free villages.
    ● Statement 3 is incorrect: Under the brahmadeyas villages peasants were still obliged to pay taxes, not to the state, but to the donees.

  3. Question 3 of 5
    3. Question
    2 points

    3. Consider the following pairs:
    Authors – Texts
    1. Akalanka – Anekantajayapataka
    2. Haribhadra – Aptamimam-salamkrita
    3. Vidyananda – Tattvartharajavarttika
    How many pairs given above are correctly matched?

    Correct

    Answer: D
    Explanation:
    All the given authors are Jaina philosophers and texts.
    The correctly matched pairs are:
    ● Akalanka- Tattvartharajavarttika
    ● Haribhadra- Anekantajayapataka
    ● Vidyananda- Aptamimam-salamkrita
    Additional Information:
    ● The sacred books of the Jainas are collectively known as the Siddhanta or Agama. The language of the earliest texts is an eastern dialect of Prakrit known as Ardha-Magadhi.
    ● The Jaina monastic order came to be divided into the Shvetambara and Digambara schools, perhaps in about the 3rd century ce. The Shvetambara canon includes the 12 Angas, 12 Uvamgas (Upangas), 10 Painnas (Prakirnas), 6 Cheya Suttas (Cheda Sutras), 4 Mula Suttas (Mula Sutras), and a number of individual texts such as the Nandi Sutta (Nandi Sutra) and Anugodara (Anuyogadvara).
    ● According to Shvetambara tradition, the Angas were compiled at a council held at Pataliputra. The compilation of the entire canon is supposed to have taken place in the 5th or 6th century at a council held in Valabhi in Gujarat, presided over by Devarddhi Kshamashramana.

    Incorrect

    Answer: D
    Explanation:
    All the given authors are Jaina philosophers and texts.
    The correctly matched pairs are:
    ● Akalanka- Tattvartharajavarttika
    ● Haribhadra- Anekantajayapataka
    ● Vidyananda- Aptamimam-salamkrita
    Additional Information:
    ● The sacred books of the Jainas are collectively known as the Siddhanta or Agama. The language of the earliest texts is an eastern dialect of Prakrit known as Ardha-Magadhi.
    ● The Jaina monastic order came to be divided into the Shvetambara and Digambara schools, perhaps in about the 3rd century ce. The Shvetambara canon includes the 12 Angas, 12 Uvamgas (Upangas), 10 Painnas (Prakirnas), 6 Cheya Suttas (Cheda Sutras), 4 Mula Suttas (Mula Sutras), and a number of individual texts such as the Nandi Sutta (Nandi Sutra) and Anugodara (Anuyogadvara).
    ● According to Shvetambara tradition, the Angas were compiled at a council held at Pataliputra. The compilation of the entire canon is supposed to have taken place in the 5th or 6th century at a council held in Valabhi in Gujarat, presided over by Devarddhi Kshamashramana.

  4. Question 4 of 5
    4. Question
    2 points

    4. Consider the following ports:
    1. Barbaricum
    2. Barygaza
    3. Arikamedu
    4. Muziris
    How many of the above-mentioned ancient ports are located on the western coast?

    Correct

    Answer: C
    Explanation:
    During the post- Mauryan period, the links by road and river between the main cities and trading towns were an essential element in the general infrastructure facilitating the volume of trade as a whole.
    All the major routes led to the five main international ports of that period, namely Barbaricum in the Indus delta, Barygaza on the Gujarat coast, Muziris on the Kerala coast, Arikamedu on the Coromandel coast and Tamralipti in the Ganges delta. These ports handled the bulk of the sea trade of India with Arabia, the Levant, the Roman Empire and South East Asia.

    Land and sea routes of ancient Indian international trade

    Incorrect

    Answer: C
    Explanation:
    During the post- Mauryan period, the links by road and river between the main cities and trading towns were an essential element in the general infrastructure facilitating the volume of trade as a whole.
    All the major routes led to the five main international ports of that period, namely Barbaricum in the Indus delta, Barygaza on the Gujarat coast, Muziris on the Kerala coast, Arikamedu on the Coromandel coast and Tamralipti in the Ganges delta. These ports handled the bulk of the sea trade of India with Arabia, the Levant, the Roman Empire and South East Asia.

    Land and sea routes of ancient Indian international trade

  5. Question 5 of 5
    5. Question
    2 points

    5. Which of the following statements about the ‘Satapatra series’ is/are correct?
    1. It was authored by Vishnukrishna Chiplunkar, one of the reformers of western India.
    2. In this series, he tried to project Shivaji as a Shudra king.
    3. Along with advocating social reforms and indigenous enterprise, it welcomed British rule.
    Select the correct answer using the code given below:

    Correct

    Answer: C
    Explanation:
    Statement 1 is incorrect: The Satapatra series (1848- 50) was written by ‘Lokahitavadi’ Gopal Hari Deshmukh.
    Statement 2 is incorrect: Projecting Shivaji as a Shudra king, was done by Jotiba Phule in one of the ballad composed in 1869.
    Statement 3 is correct: Satapatra series (1848- 50) called for social reforms, advocated indigenous enterprise, but broadly welcomed British rule.
    Additional Information:
    ● Gopal Hari Deshmukh wrote a number of open letters in the Marathi weekly newspaper Prabhakar, which was published from Raigad (Maharashtra) under the pen name Lokhitawadi during 1848-49.
    ● The letters of Lokhitawadi reflect his progressive and far-sighted outlook. He was a pioneer political thinker, who through his letters demanded Indian representation in the British parliament as a means to reforming the flawed and oppressive British regime. He even went so far as to say that if the British did not abstain from imposing their will on the people of India, they would be compelled to leave altogether.
    ● Deshmukh was also one of the pioneers of the Swadeshi philosophy and understood the need to revitalize Indian industries to fight against poverty and unemployment. Deshmukh’s political and economic ideas circulated throughout Maharashtra through his letters and played a critical role in provoking political thinking and discourse in the region.
    ● Through Prabhakar, the letters of Gopal Hari Deshmukh caught the attention of the youth of Maharashtra and his ideas laid the foundation for the future struggle for Indian freedom.

    Incorrect

    Answer: C
    Explanation:
    Statement 1 is incorrect: The Satapatra series (1848- 50) was written by ‘Lokahitavadi’ Gopal Hari Deshmukh.
    Statement 2 is incorrect: Projecting Shivaji as a Shudra king, was done by Jotiba Phule in one of the ballad composed in 1869.
    Statement 3 is correct: Satapatra series (1848- 50) called for social reforms, advocated indigenous enterprise, but broadly welcomed British rule.
    Additional Information:
    ● Gopal Hari Deshmukh wrote a number of open letters in the Marathi weekly newspaper Prabhakar, which was published from Raigad (Maharashtra) under the pen name Lokhitawadi during 1848-49.
    ● The letters of Lokhitawadi reflect his progressive and far-sighted outlook. He was a pioneer political thinker, who through his letters demanded Indian representation in the British parliament as a means to reforming the flawed and oppressive British regime. He even went so far as to say that if the British did not abstain from imposing their will on the people of India, they would be compelled to leave altogether.
    ● Deshmukh was also one of the pioneers of the Swadeshi philosophy and understood the need to revitalize Indian industries to fight against poverty and unemployment. Deshmukh’s political and economic ideas circulated throughout Maharashtra through his letters and played a critical role in provoking political thinking and discourse in the region.
    ● Through Prabhakar, the letters of Gopal Hari Deshmukh caught the attention of the youth of Maharashtra and his ideas laid the foundation for the future struggle for Indian freedom.

window.wpAdvQuizInitList = window.wpAdvQuizInitList || []; window.wpAdvQuizInitList.push({ id: '#wpAdvQuiz_4', init: { quizId: 4, mode: 1, globalPoints: 10, timelimit: 0, resultsGrade: [0], bo: 0, qpp: 0, catPoints: [10], formPos: 0, lbn: "Finish quiz", json: {"16":{"type":"single","id":16,"catId":0,"points":2,"correct":[0,0,1,0]},"17":{"type":"single","id":17,"catId":0,"points":2,"correct":[1,0,0,0]},"18":{"type":"single","id":18,"catId":0,"points":2,"correct":[0,0,0,1]},"19":{"type":"single","id":19,"catId":0,"points":2,"correct":[0,0,1,0]},"20":{"type":"single","id":20,"catId":0,"points":2,"correct":[0,0,1,0]}} } });




Day-525 | Daily MCQs | UPSC Prelims | CURRENT DEVELOPMENTS

Day-525

Time limit: 0

Quiz-summary

0 of 5 questions completed

Questions:

  1. 1
  2. 2
  3. 3
  4. 4
  5. 5

Information

To attempt the Quiz, simply click on START Button.

You have already completed the quiz before. Hence you can not start it again.

Quiz is loading...

You must sign in or sign up to start the quiz.

You have to finish following quiz, to start this quiz:

Results

0 of 5 questions answered correctly

Your time:

Time has elapsed

You have reached 0 of 0 points, (0)

Average score
 
 
Your score
 
 

Categories

  1. Not categorized 0%
Your result has been entered into leaderboard
Loading
captcha
  1. 1
  2. 2
  3. 3
  4. 4
  5. 5
  1. Answered
  2. Review
  1. Question 1 of 5
    1. Question
    2 points

    1. Recently, there was a growing awareness in our country about the importance of “Kappaphycus alvarezii” because it is found to be a sustainable source of-

    Correct

    Answer: A
    Explanation:
    ● The importance of Kappaphycus alvarezii, also known as “cottonii seaweed,” gained attention in India due to its potential as a sustainable source of biofuel production. Kappaphycus alvarezii is a red algae species that is rich in carbohydrates, particularly in the form of agar, carrageenan, and other polysaccharides. These carbohydrates can be converted into biofuels through processes such as fermentation, hydrolysis, and pyrolysis.
    ● Biofuels derived from seaweed are considered more sustainable than those derived from traditional feedstocks like corn, sugarcane, and soybean, as seaweed cultivation does not require arable land, freshwater, or chemical fertilizers. This reduces competition for resources with food crops and mitigates environmental impacts such as deforestation and eutrophication. Additionally, seaweed cultivation can help absorb excess nutrients from the water and improve water quality, providing benefits to local marine ecosystems.

    Incorrect

    Answer: A
    Explanation:
    ● The importance of Kappaphycus alvarezii, also known as “cottonii seaweed,” gained attention in India due to its potential as a sustainable source of biofuel production. Kappaphycus alvarezii is a red algae species that is rich in carbohydrates, particularly in the form of agar, carrageenan, and other polysaccharides. These carbohydrates can be converted into biofuels through processes such as fermentation, hydrolysis, and pyrolysis.
    ● Biofuels derived from seaweed are considered more sustainable than those derived from traditional feedstocks like corn, sugarcane, and soybean, as seaweed cultivation does not require arable land, freshwater, or chemical fertilizers. This reduces competition for resources with food crops and mitigates environmental impacts such as deforestation and eutrophication. Additionally, seaweed cultivation can help absorb excess nutrients from the water and improve water quality, providing benefits to local marine ecosystems.

  2. Question 2 of 5
    2. Question
    2 points

    2. The word ‘Floatovoltaics’ sometimes mentioned in the media is in reference to –

    Correct

    Answer: A
    Explanation:
    ● ‘Floatovoltaics’ refers to an innovative approach to installing solar panels on water bodies, which can help reduce land use and increase efficiency.
    ● Floating solar farms, also known as “floatovoltaics,” involve mounting solar panels on floating structures anchored to a water body, such as a lake, reservoir, or pond.
    There are several advantages to floating solar farms compared to traditional land-based solar installations:
    ● Reduced land use: By utilizing water surfaces, floating solar farms help conserve valuable land resources, which can be especially important in densely populated countries like India where land availability is limited.
    ● Increased efficiency: Solar panels on water bodies can benefit from the cooling effect of the water, which helps maintain lower temperatures for the panels. This can improve their efficiency, as solar panels tend to perform better at lower temperatures.
    ● Water conservation: Floating solar farms can help reduce water evaporation from the water bodies they are installed on by shading the surface, which can be particularly useful in regions facing water scarcity.
    ● Algae growth reduction: By covering the surface of water bodies, floating solar farms can inhibit the growth of algae, which can be detrimental to water quality and the surrounding ecosystem.

    Incorrect

    Answer: A
    Explanation:
    ● ‘Floatovoltaics’ refers to an innovative approach to installing solar panels on water bodies, which can help reduce land use and increase efficiency.
    ● Floating solar farms, also known as “floatovoltaics,” involve mounting solar panels on floating structures anchored to a water body, such as a lake, reservoir, or pond.
    There are several advantages to floating solar farms compared to traditional land-based solar installations:
    ● Reduced land use: By utilizing water surfaces, floating solar farms help conserve valuable land resources, which can be especially important in densely populated countries like India where land availability is limited.
    ● Increased efficiency: Solar panels on water bodies can benefit from the cooling effect of the water, which helps maintain lower temperatures for the panels. This can improve their efficiency, as solar panels tend to perform better at lower temperatures.
    ● Water conservation: Floating solar farms can help reduce water evaporation from the water bodies they are installed on by shading the surface, which can be particularly useful in regions facing water scarcity.
    ● Algae growth reduction: By covering the surface of water bodies, floating solar farms can inhibit the growth of algae, which can be detrimental to water quality and the surrounding ecosystem.

  3. Question 3 of 5
    3. Question
    2 points

    3. The term “Swarm Robotics” occasionally mentioned in the media in reference to –

    Correct

    Answer: D
    Explanation:
    ● The term “Swarm Robotics” refers to a group of small, coordinated robots that work together to accomplish tasks. Swarm robotics is inspired by the natural behavior of social insects like ants, bees, and termites, which work collectively to perform complex activities, such as foraging for food or constructing nests. The robots in a swarm are usually simple and operate autonomously, relying on local sensing and communication with nearby robots to coordinate their actions.
    ● Swarm robotics has several advantages over traditional robotic systems:
    1. Scalability: Swarm robotic systems can be easily scaled up or down by adding or removing robots, making them adaptable to various task requirements.
    2. Flexibility: The robots in a swarm can adapt to changes in the environment or task by adjusting their behavior, making the system more versatile and resilient.
    3. Robustness: Since the swarm operates as a collective, the failure of individual robots does not significantly impact the overall performance of the system. This makes swarm robotic systems more robust and reliable.
    4. Cost-effectiveness: Swarm robots are typically small and simple, which can result in lower production and maintenance costs compared to more complex robots.

    Incorrect

    Answer: D
    Explanation:
    ● The term “Swarm Robotics” refers to a group of small, coordinated robots that work together to accomplish tasks. Swarm robotics is inspired by the natural behavior of social insects like ants, bees, and termites, which work collectively to perform complex activities, such as foraging for food or constructing nests. The robots in a swarm are usually simple and operate autonomously, relying on local sensing and communication with nearby robots to coordinate their actions.
    ● Swarm robotics has several advantages over traditional robotic systems:
    1. Scalability: Swarm robotic systems can be easily scaled up or down by adding or removing robots, making them adaptable to various task requirements.
    2. Flexibility: The robots in a swarm can adapt to changes in the environment or task by adjusting their behavior, making the system more versatile and resilient.
    3. Robustness: Since the swarm operates as a collective, the failure of individual robots does not significantly impact the overall performance of the system. This makes swarm robotic systems more robust and reliable.
    4. Cost-effectiveness: Swarm robots are typically small and simple, which can result in lower production and maintenance costs compared to more complex robots.

  4. Question 4 of 5
    4. Question
    2 points

    4. Consider the following statements regarding the Liquid Air Energy Storage (LAES):
    1. LAES is an energy storage technology that uses cryogenic air as the storage medium to store excess electricity.
    2. During periods of high electricity demand, LAES systems release the stored energy back to the grid by expanding the cryogenic air to drive a turbine.
    3. One of the disadvantages of LAES technology is that it has a higher environmental impact compared to traditional energy storage methods, such as lithium-ion batteries.
    How many of the statements given above are correct?

    Correct

    Answer: B
    Explanation:
    ● Liquid Air Energy Storage (LAES) is an innovative energy storage technology with potential environmental advantages over traditional methods.
    ● LAES is an energy storage technology that uses cryogenic air as the storage medium to store excess electricity. LAES systems work by compressing and cooling air to extremely low temperatures until it becomes a liquid. This cryogenic liquid air is then stored in insulated tanks, effectively storing the energy for later use.
    ● During periods of high electricity demand, LAES systems release the stored energy back to the grid by expanding the cryogenic air to drive a turbine. When electricity is needed, the stored liquid air is released from the tanks, warmed, and expanded back into a gaseous state. This expansion process drives a turbine, generating electricity that is fed back into the grid.
    ● One of the advantages of LAES technology is that it has a lower environmental impact compared to traditional energy storage methods, such as lithium-ion batteries. LAES systems use air, a readily available and environmentally benign material, as their storage medium. They do not require scarce or hazardous materials like those used in lithium-ion batteries, making them a more sustainable option for large-scale energy storage.
    Thus, only 1 and 2 statements given in the question are correct.

    Incorrect

    Answer: B
    Explanation:
    ● Liquid Air Energy Storage (LAES) is an innovative energy storage technology with potential environmental advantages over traditional methods.
    ● LAES is an energy storage technology that uses cryogenic air as the storage medium to store excess electricity. LAES systems work by compressing and cooling air to extremely low temperatures until it becomes a liquid. This cryogenic liquid air is then stored in insulated tanks, effectively storing the energy for later use.
    ● During periods of high electricity demand, LAES systems release the stored energy back to the grid by expanding the cryogenic air to drive a turbine. When electricity is needed, the stored liquid air is released from the tanks, warmed, and expanded back into a gaseous state. This expansion process drives a turbine, generating electricity that is fed back into the grid.
    ● One of the advantages of LAES technology is that it has a lower environmental impact compared to traditional energy storage methods, such as lithium-ion batteries. LAES systems use air, a readily available and environmentally benign material, as their storage medium. They do not require scarce or hazardous materials like those used in lithium-ion batteries, making them a more sustainable option for large-scale energy storage.
    Thus, only 1 and 2 statements given in the question are correct.

  5. Question 5 of 5
    5. Question
    2 points

    5. Consider the following statements about Sponge City:
    1. Sponge Cities are designed to absorb, store, and purify rainwater through the use of green infrastructure and permeable surfaces.
    2. The Sponge City concept was first developed in the Netherlands, where it has been implemented in several cities to reduce the impact of flooding.
    3. Sponge Cities rely solely on the use of grey infrastructure, such as concrete channels and underground drainage systems, to manage stormwater.
    How many of the statements given above is/are correct?

    Correct

    Answer: A
    Explanation:
    The correct answer is (a) 1 only.
    ● Sponge Cities are designed to absorb, store, and purify rainwater through the use of green infrastructure and permeable surfaces. Sponge Cities are designed to mimic the natural hydrological cycle by using green infrastructure and permeable surfaces, such as rain gardens, bioswales, and green roofs, to absorb and store rainwater. This helps to reduce the impact of flooding and improve water quality.
    ● The Sponge City concept was first developed in China, where it has been implemented in several cities to reduce the impact of flooding. The Sponge City concept was first introduced in China in 2015 as part of the country’s efforts to combat flooding and improve water quality. Since then, several cities in China have implemented the Sponge City concept, including Wuhan, Shanghai, and Xiamen.
    ● Sponge Cities rely on a combination of green and grey infrastructure to manage stormwater, with an emphasis on green infrastructure to mimic natural water systems.

    Incorrect

    Answer: A
    Explanation:
    The correct answer is (a) 1 only.
    ● Sponge Cities are designed to absorb, store, and purify rainwater through the use of green infrastructure and permeable surfaces. Sponge Cities are designed to mimic the natural hydrological cycle by using green infrastructure and permeable surfaces, such as rain gardens, bioswales, and green roofs, to absorb and store rainwater. This helps to reduce the impact of flooding and improve water quality.
    ● The Sponge City concept was first developed in China, where it has been implemented in several cities to reduce the impact of flooding. The Sponge City concept was first introduced in China in 2015 as part of the country’s efforts to combat flooding and improve water quality. Since then, several cities in China have implemented the Sponge City concept, including Wuhan, Shanghai, and Xiamen.
    ● Sponge Cities rely on a combination of green and grey infrastructure to manage stormwater, with an emphasis on green infrastructure to mimic natural water systems.

window.wpAdvQuizInitList = window.wpAdvQuizInitList || []; window.wpAdvQuizInitList.push({ id: '#wpAdvQuiz_5', init: { quizId: 5, mode: 1, globalPoints: 10, timelimit: 0, resultsGrade: [0], bo: 0, qpp: 0, catPoints: [10], formPos: 0, lbn: "Finish quiz", json: {"21":{"type":"single","id":21,"catId":0,"points":2,"correct":[1,0,0,0]},"22":{"type":"single","id":22,"catId":0,"points":2,"correct":[1,0,0,0]},"23":{"type":"single","id":23,"catId":0,"points":2,"correct":[0,0,0,1]},"24":{"type":"single","id":24,"catId":0,"points":2,"correct":[0,1,0,0]},"25":{"type":"single","id":25,"catId":0,"points":2,"correct":[1,0,0,0]}} } });




Day-524 | Daily MCQs | UPSC Prelims | SCIENCE AND TECHNOLOGICAL DEVELOPMENTS

Day-524

Time limit: 0

Quiz-summary

0 of 5 questions completed

Questions:

  1. 1
  2. 2
  3. 3
  4. 4
  5. 5

Information

To attempt the Quiz, simply click on START Button.

You have already completed the quiz before. Hence you can not start it again.

Quiz is loading...

You must sign in or sign up to start the quiz.

You have to finish following quiz, to start this quiz:

Results

0 of 5 questions answered correctly

Your time:

Time has elapsed

You have reached 0 of 0 points, (0)

Average score
 
 
Your score
 
 

Categories

  1. Not categorized 0%
Your result has been entered into leaderboard
Loading
captcha
  1. 1
  2. 2
  3. 3
  4. 4
  5. 5
  1. Answered
  2. Review
  1. Question 1 of 5
    1. Question
    2 points

    1. With reference to the congenital and hereditary genetic diseases, consider the following statements:
    1. The prevalence of genetic disorders increases because consanguineous marriage is favored in many communities.
    2. The UMMID initiative is designed to treat genetic disorders and it is based on the concept of ‘Prevention is better than Cure’.
    3. This initiative has established National Inherited Diseases AdministratioN (NIDAN) Kendras in all Aspirational Districts.
    How many of the statements given above are correct?

    Correct

    Answer: B
    Explanation:
    Statement 1 is correct: Congenital and hereditary genetic diseases are a significant health burden in India, and hence there is a need for adequate and effective genetic testing and counselling services. In India’s urban areas, congenital malformations and genetic disorders are the third most common cause of mortality in newborns. With a very large population with high birth rate, and consanguineous marriage favored in many communities, prevalence of genetic disorders is high in India. Considering these facts, the Department of Biotechnology (DBT), GoI has started the Unique Methods of Management and Treatment of Inherited Disorders (UMMID) Initiative.
    The whole initiative is designed on the concept of ‘Prevention is better than Cure’. The diagnostic tests for many of the genetic disorders available in India are at a relatively nascent stage. While antenatal diagnostics for a few genetic diseases are available, these are available in very few hospitals.
    Statement 3 is incorrect: Considering these facts, the Department, in the year 2019, launched the DBT-UMMID initiative with the objectives of establishing Genetic Diagnostic Units called National Inherited Diseases AdministratioN (NIDAN) Kendras in Government Hospitals, producing skilled clinicians in the area of Human Genetics (Biochemical Genetics, Cytogenetics, Molecular Genetics, Clinical Genetics and Comprehensive Clinical Care), and screening of pregnant women and newborn babies for diagnosis of inherited genetic diseases in aspirational districts to provide comprehensive clinical care.

    Statement 2 is correct: The DBT-UMMID Initiative is designed on the concept of “Prevention is better than cure”, and is a step towards the use of cutting edge scientific technology and molecular medicine for extending Universal Health Coverage for all.

    UMMID is a significant contributor to the implementation of the National Policy for Rare Diseases, 2021. UMMID seeks to contribute to the infrastructure and human resource needs for management of genetic disorders, which account for 80% of rare disorders.
    Objectives:
    ● Establishment of genetic diagnostic centres (NIDAN Kendras) in different parts of the country to provide patient care services.
    ● Improve the components of medical genetics training in medical education, preparing medical doctors of the twenty-first century for the era of molecular medicine.
    ● Create awareness about genetic disorders amongst clinicians and the general public, so that the patients and their families get appropriate diagnosis, management & preventive services for genetic disorders.
    ● Spread the reach of diagnostic facilities for rare genetic disorders, pharmacogenetics, prenatal diagnosis, and population-based screening for prevention.
    ● Contribute data for future research on rare or common genetic disorders.
    Achievements: So far, more than 60000 beneficiaries have availed antenatal screening, while more than 33000 newborns have undergone screening for various genetic and rare disorders under the UMMID Initiative.
    Hence, option B is correct.

    Incorrect

    Answer: B
    Explanation:
    Statement 1 is correct: Congenital and hereditary genetic diseases are a significant health burden in India, and hence there is a need for adequate and effective genetic testing and counselling services. In India’s urban areas, congenital malformations and genetic disorders are the third most common cause of mortality in newborns. With a very large population with high birth rate, and consanguineous marriage favored in many communities, prevalence of genetic disorders is high in India. Considering these facts, the Department of Biotechnology (DBT), GoI has started the Unique Methods of Management and Treatment of Inherited Disorders (UMMID) Initiative.
    The whole initiative is designed on the concept of ‘Prevention is better than Cure’. The diagnostic tests for many of the genetic disorders available in India are at a relatively nascent stage. While antenatal diagnostics for a few genetic diseases are available, these are available in very few hospitals.
    Statement 3 is incorrect: Considering these facts, the Department, in the year 2019, launched the DBT-UMMID initiative with the objectives of establishing Genetic Diagnostic Units called National Inherited Diseases AdministratioN (NIDAN) Kendras in Government Hospitals, producing skilled clinicians in the area of Human Genetics (Biochemical Genetics, Cytogenetics, Molecular Genetics, Clinical Genetics and Comprehensive Clinical Care), and screening of pregnant women and newborn babies for diagnosis of inherited genetic diseases in aspirational districts to provide comprehensive clinical care.

    Statement 2 is correct: The DBT-UMMID Initiative is designed on the concept of “Prevention is better than cure”, and is a step towards the use of cutting edge scientific technology and molecular medicine for extending Universal Health Coverage for all.

    UMMID is a significant contributor to the implementation of the National Policy for Rare Diseases, 2021. UMMID seeks to contribute to the infrastructure and human resource needs for management of genetic disorders, which account for 80% of rare disorders.
    Objectives:
    ● Establishment of genetic diagnostic centres (NIDAN Kendras) in different parts of the country to provide patient care services.
    ● Improve the components of medical genetics training in medical education, preparing medical doctors of the twenty-first century for the era of molecular medicine.
    ● Create awareness about genetic disorders amongst clinicians and the general public, so that the patients and their families get appropriate diagnosis, management & preventive services for genetic disorders.
    ● Spread the reach of diagnostic facilities for rare genetic disorders, pharmacogenetics, prenatal diagnosis, and population-based screening for prevention.
    ● Contribute data for future research on rare or common genetic disorders.
    Achievements: So far, more than 60000 beneficiaries have availed antenatal screening, while more than 33000 newborns have undergone screening for various genetic and rare disorders under the UMMID Initiative.
    Hence, option B is correct.

  2. Question 2 of 5
    2. Question
    2 points

    2. Which of the following pathogens are part of the Indian Priority Pathogens List?
    1. Non-fermenting bacteria
    2. Salmonella species
    3. Streptococcus pneumoniae
    4. Shigella species
    5. Haemophilus influenzae
    How many of the above are correct?

    Correct

    Answer: D
    Explanation:
    Antimicrobial resistance (AMR) is one of the top 10 global health threats faced by the world today and can have a major impact on the economy, society, food safety and public health. To address the issue of Antimicrobial Resistance, an Indian Priority Pathogen List has been developed by the WHO Country Office for India in collaboration with the Department of Biotechnology, Government of India. The Indian Priority Pathogens List has been developed to guide research, discovery and development of new antibiotics in India.
    Hence, option D is correct.

    Incorrect

    Answer: D
    Explanation:
    Antimicrobial resistance (AMR) is one of the top 10 global health threats faced by the world today and can have a major impact on the economy, society, food safety and public health. To address the issue of Antimicrobial Resistance, an Indian Priority Pathogen List has been developed by the WHO Country Office for India in collaboration with the Department of Biotechnology, Government of India. The Indian Priority Pathogens List has been developed to guide research, discovery and development of new antibiotics in India.
    Hence, option D is correct.

  3. Question 3 of 5
    3. Question
    2 points

    3. With reference to eSanjeevani, consider the following statements:
    1. It is a national telemedicine service that aims for digital health equity to achieve Universal Health Coverage (UHC).
    2. In the eSanjeevani system, AI chatbots provide teleconsultation services to patients.
    3. It is a cloud-based telemedicine system.
    How many of the statements given above are correct?

    Correct

    Answer: B
    Explanation:
    Statement 1 is correct: eSanjeevani – National Telemedicine Service of India is a step towards digital health equity to achieve Universal Health Coverage (UHC). eSanjeevani facilitates quick and easy access to doctors and medical specialists from your smartphones. You can also access quality health services remotely via eSanjeevani by visiting the nearest Ayushman Bharat Health & Wellness Centre.
    Statement 2 is incorrect: It provides two types of consultation processes i.e. patient to doctor telemedicine and assisted telemedicine. In both the types of Telemedicine consultation there is a virtual presence of a human doctor.
    1. Patient to Doctor Telemedicine

    2. Assisted Telemedicine

    Statement 3 is correct: eSanjeevani is a cloud-based telemedicine system. It is accessible via an Internet connection from anywhere (in India) and any computing device. eSanjeevani allows users to create their Ayushman Bharat Health Account (ABHA) and use it to link and manage their existing health records.
    Hence, option B is correct.

    Incorrect

    Answer: B
    Explanation:
    Statement 1 is correct: eSanjeevani – National Telemedicine Service of India is a step towards digital health equity to achieve Universal Health Coverage (UHC). eSanjeevani facilitates quick and easy access to doctors and medical specialists from your smartphones. You can also access quality health services remotely via eSanjeevani by visiting the nearest Ayushman Bharat Health & Wellness Centre.
    Statement 2 is incorrect: It provides two types of consultation processes i.e. patient to doctor telemedicine and assisted telemedicine. In both the types of Telemedicine consultation there is a virtual presence of a human doctor.
    1. Patient to Doctor Telemedicine

    2. Assisted Telemedicine

    Statement 3 is correct: eSanjeevani is a cloud-based telemedicine system. It is accessible via an Internet connection from anywhere (in India) and any computing device. eSanjeevani allows users to create their Ayushman Bharat Health Account (ABHA) and use it to link and manage their existing health records.
    Hence, option B is correct.

  4. Question 4 of 5
    4. Question
    2 points

    4. With reference to India’s efforts towards Medical Value Travel (MVT), consider the following statements:
    1. It has rolled out the ‘National Strategy and Roadmap for Medical and Wellness Tourism’ to provide health services to the world at large.
    2. It has launched a ‘Heal in India’ campaign to market the nation as a wellness and medical tourism destination.
    3. India has launched e-visas for MVT travellers and accredited AYUSH centres, formalising their status in the Indian medical industry.
    How many of the above steps were taken by the Government of India to promote Medical Value Travel (MVT) in India?

    Correct

    Answer: C
    Explanation:
    India hosted the ‘One Earth One Health – Advantage Healthcare India – 2023’ programme in New Delhi, in a bid to promote India as a hub for medical tourism. With 500 foreign participants from over 70 countries, the two-day summit aimed to promote the export of medical services from India and provide networking opportunities for Medical Value Travel (MVT) experts, industry stakeholders and professionals.
    ● Statement 1 is correct: The Centre rolled out the ‘National Strategy and Roadmap for Medical and Wellness Tourism’ in January 2022 to provide health services to the world at large.
    ● Statement 2 is correct: The Centre has launched a ‘Heal in India’ campaign to market the nation as a wellness and medical tourism destination.
    ● Statement 3 is correct: India has launched e-visas for MVT travellers from 156 nations and has also accredited AYUSH centres, formalising their status in the Indian medical industry.

    Hence, option C is correct.

    Incorrect

    Answer: C
    Explanation:
    India hosted the ‘One Earth One Health – Advantage Healthcare India – 2023’ programme in New Delhi, in a bid to promote India as a hub for medical tourism. With 500 foreign participants from over 70 countries, the two-day summit aimed to promote the export of medical services from India and provide networking opportunities for Medical Value Travel (MVT) experts, industry stakeholders and professionals.
    ● Statement 1 is correct: The Centre rolled out the ‘National Strategy and Roadmap for Medical and Wellness Tourism’ in January 2022 to provide health services to the world at large.
    ● Statement 2 is correct: The Centre has launched a ‘Heal in India’ campaign to market the nation as a wellness and medical tourism destination.
    ● Statement 3 is correct: India has launched e-visas for MVT travellers from 156 nations and has also accredited AYUSH centres, formalising their status in the Indian medical industry.

    Hence, option C is correct.

  5. Question 5 of 5
    5. Question
    2 points

    5. With reference to lipids, consider the following statements:
    1. Lipids are generally water soluble.
    2. Lipids play important role in energy storage, cell membrane structure and hormone synthesis.
    3. Steroids are a class of lipids that include cholesterol and hormones like testosterone and estrogen.
    How many of the statements given above are correct?

    Correct

    Answer: B
    Explanation:
    Statement 1 is incorrect: Lipids are a diverse group of organic compounds that are essential for life. They include fats, oils, waxes, phospholipids, and steroids, among others. Lipids are insoluble in water but are soluble in organic solvents like alcohol and acetone. Lipids are generally water insoluble.

    Types of Lipids:
    1. Fats and oils: Fats and oils are composed of a glycerol molecule and three fatty acids. Fats are solid at room temperature, while oils are liquid. Examples include butter, lard, olive oil, and corn oil.
    2. Phospholipids: Phospholipids are important components of cell membranes. They are composed of a glycerol molecule, two fatty acids, and a phosphate group. The phosphate group is hydrophilic (water-loving), while the fatty acid tails are hydrophobic (water-fearing), which allows them to form a bilayer structure in cell membranes.
    3. Steroids: Steroids are a class of lipids that include cholesterol and hormones like testosterone and estrogen. Cholesterol is important in cell membrane structure and is also a precursor for the synthesis of other steroids. So, statement 3 is correct.
    4. Waxes: Waxes are composed of a long-chain fatty acid and a long-chain alcohol. They are found in nature as protective coatings on leaves and fruits, as well as in animal coats, like beeswax, carnauba wax, and lanolin.
    Statement 2 is correct: Lipids play important roles in energy storage, cell membrane structure and hormone synthesis.
    Hence, option B is correct.

    Incorrect

    Answer: B
    Explanation:
    Statement 1 is incorrect: Lipids are a diverse group of organic compounds that are essential for life. They include fats, oils, waxes, phospholipids, and steroids, among others. Lipids are insoluble in water but are soluble in organic solvents like alcohol and acetone. Lipids are generally water insoluble.

    Types of Lipids:
    1. Fats and oils: Fats and oils are composed of a glycerol molecule and three fatty acids. Fats are solid at room temperature, while oils are liquid. Examples include butter, lard, olive oil, and corn oil.
    2. Phospholipids: Phospholipids are important components of cell membranes. They are composed of a glycerol molecule, two fatty acids, and a phosphate group. The phosphate group is hydrophilic (water-loving), while the fatty acid tails are hydrophobic (water-fearing), which allows them to form a bilayer structure in cell membranes.
    3. Steroids: Steroids are a class of lipids that include cholesterol and hormones like testosterone and estrogen. Cholesterol is important in cell membrane structure and is also a precursor for the synthesis of other steroids. So, statement 3 is correct.
    4. Waxes: Waxes are composed of a long-chain fatty acid and a long-chain alcohol. They are found in nature as protective coatings on leaves and fruits, as well as in animal coats, like beeswax, carnauba wax, and lanolin.
    Statement 2 is correct: Lipids play important roles in energy storage, cell membrane structure and hormone synthesis.
    Hence, option B is correct.

window.wpAdvQuizInitList = window.wpAdvQuizInitList || []; window.wpAdvQuizInitList.push({ id: '#wpAdvQuiz_6', init: { quizId: 6, mode: 1, globalPoints: 10, timelimit: 0, resultsGrade: [0], bo: 0, qpp: 0, catPoints: [10], formPos: 0, lbn: "Finish quiz", json: {"26":{"type":"single","id":26,"catId":0,"points":2,"correct":[0,1,0,0]},"27":{"type":"single","id":27,"catId":0,"points":2,"correct":[0,0,0,1]},"28":{"type":"single","id":28,"catId":0,"points":2,"correct":[0,1,0,0]},"29":{"type":"single","id":29,"catId":0,"points":2,"correct":[0,0,1,0]},"30":{"type":"single","id":30,"catId":0,"points":2,"correct":[0,1,0,0]}} } });




Day-523 | Daily MCQs | UPSC Prelims | CURRENT DEVELOPMENTS

Day-523

Time limit: 0

Quiz-summary

0 of 5 questions completed

Questions:

  1. 1
  2. 2
  3. 3
  4. 4
  5. 5

Information

To attempt the Quiz, simply click on START Button.

You have already completed the quiz before. Hence you can not start it again.

Quiz is loading...

You must sign in or sign up to start the quiz.

You have to finish following quiz, to start this quiz:

Results

0 of 5 questions answered correctly

Your time:

Time has elapsed

You have reached 0 of 0 points, (0)

Average score
 
 
Your score
 
 

Categories

  1. Not categorized 0%
Your result has been entered into leaderboard
Loading
captcha
  1. 1
  2. 2
  3. 3
  4. 4
  5. 5
  1. Answered
  2. Review
  1. Question 1 of 5
    1. Question
    2 points

    1. Consider the following statements regarding the comparison between Micro-LED and OLED display:
    1. micro-LED has inorganic LED structure in comparison to organic emissive material in OLED display technology.
    2. micro-LED has higher power efficiency and response time than OLED displays.
    3. micro-LED has lower brightness and sunlight visibility than OLED displays.
    How many of the statements given above is/are correct?

    Correct

    Answer: A
    Explanation:
    Considered the next big transition in display technology, microLEDs are self-illuminating diodes that have brighter and better colour reproduction than Organic Light Emitting Diode (OLED) display technology.
    ● Statement 1 is correct: OLEDs use tiny sub-pixels made from organic emissive materials. Micro-LEDs are somewhat similar – but with an inorganic LED structure. Compared to OLEDs, Micro-LEDs promise to be much more efficient and bright, more durable (higher lifetime) and with a higher color gamut, mostly due to the superior performance of LEDs over their organic counterparts.

    ● Statement 2 is incorrect: microLED has higher power efficiency but lower response time than OLED displays.
    ● Statement 3 is incorrect: microLED has higher brightness and sunlight visibility than OLED displays.
    About Micro-LED:
    ● Micro-LED (also known as mLED or µLED) is a display technology that is based on tiny (hence, micro) LED devices that are used to directly create color pixels. Micro-LED displays have the potential to create highly efficient and great looking flexible displays, to challenge the current high-end OLED displays.
    Micro-LED vs LED:
    ● Current so-called LED displays are actually LCD displays that use LED as backlighting units – which are always on with a liquid-crystal layer that is used to create the actual image (i.e. block the light where needed). This complicated LCD structure results in a device with serious image quality drawbacks (mainly a low response time and relatively poor contrast ratio) and also difficulties in achieving flexibility and high-quality transparency.
    Micro-LED vs OLED:
    ● OLEDs use tiny sub-pixels made from organic emissive materials. Micro-LEDs are somewhat similar – but with an inorganic LED structure. Compared to OLEDs, Micro-LEDs promise to be much more efficient and bright, more durable (higher lifetime) and with a higher color gamut. Micro-LEDs are based on well-established LED devices, which means that it could potentially be a technology that is relatively easy to scale up.

    Incorrect

    Answer: A
    Explanation:
    Considered the next big transition in display technology, microLEDs are self-illuminating diodes that have brighter and better colour reproduction than Organic Light Emitting Diode (OLED) display technology.
    ● Statement 1 is correct: OLEDs use tiny sub-pixels made from organic emissive materials. Micro-LEDs are somewhat similar – but with an inorganic LED structure. Compared to OLEDs, Micro-LEDs promise to be much more efficient and bright, more durable (higher lifetime) and with a higher color gamut, mostly due to the superior performance of LEDs over their organic counterparts.

    ● Statement 2 is incorrect: microLED has higher power efficiency but lower response time than OLED displays.
    ● Statement 3 is incorrect: microLED has higher brightness and sunlight visibility than OLED displays.
    About Micro-LED:
    ● Micro-LED (also known as mLED or µLED) is a display technology that is based on tiny (hence, micro) LED devices that are used to directly create color pixels. Micro-LED displays have the potential to create highly efficient and great looking flexible displays, to challenge the current high-end OLED displays.
    Micro-LED vs LED:
    ● Current so-called LED displays are actually LCD displays that use LED as backlighting units – which are always on with a liquid-crystal layer that is used to create the actual image (i.e. block the light where needed). This complicated LCD structure results in a device with serious image quality drawbacks (mainly a low response time and relatively poor contrast ratio) and also difficulties in achieving flexibility and high-quality transparency.
    Micro-LED vs OLED:
    ● OLEDs use tiny sub-pixels made from organic emissive materials. Micro-LEDs are somewhat similar – but with an inorganic LED structure. Compared to OLEDs, Micro-LEDs promise to be much more efficient and bright, more durable (higher lifetime) and with a higher color gamut. Micro-LEDs are based on well-established LED devices, which means that it could potentially be a technology that is relatively easy to scale up.

  2. Question 2 of 5
    2. Question
    2 points

    2. Recently seen in the news, the ‘Title 42’ is related to which of the following?

    Correct

    Answer: B
    Explanation:
    ● After more than three years, a controversial Trump-era immigration policy known as Title 42 is set to expire on 11 May.
    Option(b) is correct:
    ● ‘Title 42’ policy came to the forefront of US immigration policy in March 2020, when the Trump administration invoked the statute to stop the spread of Covid-19 across its borders. With Title 42 in place, US authorities were able to swiftly expel would-be migrants attempting to cross the border from Mexico – including those seeking humanitarian asylum – using the pandemic as justification.

    Incorrect

    Answer: B
    Explanation:
    ● After more than three years, a controversial Trump-era immigration policy known as Title 42 is set to expire on 11 May.
    Option(b) is correct:
    ● ‘Title 42’ policy came to the forefront of US immigration policy in March 2020, when the Trump administration invoked the statute to stop the spread of Covid-19 across its borders. With Title 42 in place, US authorities were able to swiftly expel would-be migrants attempting to cross the border from Mexico – including those seeking humanitarian asylum – using the pandemic as justification.

  3. Question 3 of 5
    3. Question
    2 points

    3. Consider the following statements regarding Sand Batteries:
    1. Sand is used in these batteries because it has higher thermal energy storage capacity and heat conductivity than water.
    2. It can be used for a centralised heating network that can keep buildings and public water systems warm.
    3. Finland is the first country in the world to install Sand Batteries.
    How many of the statements given above is/are correct?

    Correct

    Answer: B
    Explanation:
    Finland has installed the world’s first sand battery that can store heat from renewable energy sources for months.
    ● Statement 1 is incorrect: Sand-based heat storages can store several times the amount of energy that can be stored in a water tank of a similar size. Sand can be heated up to 600 degrees Celsius (°C), whereas water starts to boil at 100°C. It also has low heat conductivity, which reduces energy loss.
    ● Statement 2 is correct: It can be used for a centralised heating network that can keep buildings and public water systems warm.
    ● Statement 3 is correct: Finland is the first country to install Sand Batteries.

    Incorrect

    Answer: B
    Explanation:
    Finland has installed the world’s first sand battery that can store heat from renewable energy sources for months.
    ● Statement 1 is incorrect: Sand-based heat storages can store several times the amount of energy that can be stored in a water tank of a similar size. Sand can be heated up to 600 degrees Celsius (°C), whereas water starts to boil at 100°C. It also has low heat conductivity, which reduces energy loss.
    ● Statement 2 is correct: It can be used for a centralised heating network that can keep buildings and public water systems warm.
    ● Statement 3 is correct: Finland is the first country to install Sand Batteries.

  4. Question 4 of 5
    4. Question
    2 points

    4. Consider the following statements regarding the High Altitude Surveillance Balloon:
    1. It is usually placed in the ionosphere.
    2. It has the ability to scan a wider range of territory than satellites.
    3. It is powered by the solar energy panels attached to it.
    4. It randomly moves on the basis of wind direction at a particular altitude.
    How many of the statements given above is/are correct?

    Correct

    Answer: C
    Explanation:
    U.S. officials said that a Chinese “surveillance balloon” has been flying over the United States for several days. It drifted across the US and flew far higher than passenger jets.
    ● Statement 1 is incorrect: High Altitude Surveillance Balloons usually placed in the stratosphere.
    ● Statement 2 is correct: The advantages of balloons over satellites include the ability to scan wide swathes of territory from closer in, and to be able to spend more time over a target area.
    ● Statement 3 is correct: It is powered by the Solar energy panels attached to it.
    ● Statement 4 is correct: It randomly moves on the basis of wind direction at a particular altitude.

    Incorrect

    Answer: C
    Explanation:
    U.S. officials said that a Chinese “surveillance balloon” has been flying over the United States for several days. It drifted across the US and flew far higher than passenger jets.
    ● Statement 1 is incorrect: High Altitude Surveillance Balloons usually placed in the stratosphere.
    ● Statement 2 is correct: The advantages of balloons over satellites include the ability to scan wide swathes of territory from closer in, and to be able to spend more time over a target area.
    ● Statement 3 is correct: It is powered by the Solar energy panels attached to it.
    ● Statement 4 is correct: It randomly moves on the basis of wind direction at a particular altitude.

  5. Question 5 of 5
    5. Question
    2 points

    5. Recently seen in the news, Phytorid technology is related to which of the following?

    Correct

    Answer: B
    Explanation:
    ● The Union Minister of Science & Technology inaugurated the efficient Phytorid Technology Sewage Treatment Plant (STP) at CSIR-NCL at Pune which is a self sustainable technology for wastewater treatment that works on the principle of natural wetland.
    Option(b) is correct:
    ● The phytorid technology treatment is a subsurface flow type in which wastewater is applied to a cell / system filled with porous media such as crushed bricks, gravel and stones. The hydraulics is maintained in such a manner that wastewater does not rise to the surface retaining a free board at the top of the filled media.

    Incorrect

    Answer: B
    Explanation:
    ● The Union Minister of Science & Technology inaugurated the efficient Phytorid Technology Sewage Treatment Plant (STP) at CSIR-NCL at Pune which is a self sustainable technology for wastewater treatment that works on the principle of natural wetland.
    Option(b) is correct:
    ● The phytorid technology treatment is a subsurface flow type in which wastewater is applied to a cell / system filled with porous media such as crushed bricks, gravel and stones. The hydraulics is maintained in such a manner that wastewater does not rise to the surface retaining a free board at the top of the filled media.

window.wpAdvQuizInitList = window.wpAdvQuizInitList || []; window.wpAdvQuizInitList.push({ id: '#wpAdvQuiz_7', init: { quizId: 7, mode: 1, globalPoints: 10, timelimit: 0, resultsGrade: [0], bo: 0, qpp: 0, catPoints: [10], formPos: 0, lbn: "Finish quiz", json: {"31":{"type":"single","id":31,"catId":0,"points":2,"correct":[1,0,0,0]},"32":{"type":"single","id":32,"catId":0,"points":2,"correct":[0,1,0,0]},"33":{"type":"single","id":33,"catId":0,"points":2,"correct":[0,1,0,0]},"34":{"type":"single","id":34,"catId":0,"points":2,"correct":[0,0,1,0]},"35":{"type":"single","id":35,"catId":0,"points":2,"correct":[0,1,0,0]}} } });




Day-522 | Daily MCQs | UPSC Prelims | ENVIRONMENT AND ECOLOGY

Day-522

Time limit: 0

Quiz-summary

0 of 5 questions completed

Questions:

  1. 1
  2. 2
  3. 3
  4. 4
  5. 5

Information

To attempt the Quiz, simply click on START Button.

You have already completed the quiz before. Hence you can not start it again.

Quiz is loading...

You must sign in or sign up to start the quiz.

You have to finish following quiz, to start this quiz:

Results

0 of 5 questions answered correctly

Your time:

Time has elapsed

You have reached 0 of 0 points, (0)

Average score
 
 
Your score
 
 

Categories

  1. Not categorized 0%
Your result has been entered into leaderboard
Loading
captcha
  1. 1
  2. 2
  3. 3
  4. 4
  5. 5
  1. Answered
  2. Review
  1. Question 1 of 5
    1. Question
    2 points

    1. Consider the following statements with respect to the National Wasteland Development Board (NWDB):
    1. It was established in 1985 under the Ministry of Urban Development.
    2. The NWDB only focuses on the development of wastelands in urban areas and does not cover rural areas.
    Which of the statements given above is/are correct?

    Correct

    Answer: D
    Explanation:
    ● Statement 1 is incorrect: The National Wasteland Development Board (NWDB) is a statutory body established by the Government of India in 1985 under the Ministry of Environment, Forest and Climate Change. In July, 1992, the National Wasteland Development Board was reconstituted and placed in the newly created Department of Wasteland Development under the Ministry of Rural Development. Subsequently, the Department of Wasteland Development was renamed as Department of Land Resources in 1999.
    ● Its primary objective is to develop and rehabilitate degraded and wasteland areas in the country, and to promote sustainable development practices in these areas. The board operates under the overall guidance of the National Land Use and Wasteland Development Council (NLWDC).
    ● The NWDB is responsible for formulating policies and programs related to wasteland development and rehabilitation, providing technical and financial assistance to state governments and other agencies involved in wasteland development activities, and coordinating research and development activities related to wasteland management. The board also plays an important role in capacity building and training of stakeholders involved in wasteland development activities.
    ● Statement 2 is incorrect: The National Wasteland Development Board (NWDB) focuses on the development of wastelands across the country, including both rural and urban areas.

    Incorrect

    Answer: D
    Explanation:
    ● Statement 1 is incorrect: The National Wasteland Development Board (NWDB) is a statutory body established by the Government of India in 1985 under the Ministry of Environment, Forest and Climate Change. In July, 1992, the National Wasteland Development Board was reconstituted and placed in the newly created Department of Wasteland Development under the Ministry of Rural Development. Subsequently, the Department of Wasteland Development was renamed as Department of Land Resources in 1999.
    ● Its primary objective is to develop and rehabilitate degraded and wasteland areas in the country, and to promote sustainable development practices in these areas. The board operates under the overall guidance of the National Land Use and Wasteland Development Council (NLWDC).
    ● The NWDB is responsible for formulating policies and programs related to wasteland development and rehabilitation, providing technical and financial assistance to state governments and other agencies involved in wasteland development activities, and coordinating research and development activities related to wasteland management. The board also plays an important role in capacity building and training of stakeholders involved in wasteland development activities.
    ● Statement 2 is incorrect: The National Wasteland Development Board (NWDB) focuses on the development of wastelands across the country, including both rural and urban areas.

  2. Question 2 of 5
    2. Question
    2 points

    2. ‘It is native to the Mediterranean region, as well as grown in Europe, Africa, Asia, and the Americas. It is a hardy plant that is well-adapted to drought conditions. It can survive in dry environments with minimal water and is often used in xeriscaping and water-wise gardening. Also known for its unique and pleasant fragrance, it is often used in perfumes, soaps, and other personal care products.’
    Which of the following plants has been described in the above paragraph?

    Correct

    Answer: B
    Explanation:
    Lavender is a type of flowering plant that belongs to the mint family, Lamiaceae. The plant is known for its distinctive scent and purple-colored flowers, and has been used for centuries for its medicinal properties and in perfumes, soaps, and cosmetics. There are over 40 different species of lavender, and it is native to the Mediterranean region, as well as areas in Europe, Africa, Asia, and the Americas. Lavender is commonly used in aromatherapy to help reduce stress and anxiety, and it is also used in traditional medicine to treat various conditions such as headaches, insomnia, and skin irritations. Additionally, the essential oils derived from lavender are used in culinary preparations and in the production of herbal teas.
    Lavender has several unique features, some of which are:
    ● Fragrance: Lavender is known for its unique and pleasant fragrance that is often used in perfumes, soaps, and other personal care products.
    ● Drought tolerance: Lavender is a hardy plant that is well-adapted to drought conditions. It can survive in dry environments with minimal water and is often used in xeriscaping and water-wise gardening.
    ● Pest resistance: Lavender is naturally resistant to pests such as aphids, mites, and caterpillars. This makes it a great addition to gardens and landscapes that are susceptible to pest infestations.
    ● Ornamental value: Lavender is a popular ornamental plant that adds beauty and color to gardens, landscapes, and indoor spaces. Its purple flowers and silver-green foliage make it an attractive addition to any setting.
    ● Medicinal properties: Lavender has several medicinal properties and is used to treat a variety of health conditions such as anxiety, insomnia, pain, and inflammation. Its essential oil is also used in aromatherapy to promote relaxation and reduce stress.

    Incorrect

    Answer: B
    Explanation:
    Lavender is a type of flowering plant that belongs to the mint family, Lamiaceae. The plant is known for its distinctive scent and purple-colored flowers, and has been used for centuries for its medicinal properties and in perfumes, soaps, and cosmetics. There are over 40 different species of lavender, and it is native to the Mediterranean region, as well as areas in Europe, Africa, Asia, and the Americas. Lavender is commonly used in aromatherapy to help reduce stress and anxiety, and it is also used in traditional medicine to treat various conditions such as headaches, insomnia, and skin irritations. Additionally, the essential oils derived from lavender are used in culinary preparations and in the production of herbal teas.
    Lavender has several unique features, some of which are:
    ● Fragrance: Lavender is known for its unique and pleasant fragrance that is often used in perfumes, soaps, and other personal care products.
    ● Drought tolerance: Lavender is a hardy plant that is well-adapted to drought conditions. It can survive in dry environments with minimal water and is often used in xeriscaping and water-wise gardening.
    ● Pest resistance: Lavender is naturally resistant to pests such as aphids, mites, and caterpillars. This makes it a great addition to gardens and landscapes that are susceptible to pest infestations.
    ● Ornamental value: Lavender is a popular ornamental plant that adds beauty and color to gardens, landscapes, and indoor spaces. Its purple flowers and silver-green foliage make it an attractive addition to any setting.
    ● Medicinal properties: Lavender has several medicinal properties and is used to treat a variety of health conditions such as anxiety, insomnia, pain, and inflammation. Its essential oil is also used in aromatherapy to promote relaxation and reduce stress.

  3. Question 3 of 5
    3. Question
    2 points

    3. Which one of the following will be a pioneer community in Hydrosere?

    Correct

    Answer: A
    Explanation:
    ● A hydrosere is a succession of plant communities that occurs over time in a freshwater ecosystem. The pioneer community in a hydrosere is typically made up of floating and submerged plants, such as algae and duckweed. These plants are adapted to living in water and can grow quickly to cover the water surface. As they grow, they create a new habitat that allows other plant species to establish themselves, leading to the development of a more complex plant community.
    ● Water lilies and cattails are usually found in later stages of hydrosere succession, while willow trees grow on land and are not part of the hydrosere succession.

    Incorrect

    Answer: A
    Explanation:
    ● A hydrosere is a succession of plant communities that occurs over time in a freshwater ecosystem. The pioneer community in a hydrosere is typically made up of floating and submerged plants, such as algae and duckweed. These plants are adapted to living in water and can grow quickly to cover the water surface. As they grow, they create a new habitat that allows other plant species to establish themselves, leading to the development of a more complex plant community.
    ● Water lilies and cattails are usually found in later stages of hydrosere succession, while willow trees grow on land and are not part of the hydrosere succession.

  4. Question 4 of 5
    4. Question
    2 points

    4. How many of the following are the objectives of the National Forest Policy 1988?
    1. Protect, improve, and enhance the production of minor forest produce with regard to the generation of employment for the tribals.
    2. To focus on methods to check soil-erosion, denudation and floods.
    3. To make provisions for the protection of degraded land which can be good land resources.
    Select the correct answer using the code given below:

    Correct

    Answer: A
    Explanation:
    Only statement 1 is correct:
    The first National Forest Policy in independent India took effect in 1952, with a second edition in 1988. The main theme of this policy of 1988 is protection, conservation, and development of forests. Salient Features and Goals of National Forest Policy-
    ● Maintenance of environmental stability through preservation and restoration of ecological balance.
    ● Conservation of Natural Heritage(existing).
    ● Checking Soil Erosion and Denudation in catchment areas of rivers, lakes, and reservoirs;
    ● Checking extension of sand dunes in desert areas of Rajasthan and along coastal tracts;
    ● Substantially increasing Forest/Tree Cover through Afforestation and Social Forestry.
    ● Taking steps to meet requirements of fuel, wood, fodder, minor forest produces, soil and timber of Rural and Tribal Population.
    ● Increasing the productivity of Forests to meet National Needs.
    ● Encouraging efficient utilization of Forest Produce and Optimum Use of Wood(Timber)
    ● Generation of Work Opportunities, the involvement of Women.
    LIMITATIONS OF THE POLICY:
    ● No Official definition of the term “Forest”, which may comprise a “Self-Sown” area which supports a Community of Creatures dependent on the plants and Interdependent on each other.
    ● “Natural Heritage” should include all green cover like grasslands, wetlands, etc.
    ● There is no provision for the protection of degraded land which can be a good land resource.
    ● It does not focus on methods to check soil-erosion, denudation, Floods, etc.

    Incorrect

    Answer: A
    Explanation:
    Only statement 1 is correct:
    The first National Forest Policy in independent India took effect in 1952, with a second edition in 1988. The main theme of this policy of 1988 is protection, conservation, and development of forests. Salient Features and Goals of National Forest Policy-
    ● Maintenance of environmental stability through preservation and restoration of ecological balance.
    ● Conservation of Natural Heritage(existing).
    ● Checking Soil Erosion and Denudation in catchment areas of rivers, lakes, and reservoirs;
    ● Checking extension of sand dunes in desert areas of Rajasthan and along coastal tracts;
    ● Substantially increasing Forest/Tree Cover through Afforestation and Social Forestry.
    ● Taking steps to meet requirements of fuel, wood, fodder, minor forest produces, soil and timber of Rural and Tribal Population.
    ● Increasing the productivity of Forests to meet National Needs.
    ● Encouraging efficient utilization of Forest Produce and Optimum Use of Wood(Timber)
    ● Generation of Work Opportunities, the involvement of Women.
    LIMITATIONS OF THE POLICY:
    ● No Official definition of the term “Forest”, which may comprise a “Self-Sown” area which supports a Community of Creatures dependent on the plants and Interdependent on each other.
    ● “Natural Heritage” should include all green cover like grasslands, wetlands, etc.
    ● There is no provision for the protection of degraded land which can be a good land resource.
    ● It does not focus on methods to check soil-erosion, denudation, Floods, etc.

  5. Question 5 of 5
    5. Question
    2 points

    5. Consider the following statements about the ‘Indian Black Turtle’:
    1. They are commonly found in the marine habitats of the Eastern Coast of India.
    2. They are currently listed as “Vulnerable” in the International Union for Conservation of Nature (IUCN) Red List.
    Which of the statements given above is/are correct?

    Correct

    Answer: B
    Explanation:
    ● Statement 1 is incorrect: Indian black turtles are freshwater turtles and are primarily found in freshwater habitats such as rivers, lakes, and ponds. They are not commonly found in marine habitats such as oceans and seas.
    ● Statement 2 is correct: The Indian black turtle is currently listed as “Vulnerable” on the International Union for Conservation of Nature (IUCN) Red List due to habitat loss, pollution, and overexploitation for their meat and eggs. Conservation efforts are underway to protect these turtles and their habitats, including the establishment of protected areas and outreach programs to raise awareness about their conservation.

    Incorrect

    Answer: B
    Explanation:
    ● Statement 1 is incorrect: Indian black turtles are freshwater turtles and are primarily found in freshwater habitats such as rivers, lakes, and ponds. They are not commonly found in marine habitats such as oceans and seas.
    ● Statement 2 is correct: The Indian black turtle is currently listed as “Vulnerable” on the International Union for Conservation of Nature (IUCN) Red List due to habitat loss, pollution, and overexploitation for their meat and eggs. Conservation efforts are underway to protect these turtles and their habitats, including the establishment of protected areas and outreach programs to raise awareness about their conservation.

window.wpAdvQuizInitList = window.wpAdvQuizInitList || []; window.wpAdvQuizInitList.push({ id: '#wpAdvQuiz_8', init: { quizId: 8, mode: 1, globalPoints: 10, timelimit: 0, resultsGrade: [0], bo: 0, qpp: 0, catPoints: [10], formPos: 0, lbn: "Finish quiz", json: {"36":{"type":"single","id":36,"catId":0,"points":2,"correct":[0,0,0,1]},"37":{"type":"single","id":37,"catId":0,"points":2,"correct":[0,1,0,0]},"38":{"type":"single","id":38,"catId":0,"points":2,"correct":[1,0,0,0]},"39":{"type":"single","id":39,"catId":0,"points":2,"correct":[1,0,0,0]},"40":{"type":"single","id":40,"catId":0,"points":2,"correct":[0,1,0,0]}} } });




Day-521 | Daily MCQs | UPSC Prelims | CURRENT DEVELOPMENTS

Day-521

Time limit: 0

Quiz-summary

0 of 5 questions completed

Questions:

  1. 1
  2. 2
  3. 3
  4. 4
  5. 5

Information

To attempt the Quiz, simply click on START Button.

You have already completed the quiz before. Hence you can not start it again.

Quiz is loading...

You must sign in or sign up to start the quiz.

You have to finish following quiz, to start this quiz:

Results

0 of 5 questions answered correctly

Your time:

Time has elapsed

You have reached 0 of 0 points, (0)

Average score
 
 
Your score
 
 

Categories

  1. Not categorized 0%
Your result has been entered into leaderboard
Loading
captcha
  1. 1
  2. 2
  3. 3
  4. 4
  5. 5
  1. Answered
  2. Review
  1. Question 1 of 5
    1. Question
    2 points

    1. ‘The gas is a poisonous gas that can cause serious health problems when inhaled. It is a colorless and odorless gas. It is highly flammable and can ignite at low concentrations. It is also explosive in high concentrations.It is slightly lighter than air and can accumulate in enclosed spaces such as garages or basements. When it is inhaled, it enters the bloodstream and reduces the amount of oxygen that can be carried by the red blood cells.’
    Which of the following gases is being described in the above paragraph?

    Correct

    Answer: D
    Explanation:
    ● Carbon monoxide (CO) is a colorless, odorless gas that is toxic to humans and animals when inhaled. It is produced by incomplete combustion of carbon-containing fuels, such as gasoline, natural gas, wood, and coal. It is highly flammable and can ignite at low concentrations. It is also explosive in high concentrations.It is slightly lighter than air and can accumulate in enclosed spaces such as garages or basements. CO is dangerous because it binds with hemoglobin in the blood more readily than oxygen, reducing the amount of oxygen that can be carried to vital organs and tissues. This can lead to symptoms ranging from mild headache and nausea to confusion, loss of consciousness, and even death.

    Incorrect

    Answer: D
    Explanation:
    ● Carbon monoxide (CO) is a colorless, odorless gas that is toxic to humans and animals when inhaled. It is produced by incomplete combustion of carbon-containing fuels, such as gasoline, natural gas, wood, and coal. It is highly flammable and can ignite at low concentrations. It is also explosive in high concentrations.It is slightly lighter than air and can accumulate in enclosed spaces such as garages or basements. CO is dangerous because it binds with hemoglobin in the blood more readily than oxygen, reducing the amount of oxygen that can be carried to vital organs and tissues. This can lead to symptoms ranging from mild headache and nausea to confusion, loss of consciousness, and even death.

  2. Question 2 of 5
    2. Question
    2 points

    2. Consider the following statements about the National Platform for Disaster Risk Reduction (NPDRR):
    1. It is a multi-stakeholder national platform established in 2005 in the aftermath of the Indian Ocean tsunami.
    2. It is chaired by the Prime Minister of India.
    3. One of the objectives of this forum is to review the National Disaster Management Policy.
    How many of the statements given above is/are correct?

    Correct

    Answer: B
    Explanation:
    ● The National Platform for Disaster Risk Reduction (NPDRR) is a forum established by the Government of India to promote coordination and collaboration among various stakeholders in disaster risk reduction. The platform brings together representatives from government, non-governmental organizations, academic institutions, private sector, media, and civil society to exchange ideas, share best practices, and develop strategies to mitigate the impact of disasters.
    ● The NPDRR was established in 2005 in the aftermath of the Indian Ocean tsunami, which highlighted the need for a coordinated approach to disaster risk reduction. The platform is chaired by the Union Home Minister and meets every two years to review the progress made in disaster risk reduction and to identify gaps and challenges that need to be addressed. Hence, statement 1 is correct and 2 is incorrect.
    FUNCTIONS:
    ● To review the progress made in the field of disaster management from time to time.
    ● To appreciate the extent and manner in which the Disaster Management Policy has been implemented by the Central and State Governments, and other concerned agencies, and to give appropriate advice in the matter.
    ● To advise on coordination between the Central and State Governments/UT Administrations, local self-governments and civil society organizations for Disaster Risk Reduction.
    ● To advice suo-moto or on a reference made by the Central Government or any other State Government or a Union territory Administration on any question pertaining to disaster management.
    ● To review the National Disaster Management Policy. Hence, statement 3 is correct.

    Incorrect

    Answer: B
    Explanation:
    ● The National Platform for Disaster Risk Reduction (NPDRR) is a forum established by the Government of India to promote coordination and collaboration among various stakeholders in disaster risk reduction. The platform brings together representatives from government, non-governmental organizations, academic institutions, private sector, media, and civil society to exchange ideas, share best practices, and develop strategies to mitigate the impact of disasters.
    ● The NPDRR was established in 2005 in the aftermath of the Indian Ocean tsunami, which highlighted the need for a coordinated approach to disaster risk reduction. The platform is chaired by the Union Home Minister and meets every two years to review the progress made in disaster risk reduction and to identify gaps and challenges that need to be addressed. Hence, statement 1 is correct and 2 is incorrect.
    FUNCTIONS:
    ● To review the progress made in the field of disaster management from time to time.
    ● To appreciate the extent and manner in which the Disaster Management Policy has been implemented by the Central and State Governments, and other concerned agencies, and to give appropriate advice in the matter.
    ● To advise on coordination between the Central and State Governments/UT Administrations, local self-governments and civil society organizations for Disaster Risk Reduction.
    ● To advice suo-moto or on a reference made by the Central Government or any other State Government or a Union territory Administration on any question pertaining to disaster management.
    ● To review the National Disaster Management Policy. Hence, statement 3 is correct.

  3. Question 3 of 5
    3. Question
    2 points

    3. ‘Mission 2070: A Green New Deal for a Net-Zero India’ report, is released by which of the following?

    Correct

    Answer: B
    Explanation:
    ● The World Economic Forum (WEF) published its report titled “Mission 2070: A Green New Deal for a Net-Zero India” to provide a roadmap for India’s energy transition. The report was published in the backdrop of India’s target to achieve net zero emissions by 2070. This target was committed by the Prime Minister at the COP26 global climate summit in Glasgow.
    Key Findings of the report:
    ● Report highlights that India’s transition towards a green economy could contribute about $1 trillion in economic impact by 2030. It will create more than 50 million jobs.
    ● The economic impact would increase to $15 trillion by 2070.
    ● Report notes that India’s commitments are a critical foundation for achieving the 1.5-degree Celsius global warming target.
    ● India is on the frontline of climate change. More Indians are exposed to negative effects of climate change and extreme weather events as compared to other nationalities.

    Incorrect

    Answer: B
    Explanation:
    ● The World Economic Forum (WEF) published its report titled “Mission 2070: A Green New Deal for a Net-Zero India” to provide a roadmap for India’s energy transition. The report was published in the backdrop of India’s target to achieve net zero emissions by 2070. This target was committed by the Prime Minister at the COP26 global climate summit in Glasgow.
    Key Findings of the report:
    ● Report highlights that India’s transition towards a green economy could contribute about $1 trillion in economic impact by 2030. It will create more than 50 million jobs.
    ● The economic impact would increase to $15 trillion by 2070.
    ● Report notes that India’s commitments are a critical foundation for achieving the 1.5-degree Celsius global warming target.
    ● India is on the frontline of climate change. More Indians are exposed to negative effects of climate change and extreme weather events as compared to other nationalities.

  4. Question 4 of 5
    4. Question
    2 points

    4. Which one of the following is an example of amensalism in an ecosystem?

    Correct

    Answer: B
    Explanation:
    ● Amensalism is a type of interaction between organisms where one species is negatively affected while the other is unaffected. In this case, a plant releasing chemicals that inhibit the growth of nearby plants is an example of amensalism, as the plant is negatively affecting the growth of other plants without being affected itself. A spider capturing and eating a fly, a bee collecting nectar from a flower, and a predator avoiding a prey species that is distasteful are examples of predation, mutualism, and prey defense, respectively, and not amensalism.

    Incorrect

    Answer: B
    Explanation:
    ● Amensalism is a type of interaction between organisms where one species is negatively affected while the other is unaffected. In this case, a plant releasing chemicals that inhibit the growth of nearby plants is an example of amensalism, as the plant is negatively affecting the growth of other plants without being affected itself. A spider capturing and eating a fly, a bee collecting nectar from a flower, and a predator avoiding a prey species that is distasteful are examples of predation, mutualism, and prey defense, respectively, and not amensalism.

  5. Question 5 of 5
    5. Question
    2 points

    5. Which of the following statements is correct about sea slugs?

    Correct

    Answer: C
    Explanation:
    ● Sea slugs are a diverse group of marine gastropod mollusks that are known for their colorful and intricate body patterns and forms. They are soft-bodied and lack a protective shell, which makes them vulnerable to predation, but they have evolved a range of defensive mechanisms such as toxic secretions, camouflage, and mimicry to deter predators.
    ● Sea slugs are found in a variety of marine habitats, from shallow coral reefs to deep-sea environments, and they play important ecological roles as herbivores, predators, and decomposers.
    ● Sea slugs, like other organisms, have the ability to acquire genetic material from other organisms through a process known as horizontal gene transfer. In the case of sea slugs, they have been found to incorporate genes from their algal food sources into their own genome. This allows sea slugs to produce chlorophyll and perform photosynthesis, a process that is typically only found in plants and algae. This unique ability is known as kleptoplasty and is thought to have evolved as a way for sea slugs to survive periods of food scarcity. By incorporating functional genes from their food, they can continue to produce energy through photosynthesis even when food is scarce. The ability to steal genes from other organisms is just one of the many fascinating adaptations that make sea slugs such interesting creatures.

    Incorrect

    Answer: C
    Explanation:
    ● Sea slugs are a diverse group of marine gastropod mollusks that are known for their colorful and intricate body patterns and forms. They are soft-bodied and lack a protective shell, which makes them vulnerable to predation, but they have evolved a range of defensive mechanisms such as toxic secretions, camouflage, and mimicry to deter predators.
    ● Sea slugs are found in a variety of marine habitats, from shallow coral reefs to deep-sea environments, and they play important ecological roles as herbivores, predators, and decomposers.
    ● Sea slugs, like other organisms, have the ability to acquire genetic material from other organisms through a process known as horizontal gene transfer. In the case of sea slugs, they have been found to incorporate genes from their algal food sources into their own genome. This allows sea slugs to produce chlorophyll and perform photosynthesis, a process that is typically only found in plants and algae. This unique ability is known as kleptoplasty and is thought to have evolved as a way for sea slugs to survive periods of food scarcity. By incorporating functional genes from their food, they can continue to produce energy through photosynthesis even when food is scarce. The ability to steal genes from other organisms is just one of the many fascinating adaptations that make sea slugs such interesting creatures.

window.wpAdvQuizInitList = window.wpAdvQuizInitList || []; window.wpAdvQuizInitList.push({ id: '#wpAdvQuiz_9', init: { quizId: 9, mode: 1, globalPoints: 10, timelimit: 0, resultsGrade: [0], bo: 0, qpp: 0, catPoints: [10], formPos: 0, lbn: "Finish quiz", json: {"41":{"type":"single","id":41,"catId":0,"points":2,"correct":[0,0,0,1]},"42":{"type":"single","id":42,"catId":0,"points":2,"correct":[0,1,0,0]},"43":{"type":"single","id":43,"catId":0,"points":2,"correct":[0,1,0,0]},"44":{"type":"single","id":44,"catId":0,"points":2,"correct":[0,1,0,0]},"45":{"type":"single","id":45,"catId":0,"points":2,"correct":[0,0,1,0]}} } });




Day-520 | Daily MCQs | UPSC Prelims | HISTORY

Day-520

Time limit: 0

Quiz-summary

0 of 5 questions completed

Questions:

  1. 1
  2. 2
  3. 3
  4. 4
  5. 5

Information

To attempt the Quiz, simply click on START Button.

You have already completed the quiz before. Hence you can not start it again.

Quiz is loading...

You must sign in or sign up to start the quiz.

You have to finish following quiz, to start this quiz:

Results

0 of 5 questions answered correctly

Your time:

Time has elapsed

You have reached 0 of 0 points, (0)

Average score
 
 
Your score
 
 

Categories

  1. Not categorized 0%
Your result has been entered into leaderboard
Loading
captcha
  1. 1
  2. 2
  3. 3
  4. 4
  5. 5
  1. Answered
  2. Review
  1. Question 1 of 5
    1. Question
    2 points

    1. With reference to the Indian Schools of Philosophy, which of the following statements is correct?

    Correct

    Answer: A
    Explanation:
    ● Cārvāka: It is the first and foremost nāstika darśana. Tradition names it as lokāyata, meaning, ‘which appeals to the mass’. It is an extreme nastika darśana which believes only one means of valid knowledge, i.e., direct perception or pratyakṣa and all other sources of knowledge are not trustworthy or misleading. This philosophy discards the fifth fundamental element of the universe, i.e., the ākāśa (the space) as it is not perceived to us. So, only four fundamental elements are there, viz., earth, water, fire and air, which our sense organs can perceive.
    ● The Jaina philosophy revolves around two main doctrines, i.e., Anekāntavāda and Syādvāda. Both are extremely connected doctrines. According to anekāntavāda, every being has numerous properties. The permanent property that constitutes the nature of a thing is called attribute (guna). The accidental property is called mode (paryāya). According to syādvāda, our knowledge is partial and relative because passion, anger, greed, etc., obstruct our knowledge. But, we deem our partial and relative knowledge as complete and absolute. Only a liberated soul can know the reality in totality.
    ● According to Sāṅkhya there can be only three valid means (pramāṇas) of acquiring the right cognition which are—direct sense perception (Dṛṣṭa/Pratyakṣa), inference (Anumāna) and verbal testimony (Āptavacana). Sāṅkhya doesn’t accept Upamāna or analogy to be a valid source of knowledge.
    ● The Vaiśeṣika system believes in the reality of the world and recognises seven ‘Padārthas’ or categories, which are: substance (dravya), quality (guṇa), action (karma), generality (sāmānya), particularity (viśeṣa), relation of inherence (samavāya) and non-existence (abhāva).

    Incorrect

    Answer: A
    Explanation:
    ● Cārvāka: It is the first and foremost nāstika darśana. Tradition names it as lokāyata, meaning, ‘which appeals to the mass’. It is an extreme nastika darśana which believes only one means of valid knowledge, i.e., direct perception or pratyakṣa and all other sources of knowledge are not trustworthy or misleading. This philosophy discards the fifth fundamental element of the universe, i.e., the ākāśa (the space) as it is not perceived to us. So, only four fundamental elements are there, viz., earth, water, fire and air, which our sense organs can perceive.
    ● The Jaina philosophy revolves around two main doctrines, i.e., Anekāntavāda and Syādvāda. Both are extremely connected doctrines. According to anekāntavāda, every being has numerous properties. The permanent property that constitutes the nature of a thing is called attribute (guna). The accidental property is called mode (paryāya). According to syādvāda, our knowledge is partial and relative because passion, anger, greed, etc., obstruct our knowledge. But, we deem our partial and relative knowledge as complete and absolute. Only a liberated soul can know the reality in totality.
    ● According to Sāṅkhya there can be only three valid means (pramāṇas) of acquiring the right cognition which are—direct sense perception (Dṛṣṭa/Pratyakṣa), inference (Anumāna) and verbal testimony (Āptavacana). Sāṅkhya doesn’t accept Upamāna or analogy to be a valid source of knowledge.
    ● The Vaiśeṣika system believes in the reality of the world and recognises seven ‘Padārthas’ or categories, which are: substance (dravya), quality (guṇa), action (karma), generality (sāmānya), particularity (viśeṣa), relation of inherence (samavāya) and non-existence (abhāva).

  2. Question 2 of 5
    2. Question
    2 points

    2. With reference to the medieval history, consider the following pairs:
    Terms – Associated with
    1. Mirasdars – Proprietary peasants
    2. Upari – Temporary peasants
    3. Balutedars – Village artisans
    4. Mokasadars – Military tenures
    How many pairs given above are correctly matched?

    Correct

    Answer: D
    Explanation:
    The rural society of medieval Deccan consisted of primarily five groups:
    ● The large landholders who held administrative positions in the village. They were primarily the deshmukhs and deshpandes.
    ● Village officers such as the headman or patil and muqaddam, accountant or kulkarni and so on.
    ● Proprietary peasants called mirasdars or thalkari or thalvaik.
    ● Temporary peasants called uparis.
    ● Village servants and artisans collectively called balutedars.
    Mokasa, Jagir Saranjam Tenures: These were essentially military tenures. Though they were in principle temporary, in course of time, they became hereditary. Civil functions were often attached to these tenures. These military tenures were common in the seventeenth century in the Sultanate of the Adil Shahis of Bijapur in the north Karnataka region. In case, the mokasadars failed in performing their duties, they were transferred or replaced or sometimes their mokasas were confiscated.

    Incorrect

    Answer: D
    Explanation:
    The rural society of medieval Deccan consisted of primarily five groups:
    ● The large landholders who held administrative positions in the village. They were primarily the deshmukhs and deshpandes.
    ● Village officers such as the headman or patil and muqaddam, accountant or kulkarni and so on.
    ● Proprietary peasants called mirasdars or thalkari or thalvaik.
    ● Temporary peasants called uparis.
    ● Village servants and artisans collectively called balutedars.
    Mokasa, Jagir Saranjam Tenures: These were essentially military tenures. Though they were in principle temporary, in course of time, they became hereditary. Civil functions were often attached to these tenures. These military tenures were common in the seventeenth century in the Sultanate of the Adil Shahis of Bijapur in the north Karnataka region. In case, the mokasadars failed in performing their duties, they were transferred or replaced or sometimes their mokasas were confiscated.

  3. Question 3 of 5
    3. Question
    2 points

    3. Arrange the following Buddhist sites from north to south:
    1. Sanchi
    2. Junnar
    3. Karle
    4. Bharhut
    Select the correct answer using the code given below:

    Correct

    Answer: B
    Explanation:
    The correct order/sequence is: Bharhut, Sanchi, Junnar and Karle.

    Incorrect

    Answer: B
    Explanation:
    The correct order/sequence is: Bharhut, Sanchi, Junnar and Karle.

  4. Question 4 of 5
    4. Question
    2 points

    4. Which of the following statements about the ‘Kudumiyamalai’ inscription is correct?

    Correct

    Answer: C
    Explanation:
    ● Inscriptions reflect the history of languages and literature and a few refer to the performing arts. For instance, the 7th century Kudumiyamalai inscription gives the musical notes used in seven classical ragas.
    Additional Information:
    ● The earliest stone inscriptions recording land grants with tax exemptions are Satavahana and Kshatrapa epigraphs found at Nashik.
    ● The mid-4th century Pallava and Shalankayana grants are the earliest surviving copper plate grants.
    ● One of the oldest copper plate grants from north India is the late 4th century CE Kalachala grant of king Ishvararata.

    Incorrect

    Answer: C
    Explanation:
    ● Inscriptions reflect the history of languages and literature and a few refer to the performing arts. For instance, the 7th century Kudumiyamalai inscription gives the musical notes used in seven classical ragas.
    Additional Information:
    ● The earliest stone inscriptions recording land grants with tax exemptions are Satavahana and Kshatrapa epigraphs found at Nashik.
    ● The mid-4th century Pallava and Shalankayana grants are the earliest surviving copper plate grants.
    ● One of the oldest copper plate grants from north India is the late 4th century CE Kalachala grant of king Ishvararata.

  5. Question 5 of 5
    5. Question
    2 points

    5. The notion of Kingship projected by the Kushana rulers is best evidenced through which of the following?

    Correct

    Answer: D
    Explanation:
    ● The Kushana kings used the title devaputra. Historians have described them as exalting the position of the king to the extent of projecting him as divine, an idea quite common in other ancient empires.
    ● The Rabatak inscription also throws important light on the Kushana conception of kingship. Kanishka claims to have obtained kingship through the agency of a number of mostly Zoroastrian deities, headed by the goddess Nana. There is also mention of images of the reigning king and his predecessors in connection with the temple that had been built.

    Incorrect

    Answer: D
    Explanation:
    ● The Kushana kings used the title devaputra. Historians have described them as exalting the position of the king to the extent of projecting him as divine, an idea quite common in other ancient empires.
    ● The Rabatak inscription also throws important light on the Kushana conception of kingship. Kanishka claims to have obtained kingship through the agency of a number of mostly Zoroastrian deities, headed by the goddess Nana. There is also mention of images of the reigning king and his predecessors in connection with the temple that had been built.

window.wpAdvQuizInitList = window.wpAdvQuizInitList || []; window.wpAdvQuizInitList.push({ id: '#wpAdvQuiz_10', init: { quizId: 10, mode: 1, globalPoints: 10, timelimit: 0, resultsGrade: [0], bo: 0, qpp: 0, catPoints: [10], formPos: 0, lbn: "Finish quiz", json: {"46":{"type":"single","id":46,"catId":0,"points":2,"correct":[1,0,0,0]},"47":{"type":"single","id":47,"catId":0,"points":2,"correct":[0,0,0,1]},"48":{"type":"single","id":48,"catId":0,"points":2,"correct":[0,1,0,0]},"49":{"type":"single","id":49,"catId":0,"points":2,"correct":[0,0,1,0]},"50":{"type":"single","id":50,"catId":0,"points":2,"correct":[0,0,0,1]}} } });




Day-519 | Daily MCQs | UPSC Prelims | HISTORY

Day-519

Time limit: 0

Quiz-summary

0 of 5 questions completed

Questions:

  1. 1
  2. 2
  3. 3
  4. 4
  5. 5

Information

To attempt the Quiz, simply click on START Button.

You have already completed the quiz before. Hence you can not start it again.

Quiz is loading...

You must sign in or sign up to start the quiz.

You have to finish following quiz, to start this quiz:

Results

0 of 5 questions answered correctly

Your time:

Time has elapsed

You have reached 0 of 0 points, (0)

Average score
 
 
Your score
 
 

Categories

  1. Not categorized 0%
Your result has been entered into leaderboard
Loading
captcha
  1. 1
  2. 2
  3. 3
  4. 4
  5. 5
  1. Answered
  2. Review
  1. Question 1 of 5
    1. Question
    2 points

    1. With reference to Shivaji’s administration, which one of the following statements is correct?

    Correct

    Answer: D
    Explanation:
    When one comes to examine Shivaji’s administration, one can easily detect a few broad principles which he shrewdly brought into force as he proceeded building up his Swarajya. They are:-
    ● The country was to be defended against enemies by means of well garrisoned forts;
    ● All services were to be paid in cash and not by grants of land; hence, statement a is incorrect.
    ● Servants were employed on the principle of merit and not heredity; hence, statement b is incorrect.
    ● Revenue was not to be collected through Zamindars or middlemen but through a well-supervised government agency; hence, statement d is correct.
    ● The system of farming lands was abolished; in other words, land revenue was never auctioned; hence, statement c is incorrect.
    ● All government work was divided among well defined separate departments;
    ● Equal opportunity was offered in the public service to all castes;
    ● Expenditure was so budgeted as to lay by some surplus every year.
    Additional Information:
    ● The whole dominion of Shivaji was fluctuating and fluid in its area, as he was all along engaged in building up and extending his kingdom. He divided it into three main provinces, the northern division from Poona to Salher, being under his Peshwa Moropant Pingle and including north Konkan also. Southern Konkan upto north Kanara was in the viceroyalty of Anāji Datto, while the southern Desh districts roughly from Satara to Dharwar and Kopbal were under Dattäjipant Wagnis.
    ● The newly conquered districts south of the Tunga- bhadra, namely, Arni, Vellore, Jinji, etc. were administered by Ilarji Mahadik, Shivaji’s son-in-law. There were many scattered outlying-districts, where specially trusted and competent agents were appointed from time to time. “The Mughal territory was to be subjected to the levy of the Chauth contribution.

    Incorrect

    Answer: D
    Explanation:
    When one comes to examine Shivaji’s administration, one can easily detect a few broad principles which he shrewdly brought into force as he proceeded building up his Swarajya. They are:-
    ● The country was to be defended against enemies by means of well garrisoned forts;
    ● All services were to be paid in cash and not by grants of land; hence, statement a is incorrect.
    ● Servants were employed on the principle of merit and not heredity; hence, statement b is incorrect.
    ● Revenue was not to be collected through Zamindars or middlemen but through a well-supervised government agency; hence, statement d is correct.
    ● The system of farming lands was abolished; in other words, land revenue was never auctioned; hence, statement c is incorrect.
    ● All government work was divided among well defined separate departments;
    ● Equal opportunity was offered in the public service to all castes;
    ● Expenditure was so budgeted as to lay by some surplus every year.
    Additional Information:
    ● The whole dominion of Shivaji was fluctuating and fluid in its area, as he was all along engaged in building up and extending his kingdom. He divided it into three main provinces, the northern division from Poona to Salher, being under his Peshwa Moropant Pingle and including north Konkan also. Southern Konkan upto north Kanara was in the viceroyalty of Anāji Datto, while the southern Desh districts roughly from Satara to Dharwar and Kopbal were under Dattäjipant Wagnis.
    ● The newly conquered districts south of the Tunga- bhadra, namely, Arni, Vellore, Jinji, etc. were administered by Ilarji Mahadik, Shivaji’s son-in-law. There were many scattered outlying-districts, where specially trusted and competent agents were appointed from time to time. “The Mughal territory was to be subjected to the levy of the Chauth contribution.

  2. Question 2 of 5
    2. Question
    2 points

    2. With reference to the Revolutionary Nationalists, consider the following statements:
    1. The constant suppression of Revolutionaries by the Colonial power and being critical of older leaders led to the growth of ‘Revolt Groups’ in Bengal.
    2. Gopinath Saha’s attempt to assassinate Charles Tegart, the Police Commissioner of Calcutta, was one of the famous actions by the Revolt Group.
    Which of the statements given above is/are correct?

    Correct

    Answer: A
    Explanation:
    In Bengal, the Revolutionary Nationalists began reorganization after 1922. They resumed large-scale propaganda in the press and developed their underground activities.

    ● The Government now took alarm and started large scale repression. It arrested a large number of revolutionary leaders and activists under a newly promulgated ordinance. Moreover, a large number of Congressmen, including Subhas Bose, suspected of being sympathetic to the revolutionaries were also arrested. Nearly all the major leaders being in jail, revolutionary activity suffered a severe setback.
    ● Revolutionary activity also suffered because of factional and personal quarrels within the ranks of the old revolutionary leaders. Quarrels on the basis of Jugantar vs. Anushilan were endemic. However, after their release after 1926 many of the younger revolutionaries, critical of the older leaders, began to organize themselves into a large number of new groups which came to be known as Revolt Groups. Hence, statement 1 is correct.
    ● Gopinath Saha’s attempt in January 1924 to assassinate Charles Tegart, the hated Police Commissioner of Calcutta, although was a revolutionary act, but was part of individual heroic action and not part of any Revolt Group. The most famous actions among the Revolt Group was the Chittagong Armoury Raid led by Surya Sen. Hence, statement 2 is incorrect.

    Incorrect

    Answer: A
    Explanation:
    In Bengal, the Revolutionary Nationalists began reorganization after 1922. They resumed large-scale propaganda in the press and developed their underground activities.

    ● The Government now took alarm and started large scale repression. It arrested a large number of revolutionary leaders and activists under a newly promulgated ordinance. Moreover, a large number of Congressmen, including Subhas Bose, suspected of being sympathetic to the revolutionaries were also arrested. Nearly all the major leaders being in jail, revolutionary activity suffered a severe setback.
    ● Revolutionary activity also suffered because of factional and personal quarrels within the ranks of the old revolutionary leaders. Quarrels on the basis of Jugantar vs. Anushilan were endemic. However, after their release after 1926 many of the younger revolutionaries, critical of the older leaders, began to organize themselves into a large number of new groups which came to be known as Revolt Groups. Hence, statement 1 is correct.
    ● Gopinath Saha’s attempt in January 1924 to assassinate Charles Tegart, the hated Police Commissioner of Calcutta, although was a revolutionary act, but was part of individual heroic action and not part of any Revolt Group. The most famous actions among the Revolt Group was the Chittagong Armoury Raid led by Surya Sen. Hence, statement 2 is incorrect.

  3. Question 3 of 5
    3. Question
    2 points

    3. Consider the following personalities:
    1. Bhulabhai Desai
    2. Mohammad Zaman Kiani
    3. C.R. Abhayankar
    4. Satyanand Puri
    How many of the above mentioned personalities are associated with the history of Azad Hind Fauz (Indian National Army)?

    Correct

    Answer: C
    Explanation:
    During the 1940s, the Indian National Army or Azad Hind Fauj, along with the Quit India Movement, emerged as one of the most important symbols of India’s will to fight for independence in the best possible manner, even through violent efforts.
    The rapid advance of the Japanese forces in Southeast Asia uprooting the European colonial powers, such as the British, Dutch and French, led to a completely changed situation when the Indians in these countries as well as the captured Indian soldiers who had fought in the British army began to be mobilized and organized to fight for Indian freedom.
    ● Giani Pritam Singh, Captain Mohan Singh, Rash Behari Bose, Pritam Singh and Satyanand Puri are associated with the first phase of INA.
    Subhas Chandra Bose arrived in Singapore on 2 July 1943 and assumed the command of the INA from Rash Behari. Bose declared in Singapore on 21 October 1943 the formation of the Azad Hind Government.
    ● Bose decided that Burma would be crucial to his strategic military manoeuvre. When the Japanese Field Marshal suggested that the INA should work only as a field propaganda unit, Bose immediately rejected it and demanded that INA brigades should be used as advance fighting units. The Japanese agreed to initially put one division of INA consisting of about 10,000 soldiers into action. Mohammad Zaman Kiani assumed the command of this unit. This division was further divided into three regiments which had been named after Gandhi, Nehru, and Azad signifying oneness with the nationalist movement at home. Out of these the best soldiers were taken out to form a guerrilla unit under Shah Nawaz Khan which would first go into action. The soldiers named this unit ‘Subhas Brigade’.
    The trial of INA officers and soldiers at Delhi’s Red Fort aroused such strong sentiments among the Indians against the British that the INA and its main officers became known in every home in the country.
    ● The Congress decided to defend the prisoners in the Court and assigned the task to a veteran nationalist lawyer, Bhulabhai Desai.
    Note: C.R. Abhayankar is not associated with INA rather with All India States People’s Conference.

    Incorrect

    Answer: C
    Explanation:
    During the 1940s, the Indian National Army or Azad Hind Fauj, along with the Quit India Movement, emerged as one of the most important symbols of India’s will to fight for independence in the best possible manner, even through violent efforts.
    The rapid advance of the Japanese forces in Southeast Asia uprooting the European colonial powers, such as the British, Dutch and French, led to a completely changed situation when the Indians in these countries as well as the captured Indian soldiers who had fought in the British army began to be mobilized and organized to fight for Indian freedom.
    ● Giani Pritam Singh, Captain Mohan Singh, Rash Behari Bose, Pritam Singh and Satyanand Puri are associated with the first phase of INA.
    Subhas Chandra Bose arrived in Singapore on 2 July 1943 and assumed the command of the INA from Rash Behari. Bose declared in Singapore on 21 October 1943 the formation of the Azad Hind Government.
    ● Bose decided that Burma would be crucial to his strategic military manoeuvre. When the Japanese Field Marshal suggested that the INA should work only as a field propaganda unit, Bose immediately rejected it and demanded that INA brigades should be used as advance fighting units. The Japanese agreed to initially put one division of INA consisting of about 10,000 soldiers into action. Mohammad Zaman Kiani assumed the command of this unit. This division was further divided into three regiments which had been named after Gandhi, Nehru, and Azad signifying oneness with the nationalist movement at home. Out of these the best soldiers were taken out to form a guerrilla unit under Shah Nawaz Khan which would first go into action. The soldiers named this unit ‘Subhas Brigade’.
    The trial of INA officers and soldiers at Delhi’s Red Fort aroused such strong sentiments among the Indians against the British that the INA and its main officers became known in every home in the country.
    ● The Congress decided to defend the prisoners in the Court and assigned the task to a veteran nationalist lawyer, Bhulabhai Desai.
    Note: C.R. Abhayankar is not associated with INA rather with All India States People’s Conference.

  4. Question 4 of 5
    4. Question
    2 points

    4. With reference to the history of India, the term ‘dalams’ can be associated with-

    Correct

    Answer: B
    Explanation:
    Among the States where the first Praja Mandals or State People’s Conferences were set up included Hyderabad, Mysore, Baroda, the Kathiawad States, the Deccan States, Jamnagar, Indore and Nawanagar. Among the leaders who emerged through this process, the more important names are those of Balwantrai Mehta, Maniklal Kothari and C.R. Abhayankar.
    The high watermark of the movement in the States was reached in the years 1938-39. Praja Mandals or People’s Association sprung up in many states, and struggles broke out in Rajkot, Travancore, Mysore, Hyderabad, Patiala, Jaipur, Kashmir and the Orissa States.
    ● The term ‘dalams’ is associated with the state of Hyderabad. The Nizam, on 12 June 1947 announced that he would become sovereign after the British left. The movement now took a different form, that of armed resistance. The State Congress set up camps on the State’s borders, and organized raids on custom’s outposts, the police stations and Razakar camps. But inside the State, and especially in the Nalgonda, Warangal and Khammam districts of Telengana, it was the Communists who took the lead in organizing armed resistance. They organized the peasants into dalams, gave the training in using arms, to attack the Razakars.

    Incorrect

    Answer: B
    Explanation:
    Among the States where the first Praja Mandals or State People’s Conferences were set up included Hyderabad, Mysore, Baroda, the Kathiawad States, the Deccan States, Jamnagar, Indore and Nawanagar. Among the leaders who emerged through this process, the more important names are those of Balwantrai Mehta, Maniklal Kothari and C.R. Abhayankar.
    The high watermark of the movement in the States was reached in the years 1938-39. Praja Mandals or People’s Association sprung up in many states, and struggles broke out in Rajkot, Travancore, Mysore, Hyderabad, Patiala, Jaipur, Kashmir and the Orissa States.
    ● The term ‘dalams’ is associated with the state of Hyderabad. The Nizam, on 12 June 1947 announced that he would become sovereign after the British left. The movement now took a different form, that of armed resistance. The State Congress set up camps on the State’s borders, and organized raids on custom’s outposts, the police stations and Razakar camps. But inside the State, and especially in the Nalgonda, Warangal and Khammam districts of Telengana, it was the Communists who took the lead in organizing armed resistance. They organized the peasants into dalams, gave the training in using arms, to attack the Razakars.

  5. Question 5 of 5
    5. Question
    2 points

    5. Consider the following pairs:
    Buddhist Monasteries – Locations
    1. Lingdum Monastery – Sikkim
    2. Mindrolling Monastery – Uttrakhand
    3. Namdroling Monastery – Karnataka
    4. Hemis Monastery – Arunachal Pradesh
    How many pairs given above are correctly matched?

    Correct

    Answer: C
    Explanation:
    ● Lingdum Monastery, also known as Ranka Monastery and Pal Zurmang Kagyud Monastery, is located in Ranka, about 20 kilometres from Gangtok, the capital of Sikkim. It was built in the Tibetan architectural style and is an important site of prayer. The monastery also serves as a training facility for new and novice monks. Lingdum Monastery adheres to the Zurmang Kagyud tradition of Buddhism, which is led by the lineage’s 12th heir, Zurmang Gharwang Rinpoche.
    ● Located in the foothills of the Himalayas in Dehradun in North India, Mindrolling Monastery has expanded steadily to become one of the largest Buddhist centers in existence today. All the great masters of Tibetan Buddhism, and especially all the teachers of the Nyingma lineage consider Mindrolling to be an inspiring example of the practice of the pure and profound Dharma of Vajrayana Buddhism.
    ● The Namdroling Monastery, popularly referred to as ‘The Golden Temple’ is one of the largest Tibetan settlements located in Bylakuppe, about 5 kms from Kushalanagara in Kodagu district, Karnataka in India.
    ● The Hemis Monastery is a Buddhist monastery located 45 kilometers from Leh city, Ladakh, India. It belongs to the Red Hat Sect or Drukpa lineage of Buddhism. The monastery was first established in the 11th century and later reestablished in the 17th century by the Ladakhi King Sengge Namgyal. It is also famous for its 2-day religious ceremony known as the Hemis Festival.

    Incorrect

    Answer: C
    Explanation:
    ● Lingdum Monastery, also known as Ranka Monastery and Pal Zurmang Kagyud Monastery, is located in Ranka, about 20 kilometres from Gangtok, the capital of Sikkim. It was built in the Tibetan architectural style and is an important site of prayer. The monastery also serves as a training facility for new and novice monks. Lingdum Monastery adheres to the Zurmang Kagyud tradition of Buddhism, which is led by the lineage’s 12th heir, Zurmang Gharwang Rinpoche.
    ● Located in the foothills of the Himalayas in Dehradun in North India, Mindrolling Monastery has expanded steadily to become one of the largest Buddhist centers in existence today. All the great masters of Tibetan Buddhism, and especially all the teachers of the Nyingma lineage consider Mindrolling to be an inspiring example of the practice of the pure and profound Dharma of Vajrayana Buddhism.
    ● The Namdroling Monastery, popularly referred to as ‘The Golden Temple’ is one of the largest Tibetan settlements located in Bylakuppe, about 5 kms from Kushalanagara in Kodagu district, Karnataka in India.
    ● The Hemis Monastery is a Buddhist monastery located 45 kilometers from Leh city, Ladakh, India. It belongs to the Red Hat Sect or Drukpa lineage of Buddhism. The monastery was first established in the 11th century and later reestablished in the 17th century by the Ladakhi King Sengge Namgyal. It is also famous for its 2-day religious ceremony known as the Hemis Festival.

window.wpAdvQuizInitList = window.wpAdvQuizInitList || []; window.wpAdvQuizInitList.push({ id: '#wpAdvQuiz_11', init: { quizId: 11, mode: 1, globalPoints: 10, timelimit: 0, resultsGrade: [0], bo: 0, qpp: 0, catPoints: [10], formPos: 0, lbn: "Finish quiz", json: {"51":{"type":"single","id":51,"catId":0,"points":2,"correct":[0,0,0,1]},"52":{"type":"single","id":52,"catId":0,"points":2,"correct":[1,0,0,0]},"53":{"type":"single","id":53,"catId":0,"points":2,"correct":[0,0,1,0]},"54":{"type":"single","id":54,"catId":0,"points":2,"correct":[0,1,0,0]},"55":{"type":"single","id":55,"catId":0,"points":2,"correct":[0,0,1,0]}} } });




Day-518 | Daily MCQs | UPSC Prelims | CURRENT DEVELOPMENTS

Day-518

Time limit: 0

Quiz-summary

0 of 5 questions completed

Questions:

  1. 1
  2. 2
  3. 3
  4. 4
  5. 5

Information

To attempt the Quiz, simply click on START Button.

You have already completed the quiz before. Hence you can not start it again.

Quiz is loading...

You must sign in or sign up to start the quiz.

You have to finish following quiz, to start this quiz:

Results

0 of 5 questions answered correctly

Your time:

Time has elapsed

You have reached 0 of 0 points, (0)

Average score
 
 
Your score
 
 

Categories

  1. Not categorized 0%
Your result has been entered into leaderboard
Loading
captcha
  1. 1
  2. 2
  3. 3
  4. 4
  5. 5
  1. Answered
  2. Review
  1. Question 1 of 5
    1. Question
    2 points

    1. With reference to the High Seas Treaty, which of the following statements is incorrect?

    Correct

    Answer: B
    Explanation:
    ● Nearly 200 countries have agreed to a legally-binding Biodiversity Beyond National Jurisdiction (BBNJ) treaty, also known as the ‘High Seas Treaty’ to protect marine life in international waters, which cover around half of the planet’s surface, but have long been essentially lawless.
    Option(d) is correct:

    How will the High Seas Treaty protect marine life?
    ● The key measure is to put the world’s international waters into protected areas (MPAs) – which will help achieve the global goal of protecting 30% of the world’s oceans by 2030, which was agreed at the 2022 UN biodiversity conference.
    ● Activity can occur in these areas but only “provided it is consistent with the conservation objectives” – meaning it doesn’t damage marine life.
    ● This could mean limiting fishing activities, shipping routes and exploration activities like deep-sea mining. It does not ban deep-sea mining. Hence, the statement in option b) is incorrect.
    ● Environmental groups are seriously concerned about the possible effects of mining, such as disturbing sediments, creating noise pollution and damaging breeding grounds.
    ● Countries will propose areas to be protected, and these will then be voted on by the countries that sign up to the treaty.
    The treaty’s other important points are:
    ● arrangements for sharing marine genetic resources; and
    ● requirements for environmental impact assessments (EIA) for deep sea activities like mining

    Incorrect

    Answer: B
    Explanation:
    ● Nearly 200 countries have agreed to a legally-binding Biodiversity Beyond National Jurisdiction (BBNJ) treaty, also known as the ‘High Seas Treaty’ to protect marine life in international waters, which cover around half of the planet’s surface, but have long been essentially lawless.
    Option(d) is correct:

    How will the High Seas Treaty protect marine life?
    ● The key measure is to put the world’s international waters into protected areas (MPAs) – which will help achieve the global goal of protecting 30% of the world’s oceans by 2030, which was agreed at the 2022 UN biodiversity conference.
    ● Activity can occur in these areas but only “provided it is consistent with the conservation objectives” – meaning it doesn’t damage marine life.
    ● This could mean limiting fishing activities, shipping routes and exploration activities like deep-sea mining. It does not ban deep-sea mining. Hence, the statement in option b) is incorrect.
    ● Environmental groups are seriously concerned about the possible effects of mining, such as disturbing sediments, creating noise pollution and damaging breeding grounds.
    ● Countries will propose areas to be protected, and these will then be voted on by the countries that sign up to the treaty.
    The treaty’s other important points are:
    ● arrangements for sharing marine genetic resources; and
    ● requirements for environmental impact assessments (EIA) for deep sea activities like mining

  2. Question 2 of 5
    2. Question
    2 points

    2. Consider the following statements regarding the Climate Change Performance Index (CCPI) 2023:
    1. CCPI is released annually by the World Economic Forum.
    2. No country has achieved the first three positions in the Index.
    3. India has secured 8th position in the Index, and it is the best position amongst the G-20 countries.
    How many of the above statements are correct?

    Correct

    Answer: B
    Explanation:
    The Climate Change Performance Index 2023 report was released, and India secured 8th position in the index which is 2 positions up from the last edition.
    ● Statement 1 is incorrect: Published annually since 2005, the Climate Change Performance Index (CCPI) tracks countries’ efforts to combat climate change. As an independent monitoring tool it aims to enhance transparency in international climate politics. Germanwatch, the NewClimate Institute and the Climate Action Network publish the index annually.
    ● Statement 2 is correct: No country was strong enough in all index categories to achieve an overall very high rating, thus the top three places i.e 1-3 are vacant.
    ● Statement 3 is correct: India secured 8th position in the index which is 2 positions up from the last edition. With India (8th), the United Kingdom (11th), and Germany (16th), only three G20 countries are among the high performers in CCPI 2023. Thus, India’s rank is the best amongst the G20 countries.

    Incorrect

    Answer: B
    Explanation:
    The Climate Change Performance Index 2023 report was released, and India secured 8th position in the index which is 2 positions up from the last edition.
    ● Statement 1 is incorrect: Published annually since 2005, the Climate Change Performance Index (CCPI) tracks countries’ efforts to combat climate change. As an independent monitoring tool it aims to enhance transparency in international climate politics. Germanwatch, the NewClimate Institute and the Climate Action Network publish the index annually.
    ● Statement 2 is correct: No country was strong enough in all index categories to achieve an overall very high rating, thus the top three places i.e 1-3 are vacant.
    ● Statement 3 is correct: India secured 8th position in the index which is 2 positions up from the last edition. With India (8th), the United Kingdom (11th), and Germany (16th), only three G20 countries are among the high performers in CCPI 2023. Thus, India’s rank is the best amongst the G20 countries.

  3. Question 3 of 5
    3. Question
    2 points

    3. Consider the following statements regarding the Academic Bank of Credits:
    1. It is an initiative proposed under the National Education Policy (NEP) 2020.
    2. It is being implemented by the University Grants Commission (UGC).
    3. It allows multiple entries and multiple exits for the students.
    4. It allows academic mobility of students in the schools and Higher Education Institutions.
    How many of the above statements are correct?

    Correct

    Answer: C
    Explanation:
    The University Grants Commission (UGC) has asked all Higher Education Institutions to immediately register online the Academic Bank of Credits (ABC) platform and upload the date of student credits earned during or after the 2021-22 academic year.
    ● Statement 1 is correct: As per the National Education Policy (NEP-2020), ABC has been envisaged to facilitate the academic mobility of the students with the freedom to study across HEIs in the country with an appropriate “credit transfer” mechanism across programmes.
    ● Statement 2 is correct: The University Grants Commission has amended the Academic Bank of Credit (ABC) regulation to bring all higher education institutions under the same platform.
    ● Statement 3 is correct: It allows multiple entry, multiple exit for students. By multiple entry, multiple exit for students coming into action, a student will get the option to drop their course and resume it at a later stage as and when they desire or deem it worth pursuing. While dropping out is perceived with a negative notion in our society, implementing the Multiple Entry and Exit System(MEES) eases the burden on a student who has to drop out because of financial circumstances or other reasons.
    ● Statement 4 is incorrect: It facilitate the academic mobility of students with the freedom to study across the Higher Education Institutions in the country with an appropriate “credit transfer” mechanism from one programme to another, leading to attain a Degree/ Diploma/PG-diploma, etc.,

    Incorrect

    Answer: C
    Explanation:
    The University Grants Commission (UGC) has asked all Higher Education Institutions to immediately register online the Academic Bank of Credits (ABC) platform and upload the date of student credits earned during or after the 2021-22 academic year.
    ● Statement 1 is correct: As per the National Education Policy (NEP-2020), ABC has been envisaged to facilitate the academic mobility of the students with the freedom to study across HEIs in the country with an appropriate “credit transfer” mechanism across programmes.
    ● Statement 2 is correct: The University Grants Commission has amended the Academic Bank of Credit (ABC) regulation to bring all higher education institutions under the same platform.
    ● Statement 3 is correct: It allows multiple entry, multiple exit for students. By multiple entry, multiple exit for students coming into action, a student will get the option to drop their course and resume it at a later stage as and when they desire or deem it worth pursuing. While dropping out is perceived with a negative notion in our society, implementing the Multiple Entry and Exit System(MEES) eases the burden on a student who has to drop out because of financial circumstances or other reasons.
    ● Statement 4 is incorrect: It facilitate the academic mobility of students with the freedom to study across the Higher Education Institutions in the country with an appropriate “credit transfer” mechanism from one programme to another, leading to attain a Degree/ Diploma/PG-diploma, etc.,

  4. Question 4 of 5
    4. Question
    2 points

    4. Consider the following statements regarding the Crypto-Asset Reporting Framework(CARF):
    1. It has been launched by the World Economic Framework.
    2. It allows exchange of information regarding digital assets with other countries.
    Which of the statements given above is/are correct?

    Correct

    Answer: B
    Explanation:
    In August 2022, the OECD approved the Crypto-Asset Reporting Framework (CARF) which provides for the reporting of tax information on transactions in Crypto-Assets in a standardised manner.
    ● Statement 1 is incorrect: With a view to increase transparency among nations, the OECD (Organisation for Economic Co-operation and Development) has developed the Crypto Asset Reporting Framework (CARF).
    ● Statement 2 is correct: The Common Reporting Standard (CRS) required jurisdictions to obtain information from financial institutions and banks and exchange such information with other jurisdictions. The CARF is a step forward in this direction as crypto assets do not automatically fall in the realm of CRS that deal with traditional financial assets and fiat currencies. With CARF, the reporting scope has been expanded to include digital assets and consequently have visibility over intermediaries, exchanges and e-wallet service providers.

    Incorrect

    Answer: B
    Explanation:
    In August 2022, the OECD approved the Crypto-Asset Reporting Framework (CARF) which provides for the reporting of tax information on transactions in Crypto-Assets in a standardised manner.
    ● Statement 1 is incorrect: With a view to increase transparency among nations, the OECD (Organisation for Economic Co-operation and Development) has developed the Crypto Asset Reporting Framework (CARF).
    ● Statement 2 is correct: The Common Reporting Standard (CRS) required jurisdictions to obtain information from financial institutions and banks and exchange such information with other jurisdictions. The CARF is a step forward in this direction as crypto assets do not automatically fall in the realm of CRS that deal with traditional financial assets and fiat currencies. With CARF, the reporting scope has been expanded to include digital assets and consequently have visibility over intermediaries, exchanges and e-wallet service providers.

  5. Question 5 of 5
    5. Question
    2 points

    5. Which of the following statements about ‘Sammed Shikharji’, recently seen in the news, is/are correct?
    1. According to Jaina tradition, it is believed that Mahavira attained kevalajnana (omniscience) at this site.
    2. According to Jaina beliefs, it is regarded as one of the ‘Panch Tirtha’ of Jainism.
    Select the correct answer using the code given below:

    Correct

    Answer: B
    Explanation:
    The Jharkhand government’s decision to turn Shri Sammed Shikharji, the holiest pilgrimage site of Jains into a tourist spot, was not welcomed by the Jain community. The reason why Sammed Shikharji is so important to the Jain community is that 20 out of the 24 Tirthankaras- original preachers of Jainism and many other Jain ascetics attained moksha through meditation there (Sammed Shikhar- peak of concentration). Jains today believe that carrying out this Tirth would help them in the deliverance from the life of sin and consequences.
    ● Although many Jain Tirthankaras attained moksha at this place, but it is not associated with Mahavira, the 24th Tirthankara. He is supposed to have attained kevalajnana (infinite knowledge, omniscience) outside the town of Jrimbhikagrama, on the banks of the Rijupalika river, in the field of a householder named Samaga, located in present day Jamui district of Bihar. Hence, statement 1 is incorrect.
    ● According to beliefs of the Jain community, Shikharji is ranked with Ashtapad, Girnar, Dilwara Temples of Mount Abu and Shatrunjaya as the ‘Pancha Tirth’ or the five principal pilgrimage shrines. Hence, statement 2 is correct.
    ● Jainism is divided into two sects based on the difference in a few principles of Jainism. These sects are Svetambar (white-clad) and Digambar (sky-clad). There is also a visible difference in the idols of Mahavir of both sects. Nevertheless, the Shikharji is a pilgrim site for both the Svetambar and the Digambar Jains and contains temples with idols belonging to both sects.
    ● The temple is on the highest hill of Jharkhand- Parasnath in the Giridh district of Jharkhand. It is situated at a great elevation of 1,365 m (4,478 ft). It is named after the 23rd Tirthankara- Lord Parshavnath.

    Incorrect

    Answer: B
    Explanation:
    The Jharkhand government’s decision to turn Shri Sammed Shikharji, the holiest pilgrimage site of Jains into a tourist spot, was not welcomed by the Jain community. The reason why Sammed Shikharji is so important to the Jain community is that 20 out of the 24 Tirthankaras- original preachers of Jainism and many other Jain ascetics attained moksha through meditation there (Sammed Shikhar- peak of concentration). Jains today believe that carrying out this Tirth would help them in the deliverance from the life of sin and consequences.
    ● Although many Jain Tirthankaras attained moksha at this place, but it is not associated with Mahavira, the 24th Tirthankara. He is supposed to have attained kevalajnana (infinite knowledge, omniscience) outside the town of Jrimbhikagrama, on the banks of the Rijupalika river, in the field of a householder named Samaga, located in present day Jamui district of Bihar. Hence, statement 1 is incorrect.
    ● According to beliefs of the Jain community, Shikharji is ranked with Ashtapad, Girnar, Dilwara Temples of Mount Abu and Shatrunjaya as the ‘Pancha Tirth’ or the five principal pilgrimage shrines. Hence, statement 2 is correct.
    ● Jainism is divided into two sects based on the difference in a few principles of Jainism. These sects are Svetambar (white-clad) and Digambar (sky-clad). There is also a visible difference in the idols of Mahavir of both sects. Nevertheless, the Shikharji is a pilgrim site for both the Svetambar and the Digambar Jains and contains temples with idols belonging to both sects.
    ● The temple is on the highest hill of Jharkhand- Parasnath in the Giridh district of Jharkhand. It is situated at a great elevation of 1,365 m (4,478 ft). It is named after the 23rd Tirthankara- Lord Parshavnath.

window.wpAdvQuizInitList = window.wpAdvQuizInitList || []; window.wpAdvQuizInitList.push({ id: '#wpAdvQuiz_12', init: { quizId: 12, mode: 1, globalPoints: 10, timelimit: 0, resultsGrade: [0], bo: 0, qpp: 0, catPoints: [10], formPos: 0, lbn: "Finish quiz", json: {"56":{"type":"single","id":56,"catId":0,"points":2,"correct":[0,1,0,0]},"57":{"type":"single","id":57,"catId":0,"points":2,"correct":[0,1,0,0]},"58":{"type":"single","id":58,"catId":0,"points":2,"correct":[0,0,1,0]},"59":{"type":"single","id":59,"catId":0,"points":2,"correct":[0,1,0,0]},"60":{"type":"single","id":60,"catId":0,"points":2,"correct":[0,1,0,0]}} } });




Day-517 | Daily MCQs | UPSC Prelims | ECONOMY

Day-517

Time limit: 0

Quiz-summary

0 of 5 questions completed

Questions:

  1. 1
  2. 2
  3. 3
  4. 4
  5. 5

Information

To attempt the Quiz, simply click on START Button.

You have already completed the quiz before. Hence you can not start it again.

Quiz is loading...

You must sign in or sign up to start the quiz.

You have to finish following quiz, to start this quiz:

Results

0 of 5 questions answered correctly

Your time:

Time has elapsed

You have reached 0 of 0 points, (0)

Average score
 
 
Your score
 
 

Categories

  1. Not categorized 0%
Your result has been entered into leaderboard
Loading
captcha
  1. 1
  2. 2
  3. 3
  4. 4
  5. 5
  1. Answered
  2. Review
  1. Question 1 of 5
    1. Question
    2 points

    1. Consider the following statements about the Central Counterparties (CCPs):
    1. A CCP is authorized by the RBI to operate in India under the Payment and Settlement Systems Act, 2007.
    2. It acts as the intermediary in a market transaction by guaranteeing the terms of trade.
    Which of the above given statements is/are correct?

    Correct

    Answer: C
    Explanation:
    The European Securities and Markets Authority (ESMA), the European Union (EU’s) financial markets regulator and supervisor has derecognised six Indian central counterparties(CCPs).
    ● Statement 1 is correct: A CCP is authorized by the RBI to operate in India under the Payment and Settlement Systems Act, 2007. Some of the important CCPs in India are the Clearing Corporation of India (CCIL), Indian Clearing Corporation Ltd (ICCL), NSE Clearing Ltd (NSCCL), Multi Commodity Exchange Clearing (MCXCCL) among others.
    ● Statement 2 is correct: CCP acts as an intermediary in a market transaction –clearing and settlement – by guaranteeing the terms of trade.

    Incorrect

    Answer: C
    Explanation:
    The European Securities and Markets Authority (ESMA), the European Union (EU’s) financial markets regulator and supervisor has derecognised six Indian central counterparties(CCPs).
    ● Statement 1 is correct: A CCP is authorized by the RBI to operate in India under the Payment and Settlement Systems Act, 2007. Some of the important CCPs in India are the Clearing Corporation of India (CCIL), Indian Clearing Corporation Ltd (ICCL), NSE Clearing Ltd (NSCCL), Multi Commodity Exchange Clearing (MCXCCL) among others.
    ● Statement 2 is correct: CCP acts as an intermediary in a market transaction –clearing and settlement – by guaranteeing the terms of trade.

  2. Question 2 of 5
    2. Question
    2 points

    2. The World Bank is in the process of electing its new head. In this context, which one of the following is incorrect?

    Correct

    Answer: C
    Explanation:
    Option C is incorrect. All other options A, B & D are correct.
    ● Option C: Indian origin Ajay Banga is the nominee from the US who has been awarded with Padma Shri in 2016, NOT Padma Vibhushan by India.
    ● Option A: Generally, a US nominee becomes the World Bank President and the IMF head belongs to the European Union. On the other hand, according to an informal understanding, the Europeans chose to lead the International Monetary Fund, which was also established alongside the World Bank. All Managing Directors of the IMF have been Europeans, with the current one being Bulgaria-born Kristalina Georgieva.
    ● Option B: The selection is done by the World Bank Executive Directors in which the voting rights depends on the share in the Bank’s capital stock held by the members. The US is the single-largest shareholder in the World Bank, with 16.35 per cent of total capital subscription and over 15 per cent of the votes. The United States is also the only country with veto power over certain changes in the Bank’s structure.
    ● India is one of the founder members of the IFC. IFC finances investments with its own resources and by mobilizing capital in the International financial markets. 3.38% of the voting power. The India-elected Executive Director represents a constituency with 99,234 votes, equal to 4.11% of voting power.
    ● Option D: The President of the World Bank Group is also the Chair of the Board of the Executive Directors of the International Bank for Reconstruction and Development (IBRD). The President is also ex officio chair of the Board of Directors of the International Development Association (IDA), International Finance Corporation (IFC), the Multilateral Investment Guarantee Agency (MIGA), and of the Administrative Council of the International Centre for Settlement of Investment Disputes (ICSID).
    ● Ajay Banga has been Selected 14th President of the World Bank for a five-year term beginning June 2, 2023.
    ● All 13 Presidents of the World Bank so far have been US citizens. IMF’s Bulgaria-born chief Kristalina Georgieva, who had become World Bank’s acting president briefly in 2019, is the only exception.
    ● Besides her brief stint at the World Bank as its acting president, the World Bank’s all 13 presidents have been from the US. The IMF chief has also tweeted to share her happiness about Banga’s nomination.
    ● The US is the single-largest shareholder in the World Bank, with 16.35 per cent of total capital subscription and over 15 percent of the votes. The United States is also the only country with veto power over certain changes in the Bank’s structure.
    Candidates for the position of World Bank President should meet the following criteria:
    ● A proven track record of leadership and accomplishment, particularly in development;
    ● Experience of managing large organizations with international exposure, and a familiarity with the public sector;
    ● The ability to articulate a clear vision of the World Bank Group’s development mission;
    ● A firm commitment to and appreciation for multilateral cooperation; and,
    ● Effective communication and diplomatic skills, impartiality and objectivity in the performance of the responsibilities of the position.

    Incorrect

    Answer: C
    Explanation:
    Option C is incorrect. All other options A, B & D are correct.
    ● Option C: Indian origin Ajay Banga is the nominee from the US who has been awarded with Padma Shri in 2016, NOT Padma Vibhushan by India.
    ● Option A: Generally, a US nominee becomes the World Bank President and the IMF head belongs to the European Union. On the other hand, according to an informal understanding, the Europeans chose to lead the International Monetary Fund, which was also established alongside the World Bank. All Managing Directors of the IMF have been Europeans, with the current one being Bulgaria-born Kristalina Georgieva.
    ● Option B: The selection is done by the World Bank Executive Directors in which the voting rights depends on the share in the Bank’s capital stock held by the members. The US is the single-largest shareholder in the World Bank, with 16.35 per cent of total capital subscription and over 15 per cent of the votes. The United States is also the only country with veto power over certain changes in the Bank’s structure.
    ● India is one of the founder members of the IFC. IFC finances investments with its own resources and by mobilizing capital in the International financial markets. 3.38% of the voting power. The India-elected Executive Director represents a constituency with 99,234 votes, equal to 4.11% of voting power.
    ● Option D: The President of the World Bank Group is also the Chair of the Board of the Executive Directors of the International Bank for Reconstruction and Development (IBRD). The President is also ex officio chair of the Board of Directors of the International Development Association (IDA), International Finance Corporation (IFC), the Multilateral Investment Guarantee Agency (MIGA), and of the Administrative Council of the International Centre for Settlement of Investment Disputes (ICSID).
    ● Ajay Banga has been Selected 14th President of the World Bank for a five-year term beginning June 2, 2023.
    ● All 13 Presidents of the World Bank so far have been US citizens. IMF’s Bulgaria-born chief Kristalina Georgieva, who had become World Bank’s acting president briefly in 2019, is the only exception.
    ● Besides her brief stint at the World Bank as its acting president, the World Bank’s all 13 presidents have been from the US. The IMF chief has also tweeted to share her happiness about Banga’s nomination.
    ● The US is the single-largest shareholder in the World Bank, with 16.35 per cent of total capital subscription and over 15 percent of the votes. The United States is also the only country with veto power over certain changes in the Bank’s structure.
    Candidates for the position of World Bank President should meet the following criteria:
    ● A proven track record of leadership and accomplishment, particularly in development;
    ● Experience of managing large organizations with international exposure, and a familiarity with the public sector;
    ● The ability to articulate a clear vision of the World Bank Group’s development mission;
    ● A firm commitment to and appreciation for multilateral cooperation; and,
    ● Effective communication and diplomatic skills, impartiality and objectivity in the performance of the responsibilities of the position.

  3. Question 3 of 5
    3. Question
    2 points

    3. With reference to the Agriculture Infrastructure Fund, consider the following statements:
    1. It is a debt financing facility for investment in projects for post-harvest management infrastructure and community farming assets through interest subvention and credit guarantee.
    2. It is a Central Sector Scheme that will be implemented from FY 2020 to FY 2024.
    3. It seeks to provide interest subvention of 3% per annum up to a limit of Rs. 2 crore.
    4. The fund will be managed and monitored through an online Management Information System (MIS) platform.
    How many of the above statements are correct?

    Correct

    Answer: C
    Explanation:
    ● ₹1 lakh crore Agri Infrastructure Fund for farm-gate infrastructure for farmers was announced on 15.05.2020.
    ● Under the scheme, Rs. 1 Lakh Crore will be provided by banks and financial institutions as loans with interest subvention of 3% per annum and credit guarantee coverage under CGTMSE for loans up to Rs. 2 Crores. Eligible beneficiaries include farmers, FPOs, PACS, Marketing Cooperative Societies, SHGs, Joint Liability Groups (JLG), Multipurpose Cooperative Societies, Agri-entrepreneurs, Start-ups, and Central/State agency or Local Body sponsored Public-Private Partnership Projects.
    ● The Scheme will be operational from 2020-21 to 2032-33. Loan disbursement under the scheme will complete in six years, i.e. by the end of Financial Year 2025-26. As on 31st December 2022, ₹14,118 crores have been sanctioned, out of which ₹9117 crores have been disbursed under the scheme. Remaining ₹90,883 crores out of ₹1 lakh crores will be disbursed during the remaining period between 2022-23 and 2025-26. Repayment period covered under the financing facility will be for a maximum period of 7 years including the moratorium period of up to 2 years.
    ● Eligible Projects: The scheme will facilitate setting up and modernization of key elements of the value chain. Following projects are eligible for all beneficiaries including private entities as well as groups such as FPOs, PACS, SHGs, JLGs, Cooperatives, National and State Level Federation of Co-operatives, FPOs federations, Federations of SHGs, National and State Level Agencies.
    ● Interest subvention for a loan upto ₹2 crore in one location is eligible under the scheme.
    ● Participating institutions: All scheduled commercial banks scheduled cooperative banks, Regional Rural Banks (RRBs), Small Finance Banks, Non-Banking Financial Companies (NBFCs) and National Cooperative Development Corporation (NCDC) may participate to provide this financing facility, after signing of Memorandum of Understanding (MoU) with National Bank for Agriculture & Rural Development (NABARD)/DA&FW.
    ● An online platform will be made available in collaboration with participating lending institutions to provide information and loan sanctioning facilities. Agri Infra fund will be managed and monitored through an online MIS platform.
    ● The National, State and District Level Monitoring Committees to ensure real-time monitoring and effective feed-back about the implementation of the proposed scheme.

    Incorrect

    Answer: C
    Explanation:
    ● ₹1 lakh crore Agri Infrastructure Fund for farm-gate infrastructure for farmers was announced on 15.05.2020.
    ● Under the scheme, Rs. 1 Lakh Crore will be provided by banks and financial institutions as loans with interest subvention of 3% per annum and credit guarantee coverage under CGTMSE for loans up to Rs. 2 Crores. Eligible beneficiaries include farmers, FPOs, PACS, Marketing Cooperative Societies, SHGs, Joint Liability Groups (JLG), Multipurpose Cooperative Societies, Agri-entrepreneurs, Start-ups, and Central/State agency or Local Body sponsored Public-Private Partnership Projects.
    ● The Scheme will be operational from 2020-21 to 2032-33. Loan disbursement under the scheme will complete in six years, i.e. by the end of Financial Year 2025-26. As on 31st December 2022, ₹14,118 crores have been sanctioned, out of which ₹9117 crores have been disbursed under the scheme. Remaining ₹90,883 crores out of ₹1 lakh crores will be disbursed during the remaining period between 2022-23 and 2025-26. Repayment period covered under the financing facility will be for a maximum period of 7 years including the moratorium period of up to 2 years.
    ● Eligible Projects: The scheme will facilitate setting up and modernization of key elements of the value chain. Following projects are eligible for all beneficiaries including private entities as well as groups such as FPOs, PACS, SHGs, JLGs, Cooperatives, National and State Level Federation of Co-operatives, FPOs federations, Federations of SHGs, National and State Level Agencies.
    ● Interest subvention for a loan upto ₹2 crore in one location is eligible under the scheme.
    ● Participating institutions: All scheduled commercial banks scheduled cooperative banks, Regional Rural Banks (RRBs), Small Finance Banks, Non-Banking Financial Companies (NBFCs) and National Cooperative Development Corporation (NCDC) may participate to provide this financing facility, after signing of Memorandum of Understanding (MoU) with National Bank for Agriculture & Rural Development (NABARD)/DA&FW.
    ● An online platform will be made available in collaboration with participating lending institutions to provide information and loan sanctioning facilities. Agri Infra fund will be managed and monitored through an online MIS platform.
    ● The National, State and District Level Monitoring Committees to ensure real-time monitoring and effective feed-back about the implementation of the proposed scheme.

  4. Question 4 of 5
    4. Question
    2 points

    4. Which of the following statements best describes the term ‘J form’, sometimes seen in the news?

    Correct

    Answer: C
    Explanation:
    ● The Punjab Mandi Board has decided to send a ‘digital form J’ on the WhatsApp number of farmers from 2022 Rabi procurement season .
    ● The move, aimed at bringing transparency as well as empowering farmers, will make Punjab the first state in the country to provide digitised form J in real time.
    ● ‘J form’ is the sale receipt of a farmer’s agricultural produce in mandis (grain market). These forms were earlier issued manually by arhtiyas (commission agents) because in Punjab, a majority of farmers sell their crops through such agents only. Also, this form is an income proof for a farmer who sells his crop.
    ● Before digitisation of the J form, several arhtiyas had the habit of keeping these forms with themselves, instead of providing it to the farmers, which was their right. Hence, many farmers ended up having no record of their income after selling their crops.
    ● But by getting digitised J form, farmers will now have a clear record of the crop sold and income received in lieu of that and it will save their time as well, ensuring that they do not need to run to arhtiyas to get copies of their sale.
    ● These forms will be delivered directly on a farmer’s WhatsApp number immediately after a sale is confirmed on the system by the arhtiyas and buyers, who are mainly government procurement agencies.

    Incorrect

    Answer: C
    Explanation:
    ● The Punjab Mandi Board has decided to send a ‘digital form J’ on the WhatsApp number of farmers from 2022 Rabi procurement season .
    ● The move, aimed at bringing transparency as well as empowering farmers, will make Punjab the first state in the country to provide digitised form J in real time.
    ● ‘J form’ is the sale receipt of a farmer’s agricultural produce in mandis (grain market). These forms were earlier issued manually by arhtiyas (commission agents) because in Punjab, a majority of farmers sell their crops through such agents only. Also, this form is an income proof for a farmer who sells his crop.
    ● Before digitisation of the J form, several arhtiyas had the habit of keeping these forms with themselves, instead of providing it to the farmers, which was their right. Hence, many farmers ended up having no record of their income after selling their crops.
    ● But by getting digitised J form, farmers will now have a clear record of the crop sold and income received in lieu of that and it will save their time as well, ensuring that they do not need to run to arhtiyas to get copies of their sale.
    ● These forms will be delivered directly on a farmer’s WhatsApp number immediately after a sale is confirmed on the system by the arhtiyas and buyers, who are mainly government procurement agencies.

  5. Question 5 of 5
    5. Question
    2 points

    5. Which one of the following statements is correct about Neo-banks?

    Correct

    Answer: C
    Explanation:
    ● A neobank is a digital bank that does not have any branches. Instead of having a physical presence at a set location, neobanking is entirely online. A broad collection of financial service providers, who primarily target tech-savvy customers, comes under the umbrella of neobanking. Basically, a neobank is a fintech firm that provides digital and mobile-first services like payments, debit cards, money transfers, lending, and more.
    ● Neobanks bridge the gap between the services that traditional banks offer and the evolving expectations of new-age customers. They do this by providing personalised experiences, employing data-driven insights and offering value-added services.
    ● While traditional banks continue to struggle with bringing their legacy-based infrastructure into the digital age, neobanks leverage its modern digital platforms to analyse customer data and make data-driven decisions. Neobanks can also afford to slash customer fees by a significant amount since they don’t have to bear the expenses of running physical locations.
    ● Among the user experience enhancements that neobanks offer because of their tech-driven nature, is hassle-free account creation.
    ● Since Neobanks don’t have a physical office in neighbourhoods, consumers can create their accounts from their mobile devices from the comfort of their home. And, given their technology-driven KYC process, the account can be ready in just a couple of minutes.
    ● And, how do you avail of a neobank’s service? The same way you order food over Zomato or book an Uber cab. By using an app. Neobanks provide services via its mobile application.
    ● In India, neobanks don’t have a bank license of their own. Instead, they count on bank partners that are regulated to provide bank-licensed services. The likes of Jupiter, Fi, Niyo, and RazorpayX are currently working in partnerships with traditional banks.

    Incorrect

    Answer: C
    Explanation:
    ● A neobank is a digital bank that does not have any branches. Instead of having a physical presence at a set location, neobanking is entirely online. A broad collection of financial service providers, who primarily target tech-savvy customers, comes under the umbrella of neobanking. Basically, a neobank is a fintech firm that provides digital and mobile-first services like payments, debit cards, money transfers, lending, and more.
    ● Neobanks bridge the gap between the services that traditional banks offer and the evolving expectations of new-age customers. They do this by providing personalised experiences, employing data-driven insights and offering value-added services.
    ● While traditional banks continue to struggle with bringing their legacy-based infrastructure into the digital age, neobanks leverage its modern digital platforms to analyse customer data and make data-driven decisions. Neobanks can also afford to slash customer fees by a significant amount since they don’t have to bear the expenses of running physical locations.
    ● Among the user experience enhancements that neobanks offer because of their tech-driven nature, is hassle-free account creation.
    ● Since Neobanks don’t have a physical office in neighbourhoods, consumers can create their accounts from their mobile devices from the comfort of their home. And, given their technology-driven KYC process, the account can be ready in just a couple of minutes.
    ● And, how do you avail of a neobank’s service? The same way you order food over Zomato or book an Uber cab. By using an app. Neobanks provide services via its mobile application.
    ● In India, neobanks don’t have a bank license of their own. Instead, they count on bank partners that are regulated to provide bank-licensed services. The likes of Jupiter, Fi, Niyo, and RazorpayX are currently working in partnerships with traditional banks.

window.wpAdvQuizInitList = window.wpAdvQuizInitList || []; window.wpAdvQuizInitList.push({ id: '#wpAdvQuiz_13', init: { quizId: 13, mode: 1, globalPoints: 10, timelimit: 0, resultsGrade: [0], bo: 0, qpp: 0, catPoints: [10], formPos: 0, lbn: "Finish quiz", json: {"61":{"type":"single","id":61,"catId":0,"points":2,"correct":[0,0,1,0]},"62":{"type":"single","id":62,"catId":0,"points":2,"correct":[0,0,1,0]},"63":{"type":"single","id":63,"catId":0,"points":2,"correct":[0,0,1,0]},"64":{"type":"single","id":64,"catId":0,"points":2,"correct":[0,0,1,0]},"65":{"type":"single","id":65,"catId":0,"points":2,"correct":[0,0,1,0]}} } });




Day-516 | Daily MCQs | UPSC Prelims | POLITY

Day-516

Time limit: 0

Quiz-summary

0 of 5 questions completed

Questions:

  1. 1
  2. 2
  3. 3
  4. 4
  5. 5

Information

To attempt the Quiz, simply click on START Button.

You have already completed the quiz before. Hence you can not start it again.

Quiz is loading...

You must sign in or sign up to start the quiz.

You have to finish following quiz, to start this quiz:

Results

0 of 5 questions answered correctly

Your time:

Time has elapsed

You have reached 0 of 0 points, (0)

Average score
 
 
Your score
 
 

Categories

  1. Not categorized 0%
Your result has been entered into leaderboard
Loading
captcha
  1. 1
  2. 2
  3. 3
  4. 4
  5. 5
  1. Answered
  2. Review
  1. Question 1 of 5
    1. Question
    2 points

    1. Which of the following statements is correct about the Estimates Committee of Parliament?

    Correct

    Answer: C
    Explanation:
    ● Option C is correct. The Estimates Committee cannot question the policies laid down by the Parliament. In general, the Committees don’t have such power to question the policy. Although they can give suggestions for alternative policies or recommend on how to improve policies.
    ● All other options A, B & D are incorrect. As a healthy convention its chairperson is from the RULING party. For the Public Accounts Committee (PAC), as a healthy convention its chairperson is from the opposition party.
    ● The Estimates Committee scrutinizes the budget estimates throughout the year and also only when the demands of grants have been voted.
    ● A minister CANNOT be elected as a member of the Committee. In general, ministers cannot become members of the Committees as they are part of the executive.

    Incorrect

    Answer: C
    Explanation:
    ● Option C is correct. The Estimates Committee cannot question the policies laid down by the Parliament. In general, the Committees don’t have such power to question the policy. Although they can give suggestions for alternative policies or recommend on how to improve policies.
    ● All other options A, B & D are incorrect. As a healthy convention its chairperson is from the RULING party. For the Public Accounts Committee (PAC), as a healthy convention its chairperson is from the opposition party.
    ● The Estimates Committee scrutinizes the budget estimates throughout the year and also only when the demands of grants have been voted.
    ● A minister CANNOT be elected as a member of the Committee. In general, ministers cannot become members of the Committees as they are part of the executive.

  2. Question 2 of 5
    2. Question
    2 points

    2. Natural justice is mainly premised on which one of the following principles?

    Correct

    Answer: C
    Explanation:
    Option C is correct. The natural justice is premised on the fairness principle or justice as fairness.
    Natural justice on the other hand is an expression of English common law, and involves a procedural requirement of fairness. The principles of natural justice have great significance in the study of administrative law. It is also known as substantial justice or fundamental justice or Universal justice or fair play in action.
    The principles of natural justice are not embodied rules and are not codified. They are judge-made rules and are regarded as counterparts of the American procedural due process.
    This principle is based on the following rules:
    i. Fairness in decision
    ii. No one should be a judge in his/her own case
    iii. Everyone should he heard adequately
    The above three principles promote, “Justice should not only be done, but manifestly and undoubtedly be seen to be done”.
    The above rules make it clear that the judiciary must be free from bias and should deliver pure and impartial justice. Judges must act judicially and decide the case without considering anything other than the principles of evidence.
    Option A: Equality is more associated with the concept of the Rule of Law.
    Option B & D: Conflict of interest and merit serve more as means/basis to take fair decisions. Conflict of interest should be avoided as one should not become a judge in one’s own case. Merit/objectivity should work as the basis for the decision. Hence, both promote fairness which becomes the main principle.

    Incorrect

    Answer: C
    Explanation:
    Option C is correct. The natural justice is premised on the fairness principle or justice as fairness.
    Natural justice on the other hand is an expression of English common law, and involves a procedural requirement of fairness. The principles of natural justice have great significance in the study of administrative law. It is also known as substantial justice or fundamental justice or Universal justice or fair play in action.
    The principles of natural justice are not embodied rules and are not codified. They are judge-made rules and are regarded as counterparts of the American procedural due process.
    This principle is based on the following rules:
    i. Fairness in decision
    ii. No one should be a judge in his/her own case
    iii. Everyone should he heard adequately
    The above three principles promote, “Justice should not only be done, but manifestly and undoubtedly be seen to be done”.
    The above rules make it clear that the judiciary must be free from bias and should deliver pure and impartial justice. Judges must act judicially and decide the case without considering anything other than the principles of evidence.
    Option A: Equality is more associated with the concept of the Rule of Law.
    Option B & D: Conflict of interest and merit serve more as means/basis to take fair decisions. Conflict of interest should be avoided as one should not become a judge in one’s own case. Merit/objectivity should work as the basis for the decision. Hence, both promote fairness which becomes the main principle.

  3. Question 3 of 5
    3. Question
    2 points

    3. Which one of the following is incorrect about the Family Courts in India?

    Correct

    Answer: D
    Explanation:
    Option D is incorrect.
    ● The central government does play the role of providing need-based grants or funds as it did in the past. In 2015, The Family Court Scheme was launched as a Centrally Sponsored Scheme for infrastructure facilities for Subordinate Judiciary. The Family Court Scheme under which Rs.2269.43 lakh was given to States as requested for the same, has been discontinued from 2016-17 as this is the responsibility of the State concerned.
    All other options are correct.
    ● Family Courts are part of ADR mechanisms and hence, they have power of civil court and are not supposed to follow the Civil Procedure Code, rather natural justice.
    About family courts
    ● The Family Courts Act, 1984 provides for establishment of Family Courts by the State Governments in consultation with the High Courts with a view to promote conciliation and secure speedy settlement of disputes relating to marriage and family affairs.
    ● Under Section 3 (1)(a) of the Family Courts Act, it is mandatory for the State Government to set up a Family Court for every area in the State comprising a city or a town whose population exceeds one million. In other areas of the States, the Family Courts may be set up if the State Governments deems it necessary.
    ● The main objectives and reasons for setting up of Family Courts are: To create a Specialized Court which will exclusively deal with family matters so that such a court may have the necessary expertise to deal with these cases expeditiously.
    ● The 14th Finance Commission had recommended setting up 235 Family Courts during 2015-2020 in districts where the same were not available. The Commission also urged State Governments to utilize enhanced fiscal space available through tax devolution (32% to 42%) for this purpose.
    ● 763 Family Courts are functional across the country (December 2022).

    Incorrect

    Answer: D
    Explanation:
    Option D is incorrect.
    ● The central government does play the role of providing need-based grants or funds as it did in the past. In 2015, The Family Court Scheme was launched as a Centrally Sponsored Scheme for infrastructure facilities for Subordinate Judiciary. The Family Court Scheme under which Rs.2269.43 lakh was given to States as requested for the same, has been discontinued from 2016-17 as this is the responsibility of the State concerned.
    All other options are correct.
    ● Family Courts are part of ADR mechanisms and hence, they have power of civil court and are not supposed to follow the Civil Procedure Code, rather natural justice.
    About family courts
    ● The Family Courts Act, 1984 provides for establishment of Family Courts by the State Governments in consultation with the High Courts with a view to promote conciliation and secure speedy settlement of disputes relating to marriage and family affairs.
    ● Under Section 3 (1)(a) of the Family Courts Act, it is mandatory for the State Government to set up a Family Court for every area in the State comprising a city or a town whose population exceeds one million. In other areas of the States, the Family Courts may be set up if the State Governments deems it necessary.
    ● The main objectives and reasons for setting up of Family Courts are: To create a Specialized Court which will exclusively deal with family matters so that such a court may have the necessary expertise to deal with these cases expeditiously.
    ● The 14th Finance Commission had recommended setting up 235 Family Courts during 2015-2020 in districts where the same were not available. The Commission also urged State Governments to utilize enhanced fiscal space available through tax devolution (32% to 42%) for this purpose.
    ● 763 Family Courts are functional across the country (December 2022).

  4. Question 4 of 5
    4. Question
    2 points

    4. Which one of the following statements is not correct related to the Constitution (One Hundred and First Amendment) Act that ushers in a GST regime in India?

    Correct

    Answer: C
    Explanation:
    All other options A, B & D are correct. The option C is incorrect. It is due to following factors:
    The Constitution (One Hundred and First Amendment) Act 2016 that ushers in a GST regime in India, gives some role to the President like:
    ● He/she constitutes the GST Council.
    ● He/she can take some measures related to the implementation of GST if there are difficulties in the process.
    Section 20 of the GST law deals with the power of the President to remove difficulties. If any difficulty arises in giving effect to the provisions of the Constitution as amended by this Act the President may, by order, make such provisions, including any adaptation or modification of any provision of the Constitution as amended by this Act or law, as appear to the President to be necessary or expedient for the purpose of removing the difficulty.
    Option A: As per Article 246A, Parliament and state legislatures have the concurrent power to make laws with respect to GST.
    Option B: The GST Council is supposed to function in a harmonised manner to reach a workable fiscal model through cooperation and collaboration. The GST Council is supposed to take decisions on the basis of consensus and if consensus is not reached then it can take decisions by a three-fourth majority.
    Option D: The GST Council shall establish a mechanism to adjudicate any dispute related to its decisions between Union and States as per Section 12 of the law. So far it has not set up any such mechanism. There is a provision for the constitution of a tribunal for dispute resolution.
    The Supreme Court in Union of India versus M/s Mohit Minerals case stated that the decisions of GST Council were not binding on the states which has given rise to a debate on review of the 101st AA 2016.

    Incorrect

    Answer: C
    Explanation:
    All other options A, B & D are correct. The option C is incorrect. It is due to following factors:
    The Constitution (One Hundred and First Amendment) Act 2016 that ushers in a GST regime in India, gives some role to the President like:
    ● He/she constitutes the GST Council.
    ● He/she can take some measures related to the implementation of GST if there are difficulties in the process.
    Section 20 of the GST law deals with the power of the President to remove difficulties. If any difficulty arises in giving effect to the provisions of the Constitution as amended by this Act the President may, by order, make such provisions, including any adaptation or modification of any provision of the Constitution as amended by this Act or law, as appear to the President to be necessary or expedient for the purpose of removing the difficulty.
    Option A: As per Article 246A, Parliament and state legislatures have the concurrent power to make laws with respect to GST.
    Option B: The GST Council is supposed to function in a harmonised manner to reach a workable fiscal model through cooperation and collaboration. The GST Council is supposed to take decisions on the basis of consensus and if consensus is not reached then it can take decisions by a three-fourth majority.
    Option D: The GST Council shall establish a mechanism to adjudicate any dispute related to its decisions between Union and States as per Section 12 of the law. So far it has not set up any such mechanism. There is a provision for the constitution of a tribunal for dispute resolution.
    The Supreme Court in Union of India versus M/s Mohit Minerals case stated that the decisions of GST Council were not binding on the states which has given rise to a debate on review of the 101st AA 2016.

  5. Question 5 of 5
    5. Question
    2 points

    5. Which one of the following is correct about the Attorney General of India?

    Correct

    Answer: D
    Explanation:
    Option D is correct. All other options A, B & C are incorrect. The details about the office of the Attorney General of India are as follows:
    Article 76
    ● The Attorney-General shall hold office during the pleasure of the President, and shall receive such remuneration as the President may determine.
    ● The President shall appoint a person who is qualified to be appointed a Judge of the Supreme Court to be Attorney-General for India.
    ● It shall be the duty of the Attorney-General to give advice to the Government of India upon such legal matters, and to perform such other duties of a legal character, as may from time to time be referred or assigned to him by the President.
    ● The Attorney-General is not debarred from private practice. Can have private practice with the consent of the government.
    Contempt cases and Attorney-General
    Types of contempt of courts: There are two types of contempt of courts- Civil and Criminal
    Section 2(b) provides for Civil contempt and Section 2(c) provides for Criminal contempt.
    The bare act definition of them are-
    “Civil contempt” means wilful disobedience to any judgment, decree, direction, order, writ or other process of a court or wilful breach of an undertaking given to a court.
    “Criminal contempt” means the publication (whether by words, spoken or written, or by signs, or by visible representation, or otherwise) of any matter or the doing of any other act whatsoever which—
    a. scandalises or tends to scandalise, or lowers or tends to lower the authority of, any court; or
    b. prejudices, or interferes or tends to interfere with, the due course of any judicial proceeding; or
    c. interferes or tends to interfere with, or obstructs or tends to obstruct, the administration of justice in any other manner.
    THE PROCESS
    As per the Rule 3 of the Rules to Regulate Proceedings for Contempt of the Supreme Court, 1975, in case of contempt other than the contempt referred to in Rule 2, the Court may take action:
    1. Suo motu; or
    2. On a petition made by Attorney General, or Solicitor General; or
    3. On a petition made by a person, and in the case of a criminal contempt with the consent in writing of the Attorney General or the Solicitor General.
    A bare reading of Rule 3 helps us understand that there are 3 ways for initiating contempt proceedings. The first is suo motu, the second is the petition made by the Attorney General or the Solicitor General, and the third is on the basis of a petition made by any person and where criminal contempt is involved then the consent of the Attorney General or the Solicitor General is necessary.

    Incorrect

    Answer: D
    Explanation:
    Option D is correct. All other options A, B & C are incorrect. The details about the office of the Attorney General of India are as follows:
    Article 76
    ● The Attorney-General shall hold office during the pleasure of the President, and shall receive such remuneration as the President may determine.
    ● The President shall appoint a person who is qualified to be appointed a Judge of the Supreme Court to be Attorney-General for India.
    ● It shall be the duty of the Attorney-General to give advice to the Government of India upon such legal matters, and to perform such other duties of a legal character, as may from time to time be referred or assigned to him by the President.
    ● The Attorney-General is not debarred from private practice. Can have private practice with the consent of the government.
    Contempt cases and Attorney-General
    Types of contempt of courts: There are two types of contempt of courts- Civil and Criminal
    Section 2(b) provides for Civil contempt and Section 2(c) provides for Criminal contempt.
    The bare act definition of them are-
    “Civil contempt” means wilful disobedience to any judgment, decree, direction, order, writ or other process of a court or wilful breach of an undertaking given to a court.
    “Criminal contempt” means the publication (whether by words, spoken or written, or by signs, or by visible representation, or otherwise) of any matter or the doing of any other act whatsoever which—
    a. scandalises or tends to scandalise, or lowers or tends to lower the authority of, any court; or
    b. prejudices, or interferes or tends to interfere with, the due course of any judicial proceeding; or
    c. interferes or tends to interfere with, or obstructs or tends to obstruct, the administration of justice in any other manner.
    THE PROCESS
    As per the Rule 3 of the Rules to Regulate Proceedings for Contempt of the Supreme Court, 1975, in case of contempt other than the contempt referred to in Rule 2, the Court may take action:
    1. Suo motu; or
    2. On a petition made by Attorney General, or Solicitor General; or
    3. On a petition made by a person, and in the case of a criminal contempt with the consent in writing of the Attorney General or the Solicitor General.
    A bare reading of Rule 3 helps us understand that there are 3 ways for initiating contempt proceedings. The first is suo motu, the second is the petition made by the Attorney General or the Solicitor General, and the third is on the basis of a petition made by any person and where criminal contempt is involved then the consent of the Attorney General or the Solicitor General is necessary.

window.wpAdvQuizInitList = window.wpAdvQuizInitList || []; window.wpAdvQuizInitList.push({ id: '#wpAdvQuiz_14', init: { quizId: 14, mode: 1, globalPoints: 10, timelimit: 0, resultsGrade: [0], bo: 0, qpp: 0, catPoints: [10], formPos: 0, lbn: "Finish quiz", json: {"66":{"type":"single","id":66,"catId":0,"points":2,"correct":[0,0,1,0]},"67":{"type":"single","id":67,"catId":0,"points":2,"correct":[0,0,1,0]},"68":{"type":"single","id":68,"catId":0,"points":2,"correct":[0,0,0,1]},"69":{"type":"single","id":69,"catId":0,"points":2,"correct":[0,0,1,0]},"70":{"type":"single","id":70,"catId":0,"points":2,"correct":[0,0,0,1]}} } });




Day-515 | Daily MCQs | UPSC Prelims | CURRENT DEVELOPMENTS

Day-515

Time limit: 0

Quiz-summary

0 of 5 questions completed

Questions:

  1. 1
  2. 2
  3. 3
  4. 4
  5. 5

Information

To attempt the Quiz, simply click on START Button.

You have already completed the quiz before. Hence you can not start it again.

Quiz is loading...

You must sign in or sign up to start the quiz.

You have to finish following quiz, to start this quiz:

Results

0 of 5 questions answered correctly

Your time:

Time has elapsed

You have reached 0 of 0 points, (0)

Average score
 
 
Your score
 
 

Categories

  1. Not categorized 0%
Your result has been entered into leaderboard
Loading
captcha
  1. 1
  2. 2
  3. 3
  4. 4
  5. 5
  1. Answered
  2. Review
  1. Question 1 of 5
    1. Question
    2 points

    1. Consider the following statements regarding the Minerals Security Partnership:
    1. It has been launched by the USA and EU.
    2. It will help reduce the dependency on China for rare earth minerals.
    3. India is a member of this partnership.
    Which of the statements given above is/are correct?

    Correct

    Answer: B
    Explanation:
    In order to break the dominance and reduce dependence on China in mining and processing rare earth minerals, the United States announced the formation of a global alliance called the Mineral Security Partnership (MSP).
    ● Statement 1 is incorrect: First announced in June 2022, the MSP is a new multilateral initiative to bolster critical mineral supply chains essential for the clean energy transition.
    ● Statement 2 is correct:The goal of the MSP is to ensure that critical minerals are produced, processed, and recycled in a manner that supports the ability of countries to realise the full economic development benefit of their geological endowments. The grouping could focus on the supply chains of minerals such as Cobalt, Nickel, Lithium and also the 17 “rare earth” minerals and decrease the dependency on China.
    ● Statement 3 is incorrect: MSP partners include Australia, Canada, Finland, France, Germany, Japan, the Republic of Korea, Sweden, the United Kingdom, the United States, and the European Commission. India is not a member of this partnership.

    Incorrect

    Answer: B
    Explanation:
    In order to break the dominance and reduce dependence on China in mining and processing rare earth minerals, the United States announced the formation of a global alliance called the Mineral Security Partnership (MSP).
    ● Statement 1 is incorrect: First announced in June 2022, the MSP is a new multilateral initiative to bolster critical mineral supply chains essential for the clean energy transition.
    ● Statement 2 is correct:The goal of the MSP is to ensure that critical minerals are produced, processed, and recycled in a manner that supports the ability of countries to realise the full economic development benefit of their geological endowments. The grouping could focus on the supply chains of minerals such as Cobalt, Nickel, Lithium and also the 17 “rare earth” minerals and decrease the dependency on China.
    ● Statement 3 is incorrect: MSP partners include Australia, Canada, Finland, France, Germany, Japan, the Republic of Korea, Sweden, the United Kingdom, the United States, and the European Commission. India is not a member of this partnership.

  2. Question 2 of 5
    2. Question
    2 points

    2. Consider the following statements regarding the FATF:
    1. It was formed by G7 in 2008 after the global financial crisis.
    2. It was mandated since its inception to counter money laundering and terror financing.
    3. Non-profit organisations and civil society organisations provide input into FATF Mutual Evaluations.
    Which of the statements given above is/are correct?

    Correct

    Answer: C
    Explanation:
    Statement 1 is incorrect: The Financial Action Task Force (FATF) was established in 1989 by the G7 to examine and develop measures to combat money laundering. It originally included the G7 countries, the European Commission and eight other countries.
    Statement 2 is incorrect: Initially it was mandated to examine and develop measures to combat money laundering and in 2001, the FATF expanded its mandate to also combat terrorist financing.
    Statement 3 is correct:
    ● FATF mutual evaluations are in-depth country reports analysing the implementation and effectiveness of measures to combat money laundering and terrorist financing. Mutual evaluations are peer reviews, where members from different countries assess another country. A mutual evaluation report provides an in-depth description and analysis of a country’s system for preventing criminal abuse of the financial system as well as focused recommendations to the country to further strengthen its system.
    ● To assist assessment teams in upcoming mutual evaluations, the FATF compiles input from non-profit organisations (NPOs) and other civil society organisations on money laundering and terrorist financing risk and context.
    ● For NPOs—as defined by the FATF—this could include information related to the implementation of risk-based measures to protect the sector from potential terrorist financing abuse. Similarly, input could include experiences of the application of the risk-based measures set out in Recommendation 8, or suggestions to help the assessment team identify which NPOs to gather more input from and/or interview during the on-site visit.

    Incorrect

    Answer: C
    Explanation:
    Statement 1 is incorrect: The Financial Action Task Force (FATF) was established in 1989 by the G7 to examine and develop measures to combat money laundering. It originally included the G7 countries, the European Commission and eight other countries.
    Statement 2 is incorrect: Initially it was mandated to examine and develop measures to combat money laundering and in 2001, the FATF expanded its mandate to also combat terrorist financing.
    Statement 3 is correct:
    ● FATF mutual evaluations are in-depth country reports analysing the implementation and effectiveness of measures to combat money laundering and terrorist financing. Mutual evaluations are peer reviews, where members from different countries assess another country. A mutual evaluation report provides an in-depth description and analysis of a country’s system for preventing criminal abuse of the financial system as well as focused recommendations to the country to further strengthen its system.
    ● To assist assessment teams in upcoming mutual evaluations, the FATF compiles input from non-profit organisations (NPOs) and other civil society organisations on money laundering and terrorist financing risk and context.
    ● For NPOs—as defined by the FATF—this could include information related to the implementation of risk-based measures to protect the sector from potential terrorist financing abuse. Similarly, input could include experiences of the application of the risk-based measures set out in Recommendation 8, or suggestions to help the assessment team identify which NPOs to gather more input from and/or interview during the on-site visit.

  3. Question 3 of 5
    3. Question
    2 points

    3. Consider the following statements regarding Haifa city:
    1. It is a port city in the southwestern Israel.
    2. It opens into the Red Sea through the Gulf of Aqaba.
    3. It is a UNESCO World Heritage Site and a destination for Baháʼí pilgrimage.
    Which of the above given statements is/are correct?

    Correct

    Answer: D
    Explanation:
    The Adani Group’s entry into Israel through its strategic acquisition of the Haifa port will turn this northern coastal Israeli city into a strong and exciting Mediterranean hub.
    ● Statement 1 is incorrect: Haifa city is located in northwestern Israel. The Port of Haifa is the second largest port in Israel in terms of shipping containers and the biggest in shipping tourist cruise ships.
    ● Statement 2 is incorrect: The principal port of the country lies along the Bay of Haifa overlooking the Mediterranean Sea.
    ● Statement 3 is correct: Haifa city is a UNESCO World Heritage Site and a destination for Baháʼí pilgrimage.

    Incorrect

    Answer: D
    Explanation:
    The Adani Group’s entry into Israel through its strategic acquisition of the Haifa port will turn this northern coastal Israeli city into a strong and exciting Mediterranean hub.
    ● Statement 1 is incorrect: Haifa city is located in northwestern Israel. The Port of Haifa is the second largest port in Israel in terms of shipping containers and the biggest in shipping tourist cruise ships.
    ● Statement 2 is incorrect: The principal port of the country lies along the Bay of Haifa overlooking the Mediterranean Sea.
    ● Statement 3 is correct: Haifa city is a UNESCO World Heritage Site and a destination for Baháʼí pilgrimage.

  4. Question 4 of 5
    4. Question
    2 points

    4. Which of the following statements regarding the Global Security Initiative (GSI) is correct?

    Correct

    Answer: C
    Explanation:
    ● In his keynote speech at the Lanting Forum in Beijing on February 21, 2023 the Chinese Minister of Foreign Affairs highlighted the recently unveiled Global Security Initiative (GSI) Concept Paper.
    Option(c) is correct:
    ● A new Global Security Initiative (GSI) was put forward by the Chinese President. The GSI looks to counter the US Indo-Pacific strategy and the Quad (India, US, Australia, Japan grouping). The GSI is presented as a China-led framework that seeks to restore stability and security, particularly in Asia. It outlined five major pillars to effectively implement the GSI, i.e., mutual respect; openness and inclusion; multilateralism; mutual benefit, and a holistic approach.

    Incorrect

    Answer: C
    Explanation:
    ● In his keynote speech at the Lanting Forum in Beijing on February 21, 2023 the Chinese Minister of Foreign Affairs highlighted the recently unveiled Global Security Initiative (GSI) Concept Paper.
    Option(c) is correct:
    ● A new Global Security Initiative (GSI) was put forward by the Chinese President. The GSI looks to counter the US Indo-Pacific strategy and the Quad (India, US, Australia, Japan grouping). The GSI is presented as a China-led framework that seeks to restore stability and security, particularly in Asia. It outlined five major pillars to effectively implement the GSI, i.e., mutual respect; openness and inclusion; multilateralism; mutual benefit, and a holistic approach.

  5. Question 5 of 5
    5. Question
    2 points

    5. Consider the following statements regarding Geographical Indications(GIs):
    1. It is not an individual’s right but a community’s right.
    2. Providing financial support to the GI products is a violation of WTO rules.
    3. It is used to identify unique agricultural and manufactured goods but not natural goods.
    Which of the statements given above is/are correct?

    Correct

    Answer: A
    Explanation:
    The Department for Promotion of Industry and Internal Trade has issued operational guidelines for financial assistance for undertaking the initiatives for the promotion of GIs.
    ● Statement 1 is correct: GI is a right enjoyed by a community / association of producers (community right). For instance, Assam Muga Silk has GI conferring rights to all such traders from that community. Trademark is a right enjoyed by only one person/company (individual right). Any association of persons, producers,organisation or authority established by or under the law can be a registered proprietor. Their name should be entered in the Register of Geographical Indication as registered proprietor for the Geographical Indication.
    ● Geographical Indications of Goods (Registration and Protection) Act,1999 seeks to provide for the registration and better protection of geographical indications relating to goods in India. The Act would be administered by the Controller General of Patents, Designs and TradeMarks- who is the Registrar of Geographical Indications. The Geographical Indications Registry would be located at Chennai.
    ● Statement 2 is incorrect: Providing financial support to the GI products is not a violation of WTO rules. The Department for Promotion of Industry and Internal Trade (DPIIT) -division of the commerce and industry ministry that manages investments and intellectual property rights (IPRs)- has announced that the Indian government will provide financial assistance to eligible agencies for taking initiatives to promote Geographical Indications (GI). Following the announcement, the DPIIT made available the related operational guidelines for the financial support.

    Incorrect

    Answer: A
    Explanation:
    The Department for Promotion of Industry and Internal Trade has issued operational guidelines for financial assistance for undertaking the initiatives for the promotion of GIs.
    ● Statement 1 is correct: GI is a right enjoyed by a community / association of producers (community right). For instance, Assam Muga Silk has GI conferring rights to all such traders from that community. Trademark is a right enjoyed by only one person/company (individual right). Any association of persons, producers,organisation or authority established by or under the law can be a registered proprietor. Their name should be entered in the Register of Geographical Indication as registered proprietor for the Geographical Indication.
    ● Geographical Indications of Goods (Registration and Protection) Act,1999 seeks to provide for the registration and better protection of geographical indications relating to goods in India. The Act would be administered by the Controller General of Patents, Designs and TradeMarks- who is the Registrar of Geographical Indications. The Geographical Indications Registry would be located at Chennai.
    ● Statement 2 is incorrect: Providing financial support to the GI products is not a violation of WTO rules. The Department for Promotion of Industry and Internal Trade (DPIIT) -division of the commerce and industry ministry that manages investments and intellectual property rights (IPRs)- has announced that the Indian government will provide financial assistance to eligible agencies for taking initiatives to promote Geographical Indications (GI). Following the announcement, the DPIIT made available the related operational guidelines for the financial support.

window.wpAdvQuizInitList = window.wpAdvQuizInitList || []; window.wpAdvQuizInitList.push({ id: '#wpAdvQuiz_15', init: { quizId: 15, mode: 1, globalPoints: 10, timelimit: 0, resultsGrade: [0], bo: 0, qpp: 0, catPoints: [10], formPos: 0, lbn: "Finish quiz", json: {"71":{"type":"single","id":71,"catId":0,"points":2,"correct":[0,1,0,0]},"72":{"type":"single","id":72,"catId":0,"points":2,"correct":[0,0,1,0]},"73":{"type":"single","id":73,"catId":0,"points":2,"correct":[0,0,0,1]},"74":{"type":"single","id":74,"catId":0,"points":2,"correct":[0,0,1,0]},"75":{"type":"single","id":75,"catId":0,"points":2,"correct":[1,0,0,0]}} } });